Wahrscheinlichkeitstheorie undStatistik...

146
Wahrscheinlichkeitstheorie und Statistik (wst) Prof. Dr. Marcel Steiner-Curtis 3. September 2020

Transcript of Wahrscheinlichkeitstheorie undStatistik...

Page 1: Wahrscheinlichkeitstheorie undStatistik (wst)web.fhnw.ch/.../Stochastik-Skript-Marcel-Steiner.pdf · Anwendungsgebiete der Statistik sind die Industrie und die Technik, die Physik,

Wahrscheinlichkeitstheorie

und Statistik (wst)

Prof. Dr. Marcel Steiner-Curtis

3. September 2020

Page 2: Wahrscheinlichkeitstheorie undStatistik (wst)web.fhnw.ch/.../Stochastik-Skript-Marcel-Steiner.pdf · Anwendungsgebiete der Statistik sind die Industrie und die Technik, die Physik,

Prof. Dr. Marcel Steiner-CurtisFHNW Fachhochschule NordwestschweizHochschule fur TechnikBahnhofstrasse 6CH-5210 Windisch

[email protected]

Page 3: Wahrscheinlichkeitstheorie undStatistik (wst)web.fhnw.ch/.../Stochastik-Skript-Marcel-Steiner.pdf · Anwendungsgebiete der Statistik sind die Industrie und die Technik, die Physik,

Liebe Studierende

Zum Fachhochschulstudium eines jeden Ingenieurs gehort heutzutage ein Kurs uber Wahr-scheinlichkeitsrechnung und Statistik – kurz Stochastik genannt. Die heutigen Anforderungendes Berufslebens sind mannigfaltig: Einerseits basieren moderne (Software)-Anwendungen aufstochastischen Modellen mit denen Unsicherheiten und zufallige Schwankungen beschriebenwerden. Andererseits mussen Sie in der Lage sein, grosse und zum Teil unubersichtliche Da-tenmengen grafisch darzustellen und zu analysieren. Einfachste stochastische Modelle und dieGrafische Aufbereitung von Daten lernen Sie in diesem Kurs kennen. Mit der Analyse vonDaten werden Sie im Kurs uber Statistische Datenanalyse, [15], vertraut gemacht.

Mein Ziel ist es, Ihnen die Stochastik naher zu bringen, so dass Sie das Gefuhl erhalten,etwas Nutzliches zu lernen, um es spater auch anwenden zu konnen. Dies geht leider nichtohne Ihren personlichen Effort. Das Motto muss auch hier heissen: Ubung macht den Meis-ter. Ich erwarte von Ihnen, dass Sie zusatzlich zu den Vorlesungsstunden noch etwa gleichviel Zeit zu Hause fur die Ubungen und die Nacharbeit des Kurses aufwenden. Einige Auf-gaben und Losungen konnen Sie vom Active Directory der Hochschule fur Technik, FHNWherunterladen.

Das Skriptum basiert im Wesentlichen auf dem Stochastik-Skriptum (vgl. [3]) meinesVorgangers Peter Gschwind, der an der Fachhochschule beider Basel wahrend mehr als dreissigJahren als Dozent tatig war. Hiermit mochte ich Peter herzlich danken, dass er mir auch aufdiesem Gebiet seine Erfahrungen in Form seines Skriptum weitergegeben hat. Weiter habeich einige Teile (vor allem Kapitel 2) zum Teil wortwortlich und zum Teil gekurzt aus demStochastik-Skriptum (vgl. [4]) von Thomas Heim ubernommen.

Sie sind nicht mehr die ersten Studierenden, die mit diesem Skriptum arbeiten. Urtei-len Sie nicht zu hart uber den Autor (und die vorangehenden Leser), wenn Sie Fehler undUngereimtheiten finden, sondern teilen Sie mir diese bitte mit.

3. September 2020, Marcel Steiner-Curtis

i

Page 4: Wahrscheinlichkeitstheorie undStatistik (wst)web.fhnw.ch/.../Stochastik-Skript-Marcel-Steiner.pdf · Anwendungsgebiete der Statistik sind die Industrie und die Technik, die Physik,

ii

Page 5: Wahrscheinlichkeitstheorie undStatistik (wst)web.fhnw.ch/.../Stochastik-Skript-Marcel-Steiner.pdf · Anwendungsgebiete der Statistik sind die Industrie und die Technik, die Physik,

Inhaltsverzeichnis

Liebe Studierende i

Inhaltsverzeichnis iii

1 Einfuhrung 1

1.1 Was ist Stochastik? . . . . . . . . . . . . . . . . . . . . . . . . . . . . . . . . 11.2 Einige Anwendungsgebiete der Statistik . . . . . . . . . . . . . . . . . . . . 11.3 Zur Arbeitsweise in der Statistik . . . . . . . . . . . . . . . . . . . . . . . . 2

1.4 Grundbegriffe der Statistik . . . . . . . . . . . . . . . . . . . . . . . . . . . . 3

2 Beschreibende Statistik 5

2.1 Umgang mit Datenmaterial . . . . . . . . . . . . . . . . . . . . . . . . . . . 52.2 Datensatze . . . . . . . . . . . . . . . . . . . . . . . . . . . . . . . . . . . . . 62.3 Haufigkeitsverteilung . . . . . . . . . . . . . . . . . . . . . . . . . . . . . . . 6

2.3.1 Haufigkeitstabellen . . . . . . . . . . . . . . . . . . . . . . . . . . . . 62.3.2 Histogramme . . . . . . . . . . . . . . . . . . . . . . . . . . . . . . . 7

2.3.3 Kreisdiagramme . . . . . . . . . . . . . . . . . . . . . . . . . . . . . 82.4 Lageparameter . . . . . . . . . . . . . . . . . . . . . . . . . . . . . . . . . . 9

2.4.1 Mittelwert . . . . . . . . . . . . . . . . . . . . . . . . . . . . . . . . . 9

2.4.2 Median und Quartile . . . . . . . . . . . . . . . . . . . . . . . . . . . 102.4.3 Optimumseigenschaften . . . . . . . . . . . . . . . . . . . . . . . . . 11

2.5 Formparameter . . . . . . . . . . . . . . . . . . . . . . . . . . . . . . . . . . 13

2.5.1 Standardabweichung . . . . . . . . . . . . . . . . . . . . . . . . . . . 132.5.2 Quartile und Ausreissergrenzen . . . . . . . . . . . . . . . . . . . . . 15

2.6 Summarische Datendarstellung . . . . . . . . . . . . . . . . . . . . . . . . . 152.6.1 Box- und Whiskersplot . . . . . . . . . . . . . . . . . . . . . . . . . . 15

3 Zufall und Ereignis 17

3.1 Zufall . . . . . . . . . . . . . . . . . . . . . . . . . . . . . . . . . . . . . . . 173.2 Zufallsexperimente und Ereignisse . . . . . . . . . . . . . . . . . . . . . . . . 17

3.3 Verknupfung von Ereignissen . . . . . . . . . . . . . . . . . . . . . . . . . . 183.4 Zusammengesetzte Versuche, Produktregel . . . . . . . . . . . . . . . . . . . 213.5 Permutationen, Variationen, Kombinationen . . . . . . . . . . . . . . . . . . 23

3.5.1 Permutationen . . . . . . . . . . . . . . . . . . . . . . . . . . . . . . 233.5.2 Geordnete Stichprobe mit Zurucklegen . . . . . . . . . . . . . . . . . 233.5.3 Geordnete Stichprobe ohne Zurucklegen - Variation . . . . . . . . . . 24

3.5.4 Ungeordnete Stichprobe ohne Zurucklegen - Kombinationen . . . . . 24

iii

Page 6: Wahrscheinlichkeitstheorie undStatistik (wst)web.fhnw.ch/.../Stochastik-Skript-Marcel-Steiner.pdf · Anwendungsgebiete der Statistik sind die Industrie und die Technik, die Physik,

iv Inhaltsverzeichnis

4 Wahrscheinlichkeit 29

4.1 Theoretische Wahrscheinlichkeit . . . . . . . . . . . . . . . . . . . . . . . . . 29

4.2 Experimentelle Wahrscheinlichkeit . . . . . . . . . . . . . . . . . . . . . . . 32

4.3 Monte-Carlo-Methoden . . . . . . . . . . . . . . . . . . . . . . . . . . . . . . 34

4.4 Wahrscheinlichkeit von zusammengesetzten Ereignissen . . . . . . . . . . . . 36

4.4.1 Der Additionssatz . . . . . . . . . . . . . . . . . . . . . . . . . . . . 36

4.4.2 Der Multiplikationssatz . . . . . . . . . . . . . . . . . . . . . . . . . 37

5 Zufallsgrossen und Wahrscheinlichkeitsverteilungen 43

5.1 Diskrete und stetige Zufallsgrossen . . . . . . . . . . . . . . . . . . . . . . . 43

6 Diskrete Zufallsgrossen und Verteilungen 49

6.1 Erwartungswert und Varianz . . . . . . . . . . . . . . . . . . . . . . . . . . 49

6.2 Die Binomialverteilung . . . . . . . . . . . . . . . . . . . . . . . . . . . . . . 51

6.2.1 Definition und Eigenschaften der Binomialverteilung . . . . . . . . . 51

6.2.2 Erwartungswert und Varianz der Binomialverteilung . . . . . . . . . 54

6.2.3 Die Binomialverteilung beim Testen von Hypothesen . . . . . . . . . 56

6.3 Die Poissonverteilung . . . . . . . . . . . . . . . . . . . . . . . . . . . . . . . 59

6.3.1 Poissonverteilung als Grenzfall der Binomialverteilung . . . . . . . . 59

6.3.2 Erwartungswert und Varianz der Poissonverteilung . . . . . . . . . . 60

7 Stetige Zufallsgrossen und Verteilungen 63

7.1 Stetige Zufallsgrossen und Wahrscheinlichkeitsdichten . . . . . . . . . . . . 63

7.1.1 Die Gleichverteilung . . . . . . . . . . . . . . . . . . . . . . . . . . . 64

7.1.2 Erwartungswert und Varianz . . . . . . . . . . . . . . . . . . . . . . 67

7.2 Die Normalverteilung . . . . . . . . . . . . . . . . . . . . . . . . . . . . . . . 68

7.2.1 Die standardisierte Normalverteilung . . . . . . . . . . . . . . . . . . 68

7.2.2 Die Normalverteilung mit den Parametern µ und σ2 . . . . . . . . . 70

7.2.3 Transformation auf die standardisierte Normalverteilung . . . . . . . 71

7.2.4 Quantile der standardisierten Normalverteilung . . . . . . . . . . . . 72

7.2.5 Anwendung der Normalverteilung in der Fehlerrechnung . . . . . . . 73

7.3 Normalverteilung als Grenzfall der Binomialverteilung . . . . . . . . . . . . 77

8 Statistische Tests 81

8.1 Das Prinzip des statistischen Tests . . . . . . . . . . . . . . . . . . . . . . . 81

8.2 Einseitiger und zweiseitiger Test . . . . . . . . . . . . . . . . . . . . . . . . . 82

8.3 Mogliche Fehler bei statistischen Tests . . . . . . . . . . . . . . . . . . . . . 85

9 Prufen von Erwartungswerten (Parametertests) 89

9.1 Problemstellung der technischen Statistik . . . . . . . . . . . . . . . . . . . 89

9.2 Einstichproben-t-Test, Student-t-Test . . . . . . . . . . . . . . . . . . . . . . 90

9.2.1 Vertrauensintervall fur den Erwartungswert . . . . . . . . . . . . . . 94

9.2.2 Ungefahr erforderlicher Stichprobenumfang . . . . . . . . . . . . . . 96

9.3 Vergleich zweier Mittelwerte unverbundener Stichproben . . . . . . . . . . . 96

9.3.1 Zweistichproben-t-Test bei unbekannten aber gleichen Varianzen . . 97

9.3.2 Zweistichproben-t-Test bei unbekannten Varianzen . . . . . . . . . . 99

9.4 Paarweiser Vergleich bei verbundenen Stichproben . . . . . . . . . . . . . . 100

Page 7: Wahrscheinlichkeitstheorie undStatistik (wst)web.fhnw.ch/.../Stochastik-Skript-Marcel-Steiner.pdf · Anwendungsgebiete der Statistik sind die Industrie und die Technik, die Physik,

v

10 Regressionsrechnung 10310.1 Regressionsgerade . . . . . . . . . . . . . . . . . . . . . . . . . . . . . . . . . 10310.2 Allgemeine Regression . . . . . . . . . . . . . . . . . . . . . . . . . . . . . . 106

11 Regressionsanalyse 11311.1 Allgemeines . . . . . . . . . . . . . . . . . . . . . . . . . . . . . . . . . . . . 11311.2 Regressionsgerade . . . . . . . . . . . . . . . . . . . . . . . . . . . . . . . . . 11511.3 Regressionsanalyse einer Geraden . . . . . . . . . . . . . . . . . . . . . . . . 11611.4 Regressionsanalyse zweier Geraden . . . . . . . . . . . . . . . . . . . . . . . 119

A Mathematische Hilfsmittel 123A.1 Rechnen mit dem Summenzeichen . . . . . . . . . . . . . . . . . . . . . . . 123A.2 Die Regeln von de Morgan . . . . . . . . . . . . . . . . . . . . . . . . . . . . 125A.3 Die Gausssche Fehlerfunktion . . . . . . . . . . . . . . . . . . . . . . . . . . 126

Tafeln 128

Literaturverzeichnis 133

Index 135

Page 8: Wahrscheinlichkeitstheorie undStatistik (wst)web.fhnw.ch/.../Stochastik-Skript-Marcel-Steiner.pdf · Anwendungsgebiete der Statistik sind die Industrie und die Technik, die Physik,

vi Inhaltsverzeichnis

Page 9: Wahrscheinlichkeitstheorie undStatistik (wst)web.fhnw.ch/.../Stochastik-Skript-Marcel-Steiner.pdf · Anwendungsgebiete der Statistik sind die Industrie und die Technik, die Physik,

Kapitel 1

Einfuhrung

1.1 Was ist Stochastik?

Der Begriff Stochastik1 befasst sich mit der Beschreibung und Untersuchung von Vorgangenund Ereignissen aus allen Lebensbereichen, die zufallig oder vom Zufall beeinflusst sind, undzwar im Sinne von nicht vorhersagbar oder nicht exakt vorhersagbar.

Dabei kann die Ursache der Ungewissheit von sehr unterschiedlicher Art sein. Im Allgemeinenhandelt es sich um die prinzipielle oder faktische Unmoglichkeit einer exakten Voraussage, wieetwa bei der Frage nach dem kunftigen Kundenverhalten oder der langfristigen Wettervor-hersage. Es kann aber auch die Unsicherheit des Beobachters bei einem bereits eingetretenenEreignis sein, wie etwa der aktuelle Warenbestand (bei unvollstandiger Information) oder dieSpielkartenwerte in der Hand des Mitspielers.

Diemathematische Stochastik befasst sich mit der mathematischen Beschreibung und Un-tersuchung von Ereignissen, zeitlichen Entwicklungen bzw. raumlichen Strukturen, die vomZufall beeinflusst werden. Solche Ereignisse, Entwicklungen bzw. Strukturen werden oft durchDaten dokumentiert, fur deren Analyse die Statistik2 - ein Teilgebiet der Stochastik - geeig-nete Methoden bereitstellt.

1.2 Einige Anwendungsgebiete der Statistik

Anwendungsgebiete der Statistik sind die Industrie und die Technik, die Physik, die Meteoro-logie, die Geodasie, die Medizin, die Botanik und Zoologie, die Landwirtschaft, das Verkehrs-wesen und viele weitere. In jedem dieser Gebiete ist die Anwendungsmoglichkeit der Statis-tik meist sehr vielgestalt. Beispielsweise bedienen wir uns in der Industrie und Wirtschaftstatistischer Methoden bei der Materialprufung, der Fertigungssteuerung und -regelung, derEnd- und Abnahmekontrolle, der Entwicklung, der Personalauswahl, der Markt- und Pro-duktforschung oder der Planung von Versuchen. Zahlreiche Firmen haben in letzter Zeit furderartige Zwecke besondere statistische Abteilungen eingerichtet oder stehen wenigstens inenger Verbindung mit statistischen Buros, denen die auftretenden Probleme zur Bearbeitungubertragen werden. Heute werden bei den meisten statistischen Untersuchungen Computereingesetzt.

1stochazomai griech. στoχαζoµαι: zielen, vermuten, erraten. Das Adjektiv stochastisch bedeutet soviel wiezufallsbeeinflusst und ist schon seit langem, zuerst wohl in der Physik, fur zufallige Erscheinungen in Gebrauch.

2status lat. Beschreibung des Zustandes eines Gemeinwesens wie der Bevolkerungsgrosse.

1

Page 10: Wahrscheinlichkeitstheorie undStatistik (wst)web.fhnw.ch/.../Stochastik-Skript-Marcel-Steiner.pdf · Anwendungsgebiete der Statistik sind die Industrie und die Technik, die Physik,

2 Kapitel 1. Einfuhrung

1.3 Zur Arbeitsweise in der Statistik

Trotz der grossen Heterogenitat der Anwendungsgebiete der Statistik besteht deren statis-tische Bearbeitung im Allgemeinen aus denselben Schritten. Wir illustrieren diese Schrittedurch ein einfaches Beispiel.Wir wollen die Lebensdauer der in den Raumlichkeiten der Fachhochschule verwendeten Ne-onrohren untersuchen.

1. Formulierung des Problems. Wir entschliessen uns, die durchschnittliche Lebens-dauer von Neonrohren eines bestimmten Typs und Fabrikats zu messen. In einem Lang-zeitversuch unterwerfen wir eine Neonrohre einem klar definierten Ein- und Ausschalt-zyklus. Diesen Versuch fuhren wir so lange durch, bis die Neonrohre als defekt angesehenwerden muss. Dies ergibt uns die Lebensdauer einer Neonrohre.

2. Planung des Experiments. Hatten alle Neonrohren dieselbe Lebensdauer, so brauch-ten wir nur eine einzige zu betrachten und deren Lebensdauer zu bestimmen. Dann kamedie Statistik uberhaupt nicht ins Spiel. Nun zeigt aber die Erfahrung, dass die Lebens-dauer von Neonrohre zu Neonrohre variiert. Wir sollten nun eigentlich alle Neonrohrenhernehmen und untersuchen. Dies ware aber unmoglich, da bei der Bestimmung der Le-bensdauer die Neonrohre zerstort wird und somit keine Neonrohren fur unseren Alltagzur Verfugung stehen wurden. Ausserdem wurde eine solche Vollprufung zu viel kostenund sehr viel Zeit in Anspruch nehmen. Eine Neonrohre ist zu wenig, alle sind zu viel.Wir entschliessen uns also fur den goldenen Mittelweg: Wir greifen einige Neonrohrenganz zufallig heraus und bestimmen deren Lebensdauer.

3. Ausfuhrung des Experimentes. Wir fuhren unseren Plan durch. Der genannte Vor-gang des zufalligen Auswahlens und Messens wird in der Statistik als ein Zufallsex-periment bezeichnet. Der Einfachheit halber wahlen wir nur 11 Neonrohren aus undmessen deren Lebensdauer. Wir erhalten dabei die der Grosse nach sortierten Werte

x1 x2 x3 x4 x5 x6 x7 x8 x9 x10 x11

24 39 45 51 55 62 64 65 67 76 123

[in Monaten]. Diese Werte werden als Stichprobe aus der Grundgesamtheit allermoglichen Messwerte bezeichnet, die bei der Ausfuhrung des Experiments denkbar sind.Ihre Anzahl n = 11 heisst der Stichprobenumfang.

4. Beschreibung der experimentellen Daten. Bei umfangreichen Untersuchungen,bei denen viele Zahlenwerte anfallen, mussen wir nun dafur sorgen, diese in eine uber-sichtliche Form (Tabelle, grafische Darstellung) zu bringen (siehe Kapitel 2). Im Falleunserer Stichprobe entscheiden wir uns fur die Darstellung der 11 Lebensdauern auf ei-ner einfachen Zeitskala und fur einen Box- und Whiskersplot, siehe Abbildung 1.3.i.

Zusatzlich berechnen wir zwei Lageparameter, namlich die durchschnittliche Lebens-dauer

x =1

11(24 + 39 + 45 + 51 + 55 + 62 + 64 + 65 + 67 + 76 + 123) =

671

11= 61.0

und die Standardabweichungs = 25.25,

Page 11: Wahrscheinlichkeitstheorie undStatistik (wst)web.fhnw.ch/.../Stochastik-Skript-Marcel-Steiner.pdf · Anwendungsgebiete der Statistik sind die Industrie und die Technik, die Physik,

1.4. Grundbegriffe der Statistik 3

b b b b b b bb b b b

x1 x2 x3 x4x5

x6

x7

x8

x9 x10 x11

|0

|Q0.25 =45

|Q0.75 =67

|x=62

|Aunten =22

|Aoben =100

b

|Ausreisser

Abbildung 1.3.i: Box- und Whiskersplot der Lebensdauermessung von 11 Neonrohren (sieheKapitel 2.6.1).

die ein Mass dafur ist, wie sehr die Stichprobenwerte um den Mittelwert streuen.

5. Schluss von der Stichprobe auf die Grundgesamtheit. Aus der Stichprobe schlies-sen wir, dass die Grundgesamtheit aller Neonrohren etwa die durchschnittliche Le-bensdauer 61 Monate hat. Es ist moglich, die Genauigkeit dieses Naherungswertes ab-zuschatzen. Dieser Schluss ist insofern nicht ganz unproblematisch, da eine Neonrohreeine sehr grosse Lebensdauer von 123 Monaten hatte. Dieser Messwert verzerrt uns dieganze Analyse (ersichtlich in der grossen Streuung der Messwerte), und wir taten even-tuell gut daran, diesen als Ausreisser zu kennzeichnen. Wir beobachten zudem, dassnach etwa 72 Monaten die meisten Neonrohren kaputt gegangen sind.

Konnen wir die Zuverlassigkeit unseres Naherungswertes fur die Lebensdauer verbes-sern? Ja, indem wir den Stichprobenumfang vergrossern. Es ist aber auch klar, dassdiese Verbesserung durch erhohte Kosten und grosseren Zeitbedarf erkauft wird.

Warum und wie wir derartige Schlusse von der Stichprobe auf die zugehorige Grund-gesamtheit ziehen konnen, ist der Inhalt der schliessenden (induktiven) Statistik,die wir in der anschliessenden Vorlesung uber Statistische Datenanalyse, [15], behandelnwerden. Wir sollten uns auch klar machen, dass es vollkommene sichere Schlusse voneiner Stichprobe auf Grundgesamtheit nicht gibt.

1.4 Grundbegriffe der Statistik

In der Statistik unterscheiden wir zwei wichtige Begriffe:

• Grundgesamtheit nennen wir eine Menge von Elementen, die auf gewisse Merkmalehin untersucht werden sollen. Wir konnen darunter eine Gesamtheit gleichartiger Ele-mente verstehen, z.B. alle Stucke einer bestimmten Produktion oder alle Messwerteeiner Messreihe, die bei standiger Wiederholung desselben Versuchs auftreten konnen.Die Anzahl der Elemente einer Grundgesamtheit kann sehr gross, sogar unendlich sein.

• Um nicht die gesamte Grundgesamtheit auf die betreffenden Merkmale hin untersuchenzu mussen, entnehmen wir ihr eine endliche Teilmenge, eine so genannte Stichprobe3.

3Das Wort stammt aus der Verhuttung von Eisenerz des ausgehenden Mittelalters. Die Huttenleute habenaus dem Schmelzofen eine zufallige Probe genommen, indem Sie mit einem Probeloffel in das flussige Eisenerzhinein stachen und so einen Stich heraus nahmen, um dessen Reinheit und Schmelzzustand zu uberprufen.

Page 12: Wahrscheinlichkeitstheorie undStatistik (wst)web.fhnw.ch/.../Stochastik-Skript-Marcel-Steiner.pdf · Anwendungsgebiete der Statistik sind die Industrie und die Technik, die Physik,

4 Kapitel 1. Einfuhrung

Erfolgt die Auswahl zufallsgemass, d.h., jedes Element der Grundgesamtheit muss diegleiche Chance haben, ausgewahlt zu werden, dann sprechen wir von einer zufalligenStichprobe. Die zufallige Auswahl kann durch Mischen oder blindes Ziehen bzw. durchFestlegung der auszuwahlenden Elemente mit Hilfe von Zufallszahlen erfolgen.

Page 13: Wahrscheinlichkeitstheorie undStatistik (wst)web.fhnw.ch/.../Stochastik-Skript-Marcel-Steiner.pdf · Anwendungsgebiete der Statistik sind die Industrie und die Technik, die Physik,

Kapitel 2

Beschreibende Statistik

Die Aufgabe der beschreibenden (deskriptiven) Statistik besteht darin, eine gegebeneStichprobe zu untersuchen und zu beschreiben. Dies kann mittels so genannter Lageparameter(z. B. Mittelwert, Standardabweichung) oder grafisch geschehen. Mit Hilfe der beschreibendenStatistik erhalten wir aber immer nur Aussagen uber die vorliegende Stichprobe.

2.1 Umgang mit Datenmaterial

Die erste Aufgabe der beschreibenden Statistik besteht im sinnvollen Umgang mit Daten.Einige wichtige Punkte, die vor der Verarbeitung der Daten abgeklart werden mussen:

• Falls die Daten von anderen bereit gestellt wurden, uberlegen wir uns zuerst: Was wurdeeigentlich gemessen und warum? Wie wurden die Daten erhalten?

• Bei selbststandiger Datenbeschaffung stellt sich die Frage: Wie viele Messwerte werdenbenotigt? Unter welchen Bedingungen soll gemessen werden?

• Sind die Daten vorhanden, so prufen wir ihre Struktur: Sind es genug Daten? Sindalle Daten notig? Hat es Ausreisser? Welche Variablen sind naturgegeben, welche sindbeeinflussbar? Welche Variablen sind unabhangig, welche abhangig?

Daten werden nach drei Typen klassifiziert:

1. Stetige oder metrische Grossen, wie etwa Langen und Massen.

2. Diskrete Grossen, z.B. die Anzahl von Produktionsfehlern.

3. Qualitative Merkmale, etwa Haarfarbe oder Wohnort.

Aufgabe

Aufgabe 2.1.1. Zu welchen Kategorien von Daten gehoren die folgenden Grossen? Windge-schwindigkeit, Sonnenscheindauer am Monatsletzten, Anzahl Regentage im April, Luftdruck,Staustunden am Gotthard, Anzahl Lastwagen durch den Belchentunnel, Zivilstand, steuer-bares Einkommen, abgeschlossene Schulbildung.

5

Page 14: Wahrscheinlichkeitstheorie undStatistik (wst)web.fhnw.ch/.../Stochastik-Skript-Marcel-Steiner.pdf · Anwendungsgebiete der Statistik sind die Industrie und die Technik, die Physik,

6 Kapitel 2. Beschreibende Statistik

2.2 Datensatze

Soll ein bestimmtes Merkmal eines Objekts statistisch analysiert werden, so beginnen wir mitdem Aufstellen eines Datensatzes. Der Datensatz ist der Ausgangspunkt fur alle weiterenAnalysen.

Bezieht sich ein Datensatz nur auf eine einzige Eigenschaft, so sprechen wir von eindimen-sionalen Daten. Als Gegensatz bilden zum Beispiel die Wertepaare (Grosse, Gewicht) einerGruppe von Menschen einen zweidimensionalen Datensatz, solche werden wir in diesem Kursnicht betrachten.

Beispiel 2.2.1. Der folgende (eindimensionale) Datensatz zeigt die Zugfestigkeit [in Nmm−2]von 50 zufallig ausgewahlten Proben aus einer Charge Walzdraht:

404 413 390 418 387 418 399 392 399 417 390 384 383 387 389 391411 422 371 369 411 405 408 349 402 378 393 424 403 414 367 407383 401 388 386 427 411 400 412 426 392 402 392 373 390 396 408386 396

Zunachst halten wir die sehr wichtige Erkenntnis fest, dass die rohen Daten zwar alle In-formation enthalten, diese aber (oft) nicht ersichtlich sind! Denken Sie an diesen Grund-satz, wenn Sie bei einer Prasentation Ihren Standpunkt mit harten Fakten untermauern wol-len. Eine Tabelle mit 50 Zahlen gehort vermutlich in ein Messprotokoll, aber sie hat keineUberzeugungskraft. Die dafur benotigte Interpretation der Daten ergibt sich erst aus ihrersinnvollen Komprimierung. Eine extreme Form der Datenkomprimierung besteht in der aus-schliesslichen Angabe des Mittelwertes, im obigen Beispiel 397.28Nmm−2. Einen Datensatzallein durch diese eine Zahl zu reprasentieren ist selten sinnvoll. Leider ist dies oft ublich,aber meistens sehr unbefriedigend, da die Variabilitat der Daten nicht zur Geltung kommt.

2.3 Haufigkeitsverteilung

Leicht zu interpretierende Darstellungen der Variabilitat von Daten sind Haufigkeitstabellen,Histogramme und Kreisdiagramme.

2.3.1 Haufigkeitstabellen

Bei diskreten oder qualitativen Daten bieten sich Haufigkeitstabellen naturlich an.

Beispiel 2.3.1. Anzahl schwerer Storungen an Baumaschinen pro Tag. Bei der Untersuch-ung von Betriebsstorung von Baumaschinen an insgesamt 100 Betriebstagen ergaben sich diefolgenden tabellarisch dargestellten Resultate:

i 0 1 2 3hi 48 38 10 4

Dabei bezeichnen i die Anzahl Storungen pro Tag und hi die Anzahl der Tage mit i Storungen.Zur Kontrolle erhalten wir

3∑

i=0

hi = 100.

Page 15: Wahrscheinlichkeitstheorie undStatistik (wst)web.fhnw.ch/.../Stochastik-Skript-Marcel-Steiner.pdf · Anwendungsgebiete der Statistik sind die Industrie und die Technik, die Physik,

2.3. Haufigkeitsverteilung 7

Fur stetige Daten ist eine Klasseneinteilung sinnvoll. Bei der Wahl der Klassen oder Inter-valle besteht einige Freiheit. Zweckmassigerweise legen wir die Grenzen auf runde Zahlen. DieAnzahl k der Intervalle soll sicher nicht grosser sein als die Halfte des Stichprobenumfangs n,aber auch nicht so klein, dass fast alle Daten in nur ein oder zwei Klassen fallen. Ein Richtwertfur die Anzahl Klassen k ist

k ≈ √n,

aufgerundet auf die nachste ganze Zahl. Dies ist eher als untere Grenze zu betrachten.Die Klassenbreite d ist dann von der Grossenordnung

d ≈ xmax − xmin

k,

wobei xmax und xmin den grossten und den kleinsten Wert der Daten bezeichnen. Wegender bevorzugten Wahl von runden Zahlen als Intervallgrenzen ist diese Beziehung aber nurnaherungsweise erfullt.Die Intervalle selbst werden halboffen gewahlt. Dabei ist es ublich, die auf die Intervallgrenzenfallenden Daten immer konsistent der rechts liegenden (grosseren) Klasse zuzuordnen, so dassdie i-te Klasse als Intervall [ai, ai+1[ fur i ∈ 1, . . . , k aufgefasst wird.

Beispiel 2.3.2. Fur die 50 Zugfestigkeitswerte aus Beispiel 2.2.1 ergibt sich auf diese Weisek ≈

√50 = 7.071. Damit wir schone Klassen mit einer Spannweite von d = 10 bilden konnen,

wahlen wir 9 Klassen. Wir erhalten somit eine Haufigkeitstabelle fur die Zugfestigkeitswerte.

Klasse i [ai, ai+1[ in Nmm−2 Anzahl hi

1 [345, 355[ 12 [355, 365[ 03 [365, 375[ 44 [375, 385[ 45 [385, 395[ 146 [395, 405[ 117 [405, 415[ 98 [415, 425[ 59 [425, 435[ 2

2.3.2 Histogramme

Selbst mit nur mehr als 50 Zahlen ist die Haufigkeitstabelle noch nicht besonders attraktiv.Schneller zu erfassen ist eine grafische Darstellung. Haufigkeitstabellen lassen sich direkt alsHistogramme (Saulendiagramme, engl. Bar charts) darstellen (siehe Abbildungen 2.3.i und2.3.ii): Dabei wird auf der Ordinate die Anzahl hi gegen die Klassen i, bzw. gegen die zu-gehorigen Intervalle, auf der Abszisse aufgetragen. Die Hohe der Saulen ist proportional zurHaufigkeit hi des Auftretens eines Wertes in der Klasse i. Auf den ersten Blick erkennen wir,dass die Haufigkeiten in den beiden Beispielen deutlich verschieden sind. Diese beiden Typenvon Diagrammen werden uns immer wieder begegnen.Beim Erstellen einer Haufigkeitstabelle fur die Zugfestigkeiten haben wir bereits eine gewisseDatenreduktion vorgenommen. Aus der Tabelle oder dem entsprechenden Histogramm konnendie einzelnen Messwerte nicht mehr rekonstruiert werden, nur ihre Klassenhaufigkeiten sinderhalten geblieben.Histogramme sind statistische Naherungen fur Dichtefunktionen, die wir in spateren Kapitelnkennen lernen werden. Sie helfen bei der Auswahl geeigneter Modelle zur Datenverarbeitung.

Page 16: Wahrscheinlichkeitstheorie undStatistik (wst)web.fhnw.ch/.../Stochastik-Skript-Marcel-Steiner.pdf · Anwendungsgebiete der Statistik sind die Industrie und die Technik, die Physik,

8 Kapitel 2. Beschreibende Statistik

0 1 2 3 4

010

2030

4050

Abbildung 2.3.i: Histogramm der Betrieb-storungen von Baumaschinen aus Bei-spiel 2.3.1

340 360 380 400 420 440

0

2

4

6

8

10

12

14

Abbildung 2.3.ii: Histogramm der Zugfes-tigkeit einer Charge Walzdraht aus Bei-spiel 2.2.1

Am Histogramm konnen wir mit einem Blick feststellen, ob die Daten in etwa normalverteiltsind, das heisst, ob das Histogramm die beruhmte Glockenform der Normalverteilung hat.Vergleichen Sie dazu auch die beiden Histogramme in Abbildungen 2.3.i und 2.3.ii.

2.3.3 Kreisdiagramme

Zur Darstellung der Haufigkeitsverteilung von qualitativen Daten eignen sich Kreisdiagramme(engl. Pie Charts) ausgezeichnet.

Beispiel 2.3.3. Die Zusammensetzung einer Kapitalanlage mit niedrigem Risiko und gros-ser Liquiditat kann wie folgt gewahlt werden: 45% Obligationen, 40% Sparkapital und 15%Aktien. Diesen Sachverhalt konnen wir am einfachsten mit einem Kreisdiagramm grafischdarstellen (siehe Abbildung 2.3.iii).

Obligationen

45%

Sparkapital

40% Aktien

15%

Abbildung 2.3.iii: Kreisdiagramm. Die Flachenanteile der Sektoren sind zu den Anteilen pro-portional.

Page 17: Wahrscheinlichkeitstheorie undStatistik (wst)web.fhnw.ch/.../Stochastik-Skript-Marcel-Steiner.pdf · Anwendungsgebiete der Statistik sind die Industrie und die Technik, die Physik,

2.4. Lageparameter 9

Aufgaben

Aufgabe 2.3.1. Der folgende Datensatz stellt die Schalldammzahl [in db] von 10 cm starkenGibsdielenwanden bei 400Hz dar.

25 24 27 24 24 23 23 28 26

Erstellen Sie eine Haufigkeitstabelle und ein Histogramm.

Aufgabe 2.3.2. Erstellen Sie eine Haufigkeitstabelle und ein Histogramm der Zugfestigkeits-daten in Beispiel 2.2.1 mit einer Klassenbreite von d = 5Nmm−2.

Aufgabe 2.3.3. Erstellen Sie ein Kreisdiagramm aus dem das Verhaltnis der Anzahl Stu-dentinnen zur Anzahl Studenten in Ihrer Klasse aufgezeigt wird.

2.4 Lageparameter

Lageparameter charakterisieren in zusammenfassender Weise die Verteilung von Datenwerten.Wichtige Beispiele dafur sind verschiedene Mittelwerte, sowie der Median.

2.4.1 Mittelwert

Das mittlere Verhalten der Daten wird durch den Mittelwert x charakterisiert. Fur den Da-tensatz x1, . . . , xn wird damit meistens das arithmetische Mittel

x =1

n

n∑

i=1

xi

gemeint. Das arithmetische Mittel ist jedoch nicht fur jedes Problem angebracht, wie diefolgenden zwei Beispiele zeigen:

1. Durchschnittsgeschwindigkeit: Ein Automobilist fahrt auf einer 60 km langen Streckedie ersten s1 = 20km mit der Geschwindigkeit v1 = 40kmh−1, die nachsten s2 = 20kmmit v2 = 120 kmh−1 und die letzten s3 = 20km mit v3 = 80kmh−1. Obwohl die dreiTeilstucke gleich lang sind, ergibt sich die Durchschnittsgeschwindigkeit nicht aus demarithmetischen Mittel va =

13(40 + 120+ 80) kmh−1 = 80kmh−1. Wir mussen vielmehr

ausrechnen, wie viel Zeit fur die einzelnen Strecken verbraucht wurde, also die Stundenpro Kilometer si

vi. Die Durchschnittsgeschwindigkeit ergibt sich dann als Gesamtstrecke

dividiert durch Gesamtzeit

v =

3∑

i=1

si

3∑

i=1

si

vi

,

in unserem Fall v = 65.45 kmh−1.

Erfahrungsgemass druckt die kleinste Geschwindigkeit den Durchschnitt also starker,als die grosste ihn anhebt. Wegen der in unserem Beispiel gleich langen Teilstrecken

Page 18: Wahrscheinlichkeitstheorie undStatistik (wst)web.fhnw.ch/.../Stochastik-Skript-Marcel-Steiner.pdf · Anwendungsgebiete der Statistik sind die Industrie und die Technik, die Physik,

10 Kapitel 2. Beschreibende Statistik

erhalten wir die Durchschnittsgeschwindigkeit in der speziellen Form

1

xh=

1

n

n∑

i=1

1

xi.

Diese Art von Mittelwert wird als harmonisches Mittel bezeichnet, und es gilt: DerKehrwert des Mittelwerts ist der Mittelwert der Kehrwerte.

2. Durchschnittsrendite: Ein Aktienkapital K wird uber mehrere Jahre bei schwankendenBorsenkursen angelegt. Die Jahresrendite betragt im ersten Jahr r1 = 8.5%, im zweitenJahr r2 = 12.2% und im dritten Jahr resultiert ein Verlust von r3 = −4.5%. Wiederumist die effektive Durchschnittsrendite nicht einfach das arithmetische Mittel der dreieinzelnen Jahresrenditen. Das Endkapital ergibt sich vielmehr als

K0 = K(1 + r1)(1 + r2)(1 + r3),

was bei dreijahriger Investitionsdauer mit der Durchschnittsrendite r als K ′ = K(1+ r)3

aufzufassen ist. Der durchschnittliche Zinsfaktor ergibt sich zu 1.0515, entsprechendeiner effektiven Jahresrendite von r = 5.15%. Zum Vergleich betragt das arithmetischenMittel ra = 5.4%. Die hier vorliegende Art von Mittelwert

xg = (x1 · · · xn)1n =

(n∏

i=1

xi

) 1n

wird als geometrisches Mittel bezeichnet. Weil beim Logarithmieren aus Produk-ten Summen werden, ist leicht einzusehen, dass fur das geometrische Mittel gilt: DerLogarithmus des Mittelwerts ist der Mittelwert der Logarithmen.

Den bisher besprochenen Arten von Mittelwerten ist gemeinsam, dass sie bei der Auswer-tung jeden einzelnen Datenwert in der gleichen Form berucksichtigen, entweder direkt, alsKehrwert, oder mit seinem Logarithmus. Dieses Vorgehen ist zwar nahe liegend, hat aberauch einen Ernst zu nehmenden Nachteil. Fugen wir namlich zu unserem Datensatz von 50Zugfestigkeitswerten (siehe Beispiel 2.2.1) nur den einen sehr grossen Wert 1408Nmm−2 hin-zu, der vermutlich auf einem Messfehler beruht, so verandert sich das arithmetische Mittelvon urpsrunglich 397.28Nmm−2 auf 417.1Nmm−2. Das arithmetische Mittel ist also nichtrobust gegenuber Ausreissern, was seinen Nutzen als summarischen Lageparameter be-trachtlich mindert.

2.4.2 Median und Quartile

Ein Lageparameter, der gegenuber Ausreissern robust ist, ist der Median x. Darunter verste-hen wir den Datenwert in der Mitte. Um ihn zu ermitteln, mussen wir eine bei den bisherigenBeispielen mehrfach stillschweigend vorgenommene Massnahme verdeutlichen: Schon beimErstellen einer Haufigkeitstabelle, eines Histogramms werden die Datenwerte automatischder Grosse nach sortiert. Dazu ordnen wir die Stichprobenwerte x1, . . . , xn des originalenDatensatzes der aufsteigenden Grosse nach

x(1) ≤ · · · ≤ x(i) ≤ · · · ≤ x(n).

Page 19: Wahrscheinlichkeitstheorie undStatistik (wst)web.fhnw.ch/.../Stochastik-Skript-Marcel-Steiner.pdf · Anwendungsgebiete der Statistik sind die Industrie und die Technik, die Physik,

2.4. Lageparameter 11

Dann ist der Median oder Zentralwert der Stichprobe durch

x =

x(n+1

2 ) wenn n ungerade

12

(x(n

2 )+ x(n

2+1)

)wenn n gerade.

gegeben. Es handelt sich dabei um den mittelsten Zahlenwert der geordneten Stichprobe.Im geordneten Datensatz sind naturlich Maximum und Minimum einfach die Werte an denGrenzen,

xmin = x(1) und xmax = x(n).

Fur die Analyse der Verteilung der Daten sind ferner die empirischen Quartile oder Vier-telwerte Q0.25 und Q0.75 nutzlich. Wahrend der Median die Mitte des Datensatzes charakte-risiert, kennzeichnen Q0.75 das obere und Q0.25 das untere Viertel. Oberhalb von Q0.75 liegenetwa 25% der Datenwerte, unterhalb von Q0.25 ebenfalls etwa 25%.

Beispiel 2.4.1. Es sei die folgende Stichprobe gegeben.

5.3 3.8 4.0 19.5 5.0 4.9 2.2 4.1 3.1 5.5

Wir bilden den Median indem wir die Stichprobe der Grosse nach ordnen

2.2 3.1 3.8 4.0 4.1 4.9 5.0 5.3 5.5 19.5

und x(5) = 4.1 und x(6) = 4.9 ablesen. Dann ist der Median x = 12 (4.1 + 4.9) = 4.5. Im

Gegensatz dazu betragt das arithmetische Mittel x = 5.74. Wir stellen also fest, dass derextrem grosse Wert 19.5 das arithmetische Mittel stark vergrossert, den Median aber nichtbeeinflusst.Weiter lesen wir xmin = 2.2 und xmax = 19.5 ab.Die empirischen Quartile betragen Q0.25 ≈ 3.8 und Q0.75 ≈ 5.3.

2.4.3 Optimumseigenschaften

Als summarische Kenngrosse fur den gesamten Datensatz soll der Mittelwert oder der Mediandie Daten x1, . . . , xn in gewisser Weise optimal reprasentieren. Eine mogliche Forderung aneinen solchen optimalen Lageparameter konnte sein: Wahle als optimalen Reprasentanten x

diejenige Zahl, fur die die Summe der betragsmassigen Abweichungen minimal ist

n∑

i=1

|xi − x| → min .

Diese Eigenschaft hat tatsachlich der Median, also x = x.Eine andere Forderung bestunde in der Minimierung der quadrierten Abweichungen

n∑

i=1

(xi − x)2 → min .

Mit Hilfe der Differenzialrechnung konnen wir leicht zeigen, dass diese Eigenschaft vom arith-metischen Mittel erfullt wird, also x = x. Der Mittelwert x ist der beste Reprasentant derDaten im Sinne der Methode der kleinsten Quadrate, die uns in anderen Vorlesungen(z.B. Analysis IV, [14]) wieder begegnen wird.

Page 20: Wahrscheinlichkeitstheorie undStatistik (wst)web.fhnw.ch/.../Stochastik-Skript-Marcel-Steiner.pdf · Anwendungsgebiete der Statistik sind die Industrie und die Technik, die Physik,

12 Kapitel 2. Beschreibende Statistik

Aufgaben

Aufgabe 2.4.1. Zeigen Sie, dass fur einen gegebenen Datensatz x1, . . . , xn das arithmeti-schen Mittel x = 1

n

∑ni=1 xi die Funktion

f(x) =

n∑

i=1

(xi − x)2

minimiert.

Aufgabe 2.4.2 (Fakultativ). Zeigen Sie, dass fur einen gegebenen Datensatz x1, . . . , xnder Median x = x die Funktion

f(x) =

n∑

i=1

|xi − x|

minimiert.

Losungen

Losung 2.4.1. Bestimmen Sie die Abszisse des Scheitelpunktes der quadratischen Polynom-funktion

f(x) =

n∑

i=1

(xi − x)2

=

n∑

i=1

(x2i − 2xix+ x2)

=n∑

i=1

x2i −n∑

i=1

2xix+n∑

i=1

x2

=

n∑

i=1

x2i − 2x

n∑

i=1

xi + nx2.

Losung 2.4.2. Bei einem Minimum hat die Funktion f eine horizontale Tangente. Alsobestimmen wir die erste Ableitung1 der Funktion f und setzen diese gleich null. Es giltbekanntlich (|x|)′ = sgn(x), wobei sgn die Vorzeichenfunktion bezeichnet. Also folgt

f ′(x) = −n∑

i=1

sgn(xi − x) = 0.

Ist xi > x, dann zahlt die Summe∑n

i=1 sgn(xi − x) eins dazu, sonst eins ab. Somit wird dieSumme

∑ni=1 sgn(xi − x) genau dann null, wenn oberhalb x genau gleich viele xi liegen wie

unterhalb.Fur ungerade n hat f eine eindeutige horizontale Tangente beim Median x = x. Es seix(1) ≤ · · · ≤ x(i) ≤ · · · ≤ x(n) der der Grosse nach geordnete Datensatz. Fur gerade n hat f

fur alle x ∈[x(n

2 ), x(n

2+1)

]eine horizontale Tangente. Der Median x = 1

2

(x(n

2 )+ x(n

2+1)

)

ist das arithmetische Mittel der beiden Intervallgrenzen.Aus geometrischen Grunden ist ersichtlich, dass Sattelpunkt oder Maximum ausgeschlossensind.

1Achtung: Die Funktion f ist an den Stellen x ∈ x1, . . . , xn nicht differenzierbar. Wieso?

Page 21: Wahrscheinlichkeitstheorie undStatistik (wst)web.fhnw.ch/.../Stochastik-Skript-Marcel-Steiner.pdf · Anwendungsgebiete der Statistik sind die Industrie und die Technik, die Physik,

2.5. Formparameter 13

2.5 Formparameter

Ausser dem optimalen Reprasentanten fur den Datensatz x1, . . . , xn sollten wir auch ei-ne Grosse angeben, die in summarischer Form die Schwankungen der Daten, ihre Streuungum den Mittelwert oder Median, charakterisieren. Die einfachste Grosse dieser Art ist dieSpannweite

∆ = xmax − xmin

2.5.1 Standardabweichung

Meistens benutzen wir die empirische Varianz2

s2 =1

n− 1

n∑

i=1

(xi − x)2

Sie heisst auch mittlere quadratische Abweichung, denn es wird eine Art Mittelwert derquadrierten Abweichungen gebildet. Die Grosse s2 charakterisiert die Abweichung der Datenvon ihrem Mittelwert. Grosses s2 bedeutet, dass die Einzeldaten stark vom Mittelwert abwei-chen (um den Mittelwert streuen). Da die Varianz die quadrierte Masseinheit der Datenwertehat, benutzen wir als Variabilitatskenngrosse oft s, die Wurzel aus der Varianz. Wir nennens die Standardabweichung.Zur Berechnung der Stichprobenstreuung brauchen wir den Mittelwert nicht im Voraus zukennen. Ausquadrieren der einzelnen Summanden zeigt, dass gilt

s2 =1

n− 1

n∑

i=1

(xi − x)2

=1

n− 1

n∑

i=1

(x2i − 2xix+ x2)

=1

n− 1

(n∑

i=1

x2i − 2xn∑

i=1

xi + nx2

)

=1

n− 1

(n∑

i=1

x2i − 2x · nx+ nx2

)

=1

n− 1

(n∑

i=1

x2i − nx2

)

=1

n− 1

n∑

i=1

x2i −1

n(n− 1)

(n∑

i=1

xi

)2

.

Wir mussen also nur die Summe der Datenwerte, die Summe ihrer Quadrate und die Anzahlder Datenwerte zuruckbehalten. Wird ein weiterer Datenwert hinzugefugt, so erhohen wir die

2In der Literatur findet sich auch die Formel

s2 =

1

n

n∑

i=1

(xi − x)2.

Fur die Anwendung ist diese Unterscheidung zwecklos, denn als Faustregel muss gelten: Falls es darauf an-kommt, ob durch n− 1 oder n geteilt werden muss, dann ist die Stichprobe sowieso zu klein. . .

Page 22: Wahrscheinlichkeitstheorie undStatistik (wst)web.fhnw.ch/.../Stochastik-Skript-Marcel-Steiner.pdf · Anwendungsgebiete der Statistik sind die Industrie und die Technik, die Physik,

14 Kapitel 2. Beschreibende Statistik

Anzahl n um 1, addieren den Datenwert und sein Quadrat zu den entsprechenden Summen,und erhalten den neuen Mittelwert und die neue Varianz.

Aufgaben

Aufgabe 2.5.1. Mittelwert und Standardabweichung einer Stichprobe von 45 Langenmess-ungen ergaben sich zu x± s = (12.3 ± 1.8)mm.

a. Wie gross ist die Summe der Datenwerte?

b. Wie gross ist die Summe der Quadrate der Datenwerte?

Aufgabe 2.5.2. Ein Datensatz vom Umfang n habe den Mittelwert x = 3.84 und die Stan-dardabweichung s = 0.960. Wird der Datenwert xn+1 = 2.50 zum Datensatz hinzugefugt, soandert sich der Mittelwert zu x′ = 3.82.

a. Wie gross ist n?

b. Wie viel betragt die Standardabweichung s′ fur den erweiterten Datensatz?

Losungen

Losung 2.5.1.

a. 553.5mm

b. 6950.61mm2

Losung 2.5.2.

a. n = 66

b. Es sei n′ = n+ 1. Entwickeln Sie die Formel

s′2 =1

n′ − 1

n′∑

i=1

(xi − x′)2

=1

n

n∑

i=1

((xi − x) + (x− x′)

)2+

1

n(xn+1 − x′)2

=1

n

n∑

i=1

(xi − x)2 + 2(x− x′)1

n

n∑

i=1

(xi − x) +1

n

n∑

i=1

(x− x′)2 +1

n(xn+1 − x′)2

=n− 1

ns2 + 0 + (x− x′)2 +

1

n(xn+1 − x′)2.

Damit ergibt sich die neue Standardabweichung s′ = 0.967.

Page 23: Wahrscheinlichkeitstheorie undStatistik (wst)web.fhnw.ch/.../Stochastik-Skript-Marcel-Steiner.pdf · Anwendungsgebiete der Statistik sind die Industrie und die Technik, die Physik,

2.6. Summarische Datendarstellung 15

2.5.2 Quartile und Ausreissergrenzen

Zusammen mit dem Median haben wir die Quartile eingefuhrt, die mit der Verteilung derDaten verknupft sind. Aus ihnen ergibt sich die Quartilsweite

dQ = Q0.75 −Q0.25.

Aus den Quartilen und der Quartilsweite konnen wir auch sinnvolle Ausreissergrenzen defi-nieren

Aunten = Q0.25 − 1.5 dQ und Aoben = Q0.75 + 1.5 dQ.

Datenwerte, die ausserhalb des Intervalls ]Aunten, Aoben[ liegen, konnen mit einer gewissenBerechtigung als extreme Werte angesehen werden und somit als Ausreisser identifiziertwerden.

2.6 Summarische Datendarstellung

Eine (eher minimale) summarische Darstellung eines Datensatzes gibt nur den Mittelwert x

und die Standardabweichung s mit der korrekten Masseinheit der Daten an

x± s [Masseinheit].

Meistens ist es auch angezeigt, zusatzlich mindestens den Stichprobenumfang n anzugeben.

2.6.1 Box- und Whiskersplot

Eine ausfuhrlichere Darstellung der Variabilitat von Daten erhalten wir mit dem so genanntenBox- und Whiskersplot3, siehe Abbildung 2.6.i.Haben wir einen Datensatz mit mindestens zehn Beobachtungen (Faustregel), dann konnenwir einen solchen Plot erstellen. Dabei wird uber einer Achse, die die Werte des Datensatzesabdeckt, ein Kasten vom unteren Quartil Q0.25 bis zum oberen Q0.75 gezeichnet. In diesemKasten zeigt eine weitere Linie den Median x an, und T-Balken4 (so genannte Whiskers)bezeichnen die Ausreissergrenzen Aunten und Aoben. Eventuell auftretende Ausreisser werdendann individuell mit • eingetragen.

|Q0.25

|Q0.75

|

x

|Aunten

|Aoben

b

|Ausreisser

Abbildung 2.6.i: Box- und Whiskersplot

Der eigentliche Zweck eines solchen Box- und Whiskersplots besteht darin, dass damit sehrschnell verschiedene Datensatze verglichen werden konnen, etwa die Resultate verschiedenerStichproben oder verschiedener Gruppen von Probanden.

3whiskers engl. das Schnurrhaar einer Katze (vgl.”Katzen wurden Whiskas kaufen!“)

4Die Regel fur die Langen der T-Balken ist ist je nach Literatur verschieden. Auch ublich ist es, die grossteBeobachtung, die kleiner ist als Aoben (resp. die kleinste Beobachtung, die grosser ist als Aunten) als Ende desT-Balkens zu definieren (vgl. [13]).

Page 24: Wahrscheinlichkeitstheorie undStatistik (wst)web.fhnw.ch/.../Stochastik-Skript-Marcel-Steiner.pdf · Anwendungsgebiete der Statistik sind die Industrie und die Technik, die Physik,

16 Kapitel 2. Beschreibende Statistik

Aufgabe

Aufgabe 2.6.1. Erstellen Sie einen Box- und Whiskersplot fur die 50 Zugfestigkeitswerteaus Beispiel 2.2.1. Entscheiden Sie weiter, ob der Datensatz Ausreisser hat.

Page 25: Wahrscheinlichkeitstheorie undStatistik (wst)web.fhnw.ch/.../Stochastik-Skript-Marcel-Steiner.pdf · Anwendungsgebiete der Statistik sind die Industrie und die Technik, die Physik,

Kapitel 3

Zufall und Ereignis

3.1 Zufall

Wir sprechen von Zufall, wenn ein Ereignis nicht notwendig oder nicht beabsichtigt auftritt.Umgangssprachlich wird ein Ereignis auch als zufallig bezeichnet, wenn es nicht absehbar, vor-hersagbar oder berechenbar ist. Zufalligkeit und Unberechenbarkeit oder Unvorhersagbarkeitsind jedoch nicht dasselbe.

3.2 Zufallsexperimente und Ereignisse

Unter einem Zufallsexperiment verstehen wir einen Vorgang,

(a) der gedanklich beliebig oft wiederholbar und

(b) dessen Ausgang innerhalb einer Menge moglicher Ergebnisse ungewiss (zufallig)

ist.

Der Begriff Zufallsexperiment ist offensichtlich eine mathematische Idealisierung. Haufig wirdes nicht moglich sein, den gleichen Versuch im wortlichen Sinn beliebig oft zu wiederholen,sondern nur ahnliche Versuche sehr viele Male.

Zu jedem Zufallsexperiment gehort eine Menge S der samtlichen Ausfallsmoglichkeiten (diesewerden auch Stichprobenwerte genannt), die auftreten konnen. Die Menge S heisst Stich-probenraum. Sie kann endlich oder unendlich viele Elemente haben.

Beispiel 3.2.1. Einige Zufallsexperimente und deren Stichprobenraume S.

(a) BeimWerfen einer Munze ist S = Kopf,Zahl, und beimWurfel ist S = 1, 2, 3, 4, 5, 6.

(b) Bei der Ziehung der Lottozahlen ist S = 1, 2, 3, . . . , 45. Beim Ziehen einer Kugel aus ei-ner Urne mit r roten und s schwarzen Kugeln ist S = Kugel ist rot,Kugel ist schwarz.

(c) Messung biologischer Masszahlen, z.B. beim Bestimmen des Alters in einer Populationvon Lebewesen besteht S aus allen Alter der Population.

(d) Bei der Messung einer physikalischen Grosse wie Temperatur oder Masse besteht S ausallen moglichen Messwerten.

17

Page 26: Wahrscheinlichkeitstheorie undStatistik (wst)web.fhnw.ch/.../Stochastik-Skript-Marcel-Steiner.pdf · Anwendungsgebiete der Statistik sind die Industrie und die Technik, die Physik,

18 Kapitel 3. Zufall und Ereignis

Betrachten wir nun einen beliebigen Versuch, so verstehen wir unter dem Begriff Ereigniseine Teilmenge des Stichprobenraums S.

Beispiel 3.2.2. Einige Beispiele von Ereignissen.

(a) Beim Werfen einer Munze interessieren wir uns fur das Ereignis, dass die Munze “Kopf”anzeigt, also A = Kopf ⊂ S = Kopf,Zahl.

(b) Beim Werfen eines Wurfels interessieren wir uns fur die Ereignisse B = 1, 3, 5, dieAugenzahl ist ungerade; oder C = 5, 6, die Augenzahl ist grosser als vier.

(c) Beim Ziehen einer Kugel aus einer Urne mit r roten und s schwarzen Kugeln interessierenwir uns fur das Ereignis: die Kugel ist rot.

Ein Ereignis ist ein moglicher Ausfall eines Versuchs. Er kann eintreten oder nicht.Der Stichprobenraum S kann als spezielles Ereignis aufgefasst werden: das so genannt sichereEreignis. Beim Werfen eines Wurfels ist das sichere Ereignis die Menge aller AugenzahlenS = 1, 2, 3, 4, 5, 6.Andererseits wird ein Ereignis, das uberhaupt nicht eintreten kann, unmogliches Ereignisgenannt und mit ∅ bezeichnet. Beispielsweise ist es unmoglich mit einem herkommlichenWurfel eine gerade und eine ungerade Augenzahl zu wurfeln.

3.3 Verknupfung von Ereignissen

Aus gegebenen Ereignissen konnen durch Verknupfung kompliziertere zusatzliche Ereignissegebildet werden.

(a) Die und-Verknupfung A ∩B ist die Menge der Ereignisse, die sowohl in A als auch inB sind (vgl. Abbildung 3.3.i), vgl. Durchschnittsmenge.

S A BA ∩ B

Abbildung 3.3.i: Die und-Verknupfung A ∩B

Beispiel 3.3.1. Eine Zahl kleiner als 4 zu wurfeln, wird mit dem Ereignis A = 1, 2, 3und eine Zahl grosser als 2 zu wurfeln mit dem Ereignis B = 3, 4, 5, 6 gekennzeichnet.Das Ereignis eine Zahl kleiner als 4 und grosser als 2 zu wurfeln, ist demzufolge

A ∩B = 3.

(b) Die oder-Verknupfung A∪B ist die Menge der Ereignisse, die entweder in A, in B oderin A und B sind (vgl. Abbildung 3.3.ii), vgl. Vereinigungsmenge.

Beispiel 3.3.2. Eine Zahl grosser als 3 zu wurfeln, wird mit dem Ereignis A = 4, 5, 6und eine ungerade Zahl zu wurfeln mit dem Ereignis B = 1, 3, 5 gekennzeichnet. DasEreignis eine Zahl grosser als 3 oder eine ungerade Zahl zu wurfeln, ist demzufolge

A ∪B = 1, 3, 4, 5, 6.

Page 27: Wahrscheinlichkeitstheorie undStatistik (wst)web.fhnw.ch/.../Stochastik-Skript-Marcel-Steiner.pdf · Anwendungsgebiete der Statistik sind die Industrie und die Technik, die Physik,

3.3. Verknupfung von Ereignissen 19

S A BA ∪ B

Abbildung 3.3.ii: Die oder-Verknupfung A ∪B

(c) Das Gegenereignis A zu einem Ereignis A tritt genau dann ein, wenn A nicht eintrittund ist die Menge aller Ereignisse, die nicht zu A gehoren (vgl. Abbildung 3.3.iii).

A

A

S

Abbildung 3.3.iii: Das Gegenereignis A

Wie wir sofort sehen, gilt A = S −A.

Beispiel 3.3.3. Eine Zahl kleiner oder gleich 2 zu wurfeln, wird mit dem EreignisA = 1, 2 gekennzeichnet. Das Ereignis nicht eine Zahl kleiner oder gleich 2 zu wurfeln,ist demzufolge

A = 3, 4, 5, 6.

Aufgaben

Aufgabe 3.3.1. Eine elektrische Schaltung habe folgende Gestalt: Ist der Schalter a auf

bc bc

bc

bc

a

b1

b2

b3

Abbildung 3.3.iv: Elektrische Schaltung

Unterbruch, dann liegt das Ereignis A vor. Sind die Schalter b1 resp. b2 resp. b3 auf Unterbruch,dann liegen die Ereignisse B1 resp. B2 resp. B3 vor. Es sind die Ausdrucke fur die EreignisseC und C anzugeben, wobei C die Unterbrechung des Stromkreises bedeute.

Aufgabe 3.3.2. Drei Bits werden uber einen digitalen Nachrichtenwesen ubertragen. JedesBit kann verfalscht oder richtig empfangen werden.

a. Geben Sie die Menge aller moglichen Ereignisse an. Wie viele Elemente enthalt sie?

b. Es sei Ai = i-tes Bit ist verfalscht, wobei i ∈ 1, 2, 3. Geben Sie A1 an.

Page 28: Wahrscheinlichkeitstheorie undStatistik (wst)web.fhnw.ch/.../Stochastik-Skript-Marcel-Steiner.pdf · Anwendungsgebiete der Statistik sind die Industrie und die Technik, die Physik,

20 Kapitel 3. Zufall und Ereignis

c. Stellen Sie die folgenden Ereignisse mit Hilfe von Mengenoperationen mit den Ai dar:

B1 = alle Bits sind verfalscht,B2 = mindestens ein Bit ist verfalscht,B3 = genau ein Bit ist verfalscht.

d. Beschreiben Sie verbal folgende Ereignisse:

C1 = A1 ∩ (A2 ∩A3),

C2 = (A1 ∩A2 ∩A3) ∪ (A1 ∩ A2 ∩A3) ∪ (A1 ∩A2 ∩ A3).

Aufgabe 3.3.3. Was bedeuten A ∪A und A ∩ A?

Aufgabe 3.3.4. Wann gilt die Gleichung A ∩B = A?

Aufgabe 3.3.5. Das Ereignis A liege vor, wenn von vier Werkstucken mindestens einesAusschuss ist, B trifft ein, sobald mindestens zwei der vier Werkstucke Ausschuss sind. Wasbedeuten die Ereignisse A und B?

Aufgabe 3.3.6. Das Ereignis A liege vor, wenn von drei gepruften Geraten mindestens einesAusschuss ist. Das Ereignis B trifft ein, wenn alle drei Gerate einwandfrei sind. Was bedeutendie Ereignisse A ∪B und A ∩B?

Aufgabe 3.3.7. Zwei Schachspieler spielen eine Partie. A bedeute: es siegt der erste Spieler.B bedeute: es siegt der zweite Spieler. Welches Ereignis mussen wir noch zu den beidenhinzufugen, um das sichere Ereignis zu erhalten?

Aufgabe 3.3.8. Eine Anlage besteht aus zwei Kesseln und einer Maschine. Ist die Maschineintakt, dann liege das Ereignis A vor. Ist der erste (resp. zweite) Kessel arbeitsfahig, so liegedas Ereignis B1 (resp. B2) vor. Es bezeichne C das Ereignis: die Anlage ist arbeitsfahig, diegewahrleistet ist, wenn die Maschine und mindestens ein Kessel intakt sind. Drucken Sie dieEreignisse C und C durch die Ereignisse A, B1 und B2 aus.

Aufgabe 3.3.9. Ein Gerat besteht aus zwei Blocken vom Typ A und drei Blocken vom TypB. Es bezeichne A1 (resp. A2) das Ereignis: der erste (resp. zweite) Block vom Typ A istintakt. Es bezeichne B1 (resp. B2, B3) das Ereignis: der erste (resp. zweite, dritte) Blockvom Typ B ist intakt. Das Gerat arbeitet, bezeichnet mit Ereignis C, wenn mindestens einBlock vom Typ A und nicht weniger als zwei Blocke vom Typ B intakt sind. Drucken Sie dieEreignisse C und C durch die Ereignisse A1, A2 und B1, B2, B3 aus.

Losungen

Losung 3.3.1. Es bedeutet zum Beispiel (0, 1, 0, 1), dass die Schalter a unterbrochen, b1, b3geschlossen und b2 unterbrochen sind. Dann gilt

C = A ∪ (B1 ∩B2 ∩B3) = (0, ∗, ∗, ∗), (1, 0, 0, 0) : wobei ∗ ∈ 0, 1C = A ∩ (B1 ∪ B2 ∪ B3)

= (1, 1, 1, 1), (1, 1, 1, 0), (1, 1, 0, 1), (1, 0, 1, 1), (1, 1, 0, 0), (1, 0, 1, 0), (1, 0, 0, 1)

unter Ausnutzung der Regeln von de Morgan (siehe Kapitel A.2).

Page 29: Wahrscheinlichkeitstheorie undStatistik (wst)web.fhnw.ch/.../Stochastik-Skript-Marcel-Steiner.pdf · Anwendungsgebiete der Statistik sind die Industrie und die Technik, die Physik,

3.4. Zusammengesetzte Versuche, Produktregel 21

Losung 3.3.2.

a. (V, V, V ), (V, V,R), (V,R, V ), (R,V, V ), (V,R,R), (R,V,R), (R,R, V ), (R,R,R)Die Menge besteht aus 23 = 8 Elemente

b. A1 = (V, V, V ), (V, V,R), (V,R, V ), (V,R,R)

c. B1 = A1 ∩A2 ∩A3,B2 = A1 ∪A2 ∪A3 undB3 = (A1 ∩ A2 ∩ A3) ∪ (A1 ∩A2 ∩ A3) ∪ (A1 ∩ A2 ∩A3).

d. Das Ereignis C1 bedeutet: Erstes Bit verfalscht und von den anderen hochstens einesverfalscht.Das Ereignis C2 bedeutet: Genau ein Bit richtig.

Losung 3.3.3. A ∪A = A und A ∩ A = ∅Losung 3.3.4. A ∩B = A impliziert A ⊆ B.

Losung 3.3.5. Das Ereignis A bedeutet, dass keine Werkstuck Ausschuss ist. Das EreignisB bedeutet, dass hochstens ein Werkstuck Ausschuss ist.

Losung 3.3.6. Das Ereignis A∪B = S bedeutet, dass alles moglich ist. Das Ereignis A∩B = ∅bedeutet, dass nichts moglich ist.

Losung 3.3.7. Remis1

Losung 3.3.8. C = A ∩ (B1 ∪B2) und C = A ∪ (B1 ∩ B2)

Losung 3.3.9.

C = (A1 ∪A2) ∩ ((B1 ∩B2) ∪ (B2 ∩B3) ∪ (B3 ∩B1))

C = (A1 ∩ A2) ∪ ((B1 ∪ B2) ∩ (B2 ∪ B3) ∩ (B3 ∪ B1))

3.4 Zusammengesetzte Versuche, Produktregel

Im Folgenden betrachten wir Versuche, die aus Teilversuchen zusammengesetzt sind. Fur dasWeitere ist es wichtig, eine vollstandige Ubersicht uber die Anzahl der moglichen Ausfalle beisolchen zusammengesetzten Versuchen zu erhalten.

Beispiel 3.4.1. Wenn wir zwei Mal hintereinander einen Wurfel werfen, handelt es sich dabeium einen zusammengesetzten Versuch. Es treten folgende Moglichkeiten auf:

1. Wurf hat 6 Moglichkeiten mit Stichprobenraum S1 = 1, 2, 3, 4, 5, 62. Wurf hat 6 Moglichkeiten mit Stichprobenraum S2 = 1, 2, 3, 4, 5, 6

1Kann ein Spieler keinen regelgemassen Zug mehr ausfuhren und steht sein Konig nicht im Schach, sospricht man von einem Patt, die Partie endet in diesem Fall remis (unentschieden). Ein Spiel endet auchdann remis, wenn es durch keine Zugkombination und ungeschicktestem Spiel des Gegners mehr moglich ist,Matt zu setzen, wenn sich die beiden Spieler darauf einigen, wenn 50 Zuge lang keine Figur geschlagen undkein Bauer bewegt wurde und ein Spieler dies reklamiert, bei dreimaliger Wiederholung derselben Stellungmit demselben Spieler am Zug und den gleichen Zugmoglichkeiten, wenn ein Spieler dies reklamiert, wenn dieZeit beider Spieler abgelaufen ist und es nicht nachweisbar ist, wer die Zeit zuerst uberschritten hat. Quelle:http://de.wikipedia.org

Page 30: Wahrscheinlichkeitstheorie undStatistik (wst)web.fhnw.ch/.../Stochastik-Skript-Marcel-Steiner.pdf · Anwendungsgebiete der Statistik sind die Industrie und die Technik, die Physik,

22 Kapitel 3. Zufall und Ereignis

Eine vollstandige Ubersicht erhalten wir mit einem Ereignisbaum.

1. Teilversuch

2. Teilversuch

1

1 2 3 4 5 6

2

1 2 3 4 5 6

3

1 2 3 4 5 6

4

1 2 3 4 5 6

5

1 2 3 4 5 6

6

1 2 3 4 5 6

Dieser Ereignisbaum stellt dar, dass zu jeder Zahl im ersten Teilversuch jede Zahl im zwei-ten Teilversuch kombiniert werden kann. Daraus ergeben sich 6 · 6 = 36 Moglichkeiten vonverschiedenen Ausfallen beim zusammengesetzten Versuch. Der Stichprobenraum beim zwei-maligen Wurfeln ist demzufolge

S = (1, 1), (1, 2), (1, 3), . . . , (1, 6), (2, 1), (2, 2), (2, 3), . . . , (6, 6).

Beispiel 3.4.2. Wenn wir vier Mal hintereinander eine Munze werfen, haben wir bei jedemWurf die Moglichkeit Kopf K oder Zahl Z zu erhalten.

Eine vollstandige Ubersicht erhalten wir wiederum mit dem Ereignisbaum.

1. Teilversuch

2. Teilversuch

3. Teilversuch

4. Teilversuch

K

K

K

K Z

Z

K Z

Z

K

K Z

Z

K Z

Z

K

K

K Z

Z

K Z

Z

K

K Z

Z

K Z

Aus diesem Ereignisbaum ergeben sich 2 · 2 · 2 · 2 = 24 = 16 Ausfallsmoglichkeiten beimzusammengesetzten Versuch. Der Stichprobenraum ist demzufolge

S = (K,K,K,K), (K,K,K,Z), (K,K,Z,K), . . . , (Z,Z,Z,Z).

Dieses Prinzip lasst sich verallgemeineren.

Produktregel

Besteht ein zusammengesetzter Versuch aus m unabhangigen Teilversuchen mit jeweils

n1, n2, n3, . . . , nm

Ausfallsmoglichkeiten, so besitzt der zusammengesetzte Versuch

n1 · n2 · n3 · · ·nm

mogliche Ausfalle.

Page 31: Wahrscheinlichkeitstheorie undStatistik (wst)web.fhnw.ch/.../Stochastik-Skript-Marcel-Steiner.pdf · Anwendungsgebiete der Statistik sind die Industrie und die Technik, die Physik,

3.5. Permutationen, Variationen, Kombinationen 23

3.5 Permutationen, Variationen, Kombinationen

3.5.1 Permutationen

Haben wir eine Anzahl verschiedener Ziffern oder Buchstaben oder allgemeiner Zeichen, sokonnen wir uns fragen, auf wie viele Arten wir diese zu einem Wort anordnen konnen.

Beispiel 3.5.1. Gegeben seien die Ziffern 1, 2, 3, 4 und 5. Nun haben wir funf Platze mit funfZeichen auszufullen. Die Aufgabe wird auf die Produktregel zuruckgefuhrt. Das Setzen einesZeichens auf einen Platz stellt einen Teilversuch dar.

1. Teilversuch © 1 © © © 5 Moglichkeiten2. Teilversuch © 1 © © 2 4 Moglichkeiten3. Teilversuch © 1 3 © 2 3 Moglichkeiten4. Teilversuch 4 1 3 © 2 2 Moglichkeiten5. Teilversuch 4 1 3 5 2 1 Moglichkeit

Beim 1. Teilversuch sind noch alle Platze frei, und wir haben genau funf Moglichkeiten dieerste Ziffer 1 zu setzen; beim 2. Teilversuch bleiben noch vier Moglichkeiten offen, die zweiteZiffer 2 zu setzen; etc. Beim letzten 5. Teilversuch haben wir keine Wahlmoglichkeit mehroffen und sind gezwungen die Ziffer 5 auf den freien Platz zu setzen.Die Produktregel ergibt nun

P (5) = 1 · 2 · 3 · 4 · 5 = 5! = 120

Ausfallsmoglichkeiten. Wir konnen somit 5 verschiedene Zahlen auf 5! Arten anordnen oder,wie wir auch sagen, permutieren (vertauschen), so dass jeweils verschiedene 5-stellige Zahlenentstehen.

Allgemein ergibt sich in vollig analoger Art die Permutation von n Elementen. Es ist moglichn Elemente auf

P (n) = n!

verschiedene Moglichkeiten anzuordnen.

3.5.2 Geordnete Stichprobe mit Zurucklegen

Wir betrachten ein typisches Beispiel.

Beispiel 3.5.2. Es seien n nummerierte Lose in einer Urne. Es werde k mal ein Los gezogenund dessen Nummer notiert. Dann wird das Los wieder in die Urne gelegt. Wir erhalten somitals Ereignis eine Anzahl von k Nummern in einer bestimmten Reihenfolge. Wie viele Ausfalleverschiedener Art sind moglich?

1. Teilversuch © © . . . © © n Moglichkeiten2. Teilversuch © © . . . © © n Moglichkeiten3. Teilversuch © © . . . © © n Moglichkeiten

......

.... . .

......

...k. Teilversuch © © . . . © © n Moglichkeiten

Bei allen k Teilversuchen haben wir n Wahlmoglichkeiten. Die Produktregel ergibt nun

V (k, n) = nk

Ausfallsmoglichkeiten.

Page 32: Wahrscheinlichkeitstheorie undStatistik (wst)web.fhnw.ch/.../Stochastik-Skript-Marcel-Steiner.pdf · Anwendungsgebiete der Statistik sind die Industrie und die Technik, die Physik,

24 Kapitel 3. Zufall und Ereignis

3.5.3 Geordnete Stichprobe ohne Zurucklegen - Variation

In dieser Situation sprechen wir auch von Variation von n Elementen zur k-ten Klasse.

Der Versuch lauft analog wie in Kapitel 3.5.2, mit der Ausnahme, dass die Lose nicht zuruck-gelegt werden. Wir haben also k Teilversuche mit n, dann mit n − 1, n − 2, . . . , n − k + 1Ausfallsmoglichkeiten. Die Produktregel ergibt in diesem Fall

V (k, n) = n(n− 1)(n − 2) · · · (n− k + 1) =n!

(n− k)!

Ausfallsmoglichkeiten.

3.5.4 Ungeordnete Stichprobe ohne Zurucklegen - Kombinationen

In dieser Situation sprechen wir auch vonKombinationen von n Elementen zur k-ten Klasse.

Ein typisches Beispiel zu dieser Art Stichprobe ist folgendes: Es seien n Kugeln gleicherFarbe in einer Urne, und es werden k Kugeln ohne Zurucklegen gezogen. Da die Kugeln nichtunterscheidbar sind, kann keine Reihenfolge berucksichtigt werden.

Dieser Fall ergibt sich aus dem Fall in Kapitel 3.5.3, wenn wir die Reihenfolge nicht beruck-sichtigen. Das bedeutet, dass wir die Anzahl der moglichen Anordnungen von k Elementen,d.h. k! Moglichkeiten, unberucksichtigt lassen mussen. Die Anzahl der moglichen Ausfallereduziert sich also um diesen Faktor. Wir erhalten in diesem Fall

C(k, n) =n!

k!(n − k)!=

(n

k

)

Ausfallsmoglichkeiten.

Aufgaben

Aufgabe 3.5.1. Eine Autonummer bestehe in der folgenden Reihenfolge aus

a. einem Buchstaben und zwei Ziffern,

b. zwei Buchstaben und zwei Ziffern,

c. zwei Buchstaben und drei Ziffern.

Wie viele Autonummern sind jeweils moglich? (Das Alphabet hat 26 Buchstaben und Auto-nummern mit den Zeichenfolgen 00 (resp. 000 in Aufgabe (c)) sind auszuschliessen.)

Aufgabe 3.5.2. Auf wie viele Arten kann ein Totozettel bei 13 Partien ausgefullt werden?

Aufgabe 3.5.3. Frau Meier hat 4 Kleider, 3 Hute und 5 Paar Schuhe. Auf wie viele Artenkann sie sich zum Ausgehen anziehen, wenn alles zueinander passt und das Tragen eines Hutes

a. Pflicht,

b. freiwillig

ist?

Page 33: Wahrscheinlichkeitstheorie undStatistik (wst)web.fhnw.ch/.../Stochastik-Skript-Marcel-Steiner.pdf · Anwendungsgebiete der Statistik sind die Industrie und die Technik, die Physik,

3.5. Permutationen, Variationen, Kombinationen 25

Aufgabe 3.5.4. Ein Wurfel wird 3 mal geworfen. Wie viele mogliche Ausfalle hat der Ver-such?

Aufgabe 3.5.5. Ein Munze wird 8 mal geworfen. Welcher Bruchteil der moglichen Ausfalleenthalt Kopf und Zahl gleich oft?

Aufgabe 3.5.6. Gegeben seien die folgenden 15 Zeichen.

a. a1, a2, a3, a4, b1, b2, b3, b4, b5, c1, c2, c3, c4, c5, c6

b. a, a, a, a, b1, b2, b3, b4, b5, c1, c2, c3, c4, c5, c6

c. a, a, a, a, b, b, b, b, b, c1 , c2, c3, c4, c5, c6

d. a, a, a, a, b, b, b, b, b, c, c, c, c, c, c

Wie viele Anordnungen gibt es jeweils?

Aufgabe 3.5.7. Mit den Ziffern 1 bis 9 soll eine 4-stellige Zahl gebildet werden. Auf wieviele Arten geht dies, wenn Ziffernwiederholungen

a. erlaubt,

b. verboten

sind?

Aufgabe 3.5.8. Auf wie viele Arten konnen die Buchstaben der Worter

a. BERLIN

b. PFEFFER

c. MISSISSIPPI

d. OBERRHEINDAMPFSCHIFFFAHRTSKAPITANSMUTZE

permutiert werden?

Aufgabe 3.5.9. Auf wie viele unterscheidbare Arten konnen 16 Felder eingefarbt werden?

a. Wenn jedes Feld nach freier Wahl schwarz oder weiss gefarbt wird?

b. Wenn 8 Felder weiss und 8 Felder schwarz gefarbt werden?

c. Wenn 2 Felder weiss, 4 schwarz und 10 rot gefarbt werden?

d. Wenn jedes Feld mit einer anderen von 16 Farben gefarbt wird?

Aufgabe 3.5.10. Eine Klasse hat 15 Fussballspieler, einer davon heisst Klaus. Auf wie vieleArten kann eine Mannschaft von 11 Spielern

a. mit Klaus,

b. ohne Klaus

zusammengestellt werden.

Page 34: Wahrscheinlichkeitstheorie undStatistik (wst)web.fhnw.ch/.../Stochastik-Skript-Marcel-Steiner.pdf · Anwendungsgebiete der Statistik sind die Industrie und die Technik, die Physik,

26 Kapitel 3. Zufall und Ereignis

Aufgabe 3.5.11. Aus n Personen ist ein Ausschuss von r, wobei 0 < r < n, Personen undim Ausschuss ein Vorsitzender zu wahlen. Auf wie viele Arten geht dies?

Aufgabe 3.5.12. Wie viele Personen befinden sich in einer Gesellschaft, wenn beim Anstos-sen 253 mal die Glaser klirren?

Aufgabe 3.5.13. Wie viele Wurfbilder gibt es beim Kegeln prinzipiell?

Aufgabe 3.5.14. Wie viele Moglichkeiten gibt es, die 36 Jasskarten auf vier Spieler A, B,C und D zu verteilen?

Aufgabe 3.5.15. Wie viele Spielverteilungen gibt es beim Schieber, bei denen ein Spieleralle vier Bauern erhalt?

Aufgabe 3.5.16. Herr Meier hat 7 Sorten Wein im Keller. Fur eine Party benotigt er 3Flaschen Wein von derselben Sorte. Die Sorte selbst ist ihm gleichgultig. Wegen eines Defektesan der Kellerbeleuchtung muss er die Flaschen im Dunkeln heraus greifen. Wie viele Flaschenmuss er mindestens aus dem Keller mitnehmen, damit sicher drei Flaschen von der gleichenSorte darunter sind?

Aufgabe 3.5.17. In einem Parlament sind 3 Parteien vertreten: 60 Liberale, 40 Konservativeund 30 Sozialisten.

a. Wie viele zehner-Kommissionen lassen sich mit dem Verteilschlussel funf-drei-zwei bil-den?

b. Wie viele siebner-Kommissionen lassen sich mit dem Verteilschlussel drei-zwei-zwei bil-den?

Aufgabe 3.5.18. Wie viele dreistellige Zahlen aus lauter verschiedenen Ziffern gibt es imDezimalsystem?

Aufgabe 3.5.19. Wie viele echte (ohne fuhrende Nullen) siebenstellige Zahlen konnen ausden Ziffern 1, 2, 3, 3, 0, 0, 0 gebildet werden?

Aufgabe 3.5.20. Wie viele Moglichkeiten gibt es, mindestens eine Sechs zu erhalten, wenngleichzeitig funf Wurfel geworfen werden?

Aufgabe 3.5.21. Wie viele aller moglichen Tippreihen beim Toto mit 13 Partien enthaltenk richtige?

Losungen

Losung 3.5.1.

a. 2 574

b. 66 924

c. 675 324

Losung 3.5.2. 1 594 323

Page 35: Wahrscheinlichkeitstheorie undStatistik (wst)web.fhnw.ch/.../Stochastik-Skript-Marcel-Steiner.pdf · Anwendungsgebiete der Statistik sind die Industrie und die Technik, die Physik,

3.5. Permutationen, Variationen, Kombinationen 27

Losung 3.5.3.

a. 60

b. 80

Losung 3.5.4. 216

Losung 3.5.5. 70 : 256

Losung 3.5.6.

a. 1 307 674 368 000

b. 54 486 432 000

c. 454 053 600

d. 630 630

Losung 3.5.7.

a. 6 561

b. 3 024

Losung 3.5.8.

a. 720

b. 420

c. 34 650

d. 379 511 635 419 893 750 000 000 000 000 000 000 000 ≈ 3.795 · 1038

Losung 3.5.9.

a. 65 536

b. 12 870

c. 120 120

d. 20 922 789 888 000

Losung 3.5.10.

a. 1 001

b. 364

Losung 3.5.11. N = r(nr

)Moglichkeiten

Losung 3.5.12. 23

Page 36: Wahrscheinlichkeitstheorie undStatistik (wst)web.fhnw.ch/.../Stochastik-Skript-Marcel-Steiner.pdf · Anwendungsgebiete der Statistik sind die Industrie und die Technik, die Physik,

28 Kapitel 3. Zufall und Ereignis

Losung 3.5.13. Ein Kegelspiel hat in der Regel 9 Kegel, also gibt es 512 verschiedene Wurf-bilder.

Losung 3.5.14. 21 452 752 266 265 320 000 ≈ 21 · 1018

Losung 3.5.15. 45 888 240 141 744 000 ≈ 46 · 1015

Losung 3.5.16. Er wahlt 15 Flaschen.

Losung 3.5.17.

a. 23 472 486 273 600 ≈ 23 · 1012

b. 11 610 846 000 ≈ 12 · 109

Losung 3.5.18. 648

Losung 3.5.19. 240

Losung 3.5.20. 4 651

Losung 3.5.21. Die Anzahl Gewinnmoglichkeiten bei k richtigen betragt

G(k) =

(13

k

)1k213−k.

Zur Kontrolle gilt∑13

k=0

(13k

)1k213−k = (1 + 2)13 = 313, vgl. Losung 3.5.2.

Page 37: Wahrscheinlichkeitstheorie undStatistik (wst)web.fhnw.ch/.../Stochastik-Skript-Marcel-Steiner.pdf · Anwendungsgebiete der Statistik sind die Industrie und die Technik, die Physik,

Kapitel 4

Wahrscheinlichkeit

4.1 Theoretische Wahrscheinlichkeit

Die theoretische Wahrscheinlichkeit ergibt sich aus der rein gedanklichen Analyse derverschiedenen Ausfallsmoglichkeiten bei einem gegebenen Versuch. Ist von vornherein klar,dass alle Moglichkeiten die gleiche Chance haben aufzutreten, so lasst sich sofort angeben,wie gross das Verhaltnis des betrachteten Ereignisses zu allen Ereignissen in bezug auf dieMoglichkeit des Auftretens ist. Dazu bilden wir den Quotienten der Anzahl der gunstigenAusfalle zur Anzahl dermoglichen Ausfalle. Wir erhalten damit eine Zahl, die in Bezug auf einbestimmtes Ereignis angibt, wie gross seine Chance, d.h. wie gross die Wahrscheinlichkeitist, dass das Ereignis eintrifft. Wir definieren deshalb die Wahrscheinlichkeit eines Ereignissesdurch

p =Anzahl der gunstigen Falle

Anzahl der moglichen Falle=

g

m.

Beispiel 4.1.1. Wie gross ist die Wahrscheinlichkeit, beim Wurfeln eine Sechs zu erhalten?Es gibt m = 6 mogliche Falle und g = 1 gunstige Falle. Damit ist die Wahrscheinlichkeit

p =1

6.

Beispiel 4.1.2. Wie gross ist die Wahrscheinlichkeit, beim Werfen einer Munze Kopf zuerhalten? Es gibt m = 2 mogliche Falle und g = 1 gunstige Falle. Damit ist die Wahrschein-lichkeit

p =1

2.

Beispiel 4.1.3. Gegeben seien 10 Nusse, davon seien 3 verdorben. Wie gross ist die Wahr-scheinlichkeit, dass zwei gute Nusse mit einem Griff genommen werden? Es gibt m =

(102

)

mogliche Falle und g =(72

)gunstige Falle. Damit ist die Wahrscheinlichkeit

p =

(72

)(102

) =7

15.

Beispiel 4.1.4. Beim Schweizer Zahlenlotto 6 aus 45 gibt es fur den Sechser einen einzigengunstigen Fall, bei

(456

)= 8145 060 moglichen Fallen. Die Gewinnchancen fur einen Sechser

sind also

p =1

8145 060= 0.000 000 123.

29

Page 38: Wahrscheinlichkeitstheorie undStatistik (wst)web.fhnw.ch/.../Stochastik-Skript-Marcel-Steiner.pdf · Anwendungsgebiete der Statistik sind die Industrie und die Technik, die Physik,

30 Kapitel 4. Wahrscheinlichkeit

Aufgaben

Aufgabe 4.1.1. Wie gross ist die Wahrscheinlichkeit

a. aus einer Urne mit 100 Gewinnlosen und 400 Nieten einen Gewinn zu ziehen?

b. aus 36 Jasskarten ein As zu ziehen?

c. mit einem idealen Wurfel mehr als 2 zu wurfeln?

Aufgabe 4.1.2. Wie gross ist die Wahrscheinlichkeit, bei sechsmaligem Wurfeln mit einemidealen Wurfel

a. keine Sechs,

b. lauter verschiedene Augenzahlen

zu erhalten?

Aufgabe 4.1.3. Von 10 Nussen seien 4 verdorben. Wie gross ist die Wahrscheinlichkeit, miteinem Griff blindlings 3 gute heraus zu greifen?

Aufgabe 4.1.4. Wie gross ist die Wahrscheinlichkeit, dass 7 beliebige Schuler an 7 verschie-denen Wochentagen geboren sind, wenn angenommen wird, dass alle Wochentage gleichbe-rechtigt sind?

Aufgabe 4.1.5. Wie gross ist die Wahrscheinlichkeit, dass bei 4 Wurfen mit einem symme-trischen Wurfel mindestens einmal eine Sechs auftritt?

Aufgabe 4.1.6. Unter 20 Gluhbirnen sind 6 unbrauchbar. Es werden 6 gepruft. Wie grossist die Wahrscheinlichkeit, dabei 6 gute zu finden?

Aufgabe 4.1.7. Wie gross ist die Wahrscheinlichkeit, dass eine Familie mit n Kindern genauk Madchen hat, wenn Knaben und Madchengeburten gleich wahrscheinlich sind?

Aufgabe 4.1.8. Ein Kartenspiel enthalte 36 Karten von vier verschiedenen Farben. NachHerausziehen und Zurucklegen einer Karte des Spiels wird das Spiel neu gemischt, und eswird noch einmal eine Karte gezogen. Bestimmen Sie die Wahrscheinlichkeit dafur, dass beideKarten von der gleichen Farbe sind.

Aufgabe 4.1.9. Ein Buchstabenschloss besitze funf um eine Achse drehbare Ringschei-ben, von denen jede in sechs Sektoren mit verschiedenen Buchstaben eingeteilt ist. DasSchloss offnet sich nur in dem Falle, wo jede Ringscheibe eine bestimmte Lage bezuglich desSchlosskorpers einnimmt. Bestimmen Sie die Wahrscheinlichkeit fur das Offnen des Schlosses,wenn eine beliebige Buchstabenkombination eingestellt wird.

Aufgabe 4.1.10. Zehn Bucher werden willkurlich in ein Regal gestellt. Bestimmen Sie dieWahrscheinlichkeit dafur, dass dabei drei bestimmte Bucher nebeneinander gestellt werden?

Aufgabe 4.1.11. Funf Strecken seien gegeben, deren Lange jeweils 1, 3, 5, 7, 9 Einheitenbetragen. Bestimmen Sie die Wahrscheinlichkeit dafur, dass mit drei aus den funf willkurlichausgewahlten Strecken ein Dreieck gebildet werden kann.

Page 39: Wahrscheinlichkeitstheorie undStatistik (wst)web.fhnw.ch/.../Stochastik-Skript-Marcel-Steiner.pdf · Anwendungsgebiete der Statistik sind die Industrie und die Technik, die Physik,

4.1. Theoretische Wahrscheinlichkeit 31

Aufgabe 4.1.12. Von zehn Losen gewinnen zwei. Bestimmen Sie die Wahrscheinlichkeitdafur, dass sich unter funf willkurlich ausgewahlten Losen

a. genau ein Gewinnlos befindet.

b. beide Gewinnlose befinden.

c. mindestens ein Gewinnlos befindet.

Aufgabe 4.1.13. Es liegen m + n Lose vor, unter denen n Gewinnlose sind. Es werden k

Lose auf einmal gezogen. Bestimmen Sie die Wahrscheinlichkeit dafur, dass sich unter den k

Losen genau s Gewinnlose befinden.

Aufgabe 4.1.14. Zur Verringerung der Spielanzahl werden 2n Mannschaften in zwei gleichgrosse Gruppen eingeteilt. Bestimmen Sie die Wahrscheinlichkeit dafur, dass die beiden spiel-starksten Mannschaften

a. in verschiedenen Gruppen

b. in der gleichen Gruppe

sind.

Aufgabe 4.1.15. In einer Sendung von Maschinenteilen seien n einwandfreie und m minder-wertige. Wie gross ist die Wahrscheinlichkeit, dass wenn beim heraus greifen von einer AnzahlTeile die ersten k Teile einwandfrei sind, dann auch das (k + 1)-te Teil einwandfrei ist?

Losungen

Losung 4.1.1.

a. p = 15

b. p = 19

c. p = 23

Losung 4.1.2.

a. p = 0.335

b. p = 0.015

Losung 4.1.3. p = 0.167

Losung 4.1.4. p = 0.006

Losung 4.1.5. p = 0.518

Losung 4.1.6. p = 0.077

Losung 4.1.7. p =(nk)2n

Losung 4.1.8. p = 14

Page 40: Wahrscheinlichkeitstheorie undStatistik (wst)web.fhnw.ch/.../Stochastik-Skript-Marcel-Steiner.pdf · Anwendungsgebiete der Statistik sind die Industrie und die Technik, die Physik,

32 Kapitel 4. Wahrscheinlichkeit

Losung 4.1.9. p = 0.00013

Losung 4.1.10. p = 115

Losung 4.1.11. p = 0.3

Losung 4.1.12.

a. p = 59

b. p = 29

c. p = 79

Losung 4.1.13. Es gibt m =(m+nk

)mogliche Ausfalle und genau g =

(mk−s

)(ns

)Moglichkeiten

fur s Gewinnlose. Dabei zahlt der Faktor(

mk−s

)die Moglichkeiten, k−s Nieten zu haben; und

der Faktor(ns

)zahlt die Moglichkeiten, s Treffer zu haben. Damit folgt

p(s) =

(mk−s

)(ns

)(m+nk

) .

Losung 4.1.14.

a. p = n2n−1

b. q = 1− p = n−12n−1

Losung 4.1.15. p = n−kn+m−k

4.2 Experimentelle Wahrscheinlichkeit

Betrachten wir die relative Haufigkeit der Ausfalle Kopf beim Munzenwerfen bei steigenderWurfzahl. So zeigt sich, dass bei einer fairen Munze etwa die Halfte aller Ausfalle Kopfzeigen. Wir erhalten somit einen Wert 1

2 als eine Art Grenzwert der empirischen relativenHaufigkeit (siehe Abbildung 4.2.i). Dieser Wert kann als Eigenschaft der Munze aufgefasstwerden, wie Gewicht und Farbe. Das experimentelle Bestimmen der relativen Haufigkeitenheisst Simulation.

Allgemeine Formulierung der experimentellen Wahrscheinlichkeit

Gegeben sei ein Stichprobenraum S = s1, s2, . . . , sn eines Versuchs. Zu jedem Ausfall si ∈ S

gehort eine relative Haufigkeit

h(si) =Anzahl des Auftretens von si

Anzahl Versuche N

seines Auftretens. Dabei gilt

0 ≤ h(si) ≤ 1 und h(s1) + · · · + h(sn) = 1.

Betrachten wir nun ein spezielles Ereignis A = si1 , si2 , . . . , sik ⊆ S, so erhalten wir dierelative Haufigkeit des Eintretens von A mit

h(A) = h(si1) + h(si2) + · · ·+ h(sik).

Page 41: Wahrscheinlichkeitstheorie undStatistik (wst)web.fhnw.ch/.../Stochastik-Skript-Marcel-Steiner.pdf · Anwendungsgebiete der Statistik sind die Industrie und die Technik, die Physik,

4.2. Experimentelle Wahrscheinlichkeit 33

0 20 40 60 80 100

0.0

0.2

0.4

0.6

0.8

1.0

Anzahl Würfe

h(K

opf)

Abbildung 4.2.i: Beim Werfen einer fairen Munze strebt die relative Haufigkeit h(Kopf) desAuftretens von Kopf mit steigender Anzahl Wurfe gegen 1

2 .

Ferner gilt allgemeinh(S) = 1 und h(∅) = 0.

Nehmen wir nun grosse Werte fur die Anzahl Versuche N , so weichen im Allgemeinen die h

nicht viel von den entsprechenden Wahrscheinlichkeiten ab. Es gilt

P

limN→+∞

h(si) = p(si) = pi.

Dabei ist zu beachten, dassP

lim nicht einen gewohnlichen Grenzwert darstellt, sondern einenGrenzwert in Wahrscheinlichkeit, d.h. die Wahrscheinlichkeit, dass h(si) gegen pi strebt, strebtmit N → +∞ gegen 1.

Beispiel 4.2.1. Beim Wurfelwerfen hatten wir zum Beispiel A = 1, 3, 5 und demzufolgeware

h(A) = h(1) + h(3) + h(5)

und damit im Grenzubergang

p(A) = p(1) + p(3) + p(5) =1

6+

1

6+

1

6=

1

2.

Aus den Eigenschaften der relativen Haufigkeit erhalten wir die der Wahrscheinlichkeit. Fureinen Ausfall s ∈ S bezeichne p(s) die Wahrscheinlichkeit fur dessen Auftreten. Die Zuordnung

s 7−→ p(s)

heisst Wahrscheinlichkeitsverteilung.

Beispiel 4.2.2. Der Versuch bestehe aus einmaligem Wurfeln mit einem symmetrischenWurfel. Die Wahrscheinlichkeit der einzelnen Ausfalle seien:

Page 42: Wahrscheinlichkeitstheorie undStatistik (wst)web.fhnw.ch/.../Stochastik-Skript-Marcel-Steiner.pdf · Anwendungsgebiete der Statistik sind die Industrie und die Technik, die Physik,

34 Kapitel 4. Wahrscheinlichkeit

Ausfalle 1 2 3 4 5 6

Wahrscheinlickeiten 16

16

16

16

16

16

Die Wahrscheinlichkeitsverteilung ist eine Funktion, die jedem Ereignis eine Wahrscheinlich-keit im Intervall [0, 1] zuordnet, so dass die Summe aller Wahrscheinlichkeiten eins ergibt.

Axiome der Wahrscheinlichkeit

Es sei S = s1, s2, . . . , sn der Stichprobenraum. Die folgenden Eigenschaften werden alsAxiome der Wahrscheinlichkeit genommen.

1. 0 ≤ p(si) ≤ 1 fur alle si ∈ S

2. p(S) = p(s1) + · · ·+ p(sn) = 1

3. p(A) = p(si1) + p(si2) + · · ·+ p(sik) fur eine Teilmenge A = si1 , si2 , . . . , sik von S

Die Axiome der Wahrscheinlichkeit wurden zum ersten Mal von Andrey Nikolaevich Kolmo-goroff, 1903-1987, formuliert (siehe Abbildung 4.2.ii).

Abbildung 4.2.ii: Andrey Nikolaevich Kolmogoroff, 1903-1987

4.3 Monte-Carlo-Methoden

Die experimentelle Wahrscheinlichkeit, d.h. die Wahrscheinlichkeit als eine Art Grenzwertder relativen Haufigkeit eines Ereignisses, hat in den letzten Jahren stark an Bedeutunggewonnen. Durch den Einsatz von Computern lasst sich schnell und relativ einfach die relativeHaufigkeit von simulierbaren, also auf dem Rechner mathematisch durchfuhrbaren Ereignissenbestimmen. Die Zahl der Versuche kann sehr hoch angesetzt werden, so dass wir stabileAussagen uber die betreffende Wahrscheinlichkeit machen konnen. Solche Methoden werdenMonte-Carlo-Methoden genannt. Sie haben in den meisten Bereichen der Technik und derWissenschaft Einzug gehalten.An einem einfachen Beispiel wollen wir das prinzipielle Vorgehen erlautern.

Beispiel 4.3.1. Wir wollen das bestimmte Integral

A =

∫ a

0f(x)dx

Page 43: Wahrscheinlichkeitstheorie undStatistik (wst)web.fhnw.ch/.../Stochastik-Skript-Marcel-Steiner.pdf · Anwendungsgebiete der Statistik sind die Industrie und die Technik, die Physik,

4.3. Monte-Carlo-Methoden 35

berechnen.Geometrisch handelt es sich um die Berechnung der Masszahl der Flache unter der Kurvey = f(x) uber dem Intervall [0, a] (siehe Abbildung 4.3.i). Wir bestimmen eine Anzahl N

a

b

y = f(x)

b

Pi(xi, yi)

A

Q

y

x

Abbildung 4.3.i: Masszahl der Flache unter der Kurve y = f(x) uber dem Intervall [0, a]

zufalliger Punkte P1(x1, y1), . . . , PN (xN , yN ) im Rechteck Q = [0, a]× [0, b].Das geschieht mit Hilfe von Zufallszahlen, die wir auf jedem Rechner zur Verfugung haben.Wir wahlen eine erste Zufallszahl x1 im Intervall [0, a] und eine zweite Zufallszahl y1 in [0, b].Dies ergibt uns einen ersten zufalligen Punkt P1(x1, y1) im Rechteck Q. Dieses Prozederefuhren wir nun N mal durch. Dann haben wir N zufallige Punkte P1(x1, y1), . . . , PN (xN , yN )bestimmt, die zufallig verteilt im Rechteck Q liegen. Einige Punkte werden nun oberhalb derKurve y = f(x) und andere unterhalb dieser liegen.Die Wahrscheinlichkeit, dass ein Punkt in der Flache mit der gesuchten Masszahl liegt, ist

p =A

area(Q).

Bestimmen wir N zufallige Punkte, ist die relative Haufigkeit

h =n

N

der n Punkte, die in der zu berechnenden Flache liegen, zur gesamten Zahl N eine guteSchatzung fur p. Daraus lasst sich A naherungsweise berechnen

A = p · area(Q) ≈ n

Narea(Q).

Wir haben somit nur rechnerisch zu bestimmen, ob ein Punkt Pi(xi, yi) eine Ordinate

yi < f(xi)

hat. Trifft dies zu, muss ein Zahler um eins erhoht werden. Sind alle N Punkte getestet, dannergibt sich das gesuchte Verhaltnis n

N.

Nach dieser Methode lassen sich auch mehrfache Integrale naherungsweise berechnen.Monte-Carlo-Methoden sind im Allgemeinen sehr einfach durchzufuhren. Allerdings ist ihreGenauigkeit fur kleine Versuchszahlen N gering. Die Genauigkeit in diesem Beispiel erhohtsich proportional zu

√N . Das heisst, um eine Dezimalstelle zu gewinnen, braucht es 100 mal

mehr Versuche.

Page 44: Wahrscheinlichkeitstheorie undStatistik (wst)web.fhnw.ch/.../Stochastik-Skript-Marcel-Steiner.pdf · Anwendungsgebiete der Statistik sind die Industrie und die Technik, die Physik,

36 Kapitel 4. Wahrscheinlichkeit

4.4 Wahrscheinlichkeit von zusammengesetzten Ereignissen

4.4.1 Der Additionssatz

Satz 4.4.1 (Additionssatz). Sind die Ereignisse A und B elementfremd, d.h., es gilt A∩B =∅, dann gilt fur die Wahrscheinlichkeiten

p(A ∪B) = p(A) + p(B).

Sind sie nicht elementfremd, dann gilt die allgemeinere Version

p(A ∪B) = p(A) + p(B)− p(A ∩B).

Die folgenden Beispiele illustrieren den Additionssatz.

Beispiel 4.4.1 (Geometrische Wahrscheinlichkeit). Wir schiessen zufallig auf eine rechteckigeZielscheibe (vgl. Abbildung 4.4.i) und fragen nach der Wahrscheinlichkeit, entweder die FlacheA oder B zu treffen.

S A BA ∩ B

Abbildung 4.4.i: Geometrische Wahrscheinlichkeit

Die Wahrscheinlichkeit p(A) ins Gebiet A zu treffen ist nichts anderes als das Verhaltnis desFlacheninhalts von A zum gesamten Flacheninhalt von S und analog fur die Flache B. Somitfolgt

p(A) =area(A)

area(S)und p(B) =

area(B)

area(S).

Mit dem Additionssatz 4.4.1 folgt nun

p(A ∪B) = p(A) + p(B)− p(A ∩B) =area(A) + area(B)− area(A ∩B)

area(S).

Wir sehen, dass die Flache des Durchschnittes abgezogen werden muss, da sie sonst doppeltgezahlt wurde.

Beispiel 4.4.2. Wir werfen einen fairen Wurfel und betrachten die Ereignisse A = 1, 3, 5und B = 1, 2, 3. Es gilt A ∪ B = 1, 2, 3, 5 und A ∩ B = 1, 3. Uns interessiert dieWahrscheinlichkeit dafur, dass das Ereignis A ∪ B eintritt. Wir erhalten also mit Hilfe desAdditionssatzes

p(A ∪B) = p(A) + p(B)− p(A ∩B)

=

(1

6+

1

6+

1

6

)+

(1

6+

1

6+

1

6

)−(1

6+

1

6

)

=4

6.

Page 45: Wahrscheinlichkeitstheorie undStatistik (wst)web.fhnw.ch/.../Stochastik-Skript-Marcel-Steiner.pdf · Anwendungsgebiete der Statistik sind die Industrie und die Technik, die Physik,

4.4. Wahrscheinlichkeit von zusammengesetzten Ereignissen 37

Beispiel 4.4.3. Ist A das Gegenereignis von A, also A∪ A = S und A∩ A = ∅, dann gilt mitdem Additionssatz 4.4.1

p(A) + p(A) = p(A ∪ A) = p(S) = 1

oder etwas anders geschrieben

p(A) = 1− p(A). (4.4.a)

4.4.2 Der Multiplikationssatz

Oft sind Ereignisse aus unabhangigen Teilereignissen zusammengesetzt, deren Wahrschein-lichkeiten mit der bekannten Formel berechnet werden konnen, falls die Ausfalle gleich wahr-scheinlich sind, oder, im anderen Fall, simuliert werden konnen. Wie berechnet sich die Wahr-scheinlichkeit zusammengesetzter Ereignisse?

Die Analyse fuhren wir an Hand eines einfachen Beispiels durch.

Beispiel 4.4.4. Der Versuch bestehe aus einmaligem Wurfeln mit zwei unsymmetrischenWurfeln. Wir erhalten somit zwei unabhangige Teilversuche. Die Wahrscheinlichkeit der ein-zelnen Ausfalle seien:

Ausfalle beim 1. Wurfel 1 2 3 4 5 6

Wahrscheinlickeiten p1 p2 p3 p4 p5 p6

Ausfalle beim 2. Wurfel 1 2 3 4 5 6

Wahrscheinlickeiten q1 q2 q3 q4 q5 q6

mit6∑

i=1

pi =

6∑

i=1

qi = 1, wobei 0 ≤ pi, qi ≤ 1 fur alle i ∈ 1, 2, 3, 4, 5, 6.

Die Wahrscheinlichkeit des zusammengesetzten Versuchs seien nun

Ausfalle bei zwei Wurfeln (1, 1) (1, 2) (1, 3) · · · (6, 5) (6, 6)

Wahrscheinlickeiten p11 p12 p13 · · · p65 p66

auch hier gilt

6∑

i=1

6∑

j=1

pij = 1, wobei 0 ≤ pij ≤ 1 fur alle i, j ∈ 1, 2, 3, 4, 5, 6.

Wie lassen sich nun die pij aus den pi und qi berechnen?

Dazu benutzen wir die experimentelle Wahrscheinlichkeit als Grenzwert der relativen Haufig-keit. Betrachten wir einmal das spezielle Ereignis (3, 4):

• Wie viele Dreier erhalten wir ungefahr beiN Wurfen mit dem ersten Wurfel? – Antwort:ungefahr Np3.

• Wie viele Vierer erhalten wir ungefahr bei N Wurfen in diesen Np3 Wurfen mit demzweiten Wurfel? – Antwort: ungefahr Np3 · q4.

Page 46: Wahrscheinlichkeitstheorie undStatistik (wst)web.fhnw.ch/.../Stochastik-Skript-Marcel-Steiner.pdf · Anwendungsgebiete der Statistik sind die Industrie und die Technik, die Physik,

38 Kapitel 4. Wahrscheinlichkeit

Die relative Haufigkeit fur das Ereignis (3, 4) ist demzufolge ungefahr p3 · q4. Somit ist esnahe liegend p34 = p3 · q4 zu setzen. Allgemein gilt dann offenbar

pij = pi · qj fur alle i, j ∈ 1, 2, 3, 4, 5, 6.

Dieses Resultat lasst sich ohne weiteres auf kompliziertere Ereignisse verallgemeinern. Dabeierhalten wir den so genannten Multiplikationssatz.

Satz 4.4.2 (Multiplikationssatz). Besteht ein zusammengesetzter Versuch aus mehrerenTeilversuchen und beziehen sich die Ereignisse A1, . . . , Ai, . . . , An nur auf den jeweils i-tenTeilversuch, dann gilt

p(A1 ∩ · · · ∩An) = p(A1) · · · p(An).

Beispiel 4.4.5. Wie gross ist die Wahrscheinlichkeit, mit einem idealen Wurfel funf malhintereinander eine Sechs zu wurfeln?Es sei

Ai = i-ter Wurf ist eine Sechs wobei i ∈ 1, 2, 3, 4, 5.Dann ist das Ereignis funf mal eine Sechs zu wurfeln gleich A1 ∩ · · · ∩ A5 = (6, 6, 6, 6, 6)und hat die Wahrscheinlichkeit

p(A1 ∩ · · · ∩A5) = p(A1) · · · p(A5) =1

6· 16· 16· 16· 16=

1

65= 0.00013.

Einige Anwendungen des Multiplikationssatzes fur typische Versuche:

Beispiel 4.4.6. Wir betrachten einen Versuch mit zwei moglichen Ausfallen:

• Erfolg mit Wahrscheinlichkeit p(Erfolg) = p ∈ [0, 1].

• Misserfolg mit Wahrscheinlichkeit p(Misserfolg) = q = 1− p ∈ [0, 1].

Nach dem Multiplikationssatz

a. treten n Erfolge bei n Versuchen mit der Wahrscheinlichkeit pn auf;

b. treten n Misserfolge bei n Versuchen mit der Wahrscheinlichkeit qn = (1− p)n auf;

c. tritt mindestens ein Erfolg mit der Wahrscheinlichkeit 1− (1− p)n auf; und

d. tritt der erste Erfolg bei der n-ten Wiederholung mit Wahrscheinlichkeit (1 − p)n−1p

auf.

Aufgaben

Aufgabe 4.4.1. Ein symmetrischer Wurfel wird sechs mal geworfen. Wie gross ist die Wahr-scheinlichkeit, dass

a. jedes Mal eine gerade Zahl,

b. kein Sechs,

c. weder eine Funf noch eine Sechs

Page 47: Wahrscheinlichkeitstheorie undStatistik (wst)web.fhnw.ch/.../Stochastik-Skript-Marcel-Steiner.pdf · Anwendungsgebiete der Statistik sind die Industrie und die Technik, die Physik,

4.4. Wahrscheinlichkeit von zusammengesetzten Ereignissen 39

erscheint?

Aufgabe 4.4.2. Wie gross ist die Wahrscheinlichkeit, mit einem symmetrischen Wurfel beizwei Wurfen mindestens einmal eine Sechs zu erhalten?

Aufgabe 4.4.3. Aus einem Kartenspiel mit 36 Karten wird eine Karte gezogen und, wenndiese wieder eingemischt ist, eine zweite. Wie gross ist die Wahrscheinlichkeit, dass die ersteKarte ein As und die zweite eine Konigskarte ist?

Aufgabe 4.4.4. Ein Geldstuck wird zweimal geworfen. Wie gross ist die Wahrscheinlichkeit

a. genau einmal Kopf zu werfen,

b. zweimal Kopf zu werfen,

c. mindestens einmal Kopf zu werfen?

Aufgabe 4.4.5. Die Wahrscheinlichkeit, dass in einem gewissen Gebiet eine Olbohrungfundig wird, sei p = 0.1. Mit welcher Wahrscheinlichkeit haben 10 Bohrungen mindestenseinen Erfolg?

Aufgabe 4.4.6. Auf einem Abschnitt AB einer Moto-Cross-Bahn befinden sich 12 Hindernis-se, wovon jedes mit der Wahrscheinlichkeit p = 0.1 ein Anhalten des Rennfahrers verursacht.Die Wahrscheinlichkeit dafur, dass ein Rennfahrer von B bis C nicht anhalten muss, seiq = 0.7. Bestimmen Sie die Wahrscheinlichkeit, dass zwischen A und C kein Anhalten notigist.

Aufgabe 4.4.7. Wie gross ist die Wahrscheinlichkeit, dass sich vier Betriebsunfalle bei einerfunftage-Woche rein zufallig am Montag ereignen?

Aufgabe 4.4.8. Ein Schuss trifft das Ziel mit der Wahrscheinlichkeit p = 0.5. Wie vieleSchusse sind notig, um mit 99% Wahrscheinlichkeit mindestens einmal zu treffen?

Aufgabe 4.4.9. Die Herren A, B und C treffen eine fliegende Tontaube mit der Wahrschein-lichkeit pA = 0.5, pB = 2

3 und pC = 0.75. Eine Tontaube fliegt vorbei, und sie schiessen allegleichzeitig. Wie gross ist die Wahrscheinlichkeit, dass die Taube getroffen wird?

Aufgabe 4.4.10. Ein Gerat besteht aus drei Teilen A, B und C, die alle dieselbe Wahr-scheinlichkeit q haben, wahrend eines Tages zu versagen. Das Gerat fallt genau dann aus,wenn

a. mindestens ein Teil versagt,

b. alle drei Teile ausfallen.

Wie gross ist die Wahrscheinlichkeit, dass das Gerat den ganzen Tag arbeitet?

Aufgabe 4.4.11. Ein Flugzeug hat an jedem Flugel zwei Motoren. Die Wahrscheinlichkeit,dass ein Motor beim Flug uber den Atlantik versagt, sei q. Wie gross ist die Wahrscheinlichkeit,dass es uber dem Ozean absturzt, wenn

a. fur einen Flug mindestens zwei Motoren funktionieren mussen,

b. an jedem Flugel mindestens ein Motor intakt sein muss?

Page 48: Wahrscheinlichkeitstheorie undStatistik (wst)web.fhnw.ch/.../Stochastik-Skript-Marcel-Steiner.pdf · Anwendungsgebiete der Statistik sind die Industrie und die Technik, die Physik,

40 Kapitel 4. Wahrscheinlichkeit

Aufgabe 4.4.12. Damit ein zu bauender Apparat richtig funktioniert, mussen vier verschie-dene einwandfreie Teilstucke A, B, C und D richtig zusammengebaut werden. Fur jedes dieservier Teilstucke bestehe die Wahrscheinlichkeit von 2% nicht einwandfrei zu sein. In 1% allerFalle werden die vier Teilstucke nicht richtig zusammengebaut. Wie gross ist die Wahrschein-lichkeit, dass ein so gebauter Apparat bei der ersten Kontrolle richtig arbeitet?

Aufgabe 4.4.13. Eine aus 100 Produkten bestehende Serie testen wir durch eine Stichprobe.Die Serie ist unbrauchbar, wenn unter 5 ausgewahlten Produkten mindestens eines Ausschussist. Wie gross ist die Wahrscheinlichkeit fur die Unbrauchbarkeit der gegebenen Serie, wenndiese 5% ausschussige Produkte enthalt?

Aufgabe 4.4.14. Bei jedem Versuch trete ein bestimmtes Ereignis mit der Wahrscheinlichkeitp = 0.2 ein. Die Versuche werden solange nacheinander durchgefuhrt, bis dieses Ereigniseintrifft. Wie gross ist die Wahrscheinlichkeit, dass vier oder mehr Versuche gemacht werdenmussen?

Aufgabe 4.4.15. Mittels zweier Technologien konnen bestimmte Produkte hergestellt wer-den. Bei der ersten durchlauft ein Werkstuck drei Arbeitsgange, bei denen mit den Wahr-scheinlichkeiten 0.1, 0.2 und 0.3 Ausschuss entsteht. Die zweite Technologie umfasst zweiArbeitsgange, die beide mit der Wahrscheinlichkeit 0.3 Ausschuss liefern. Nach der Produk-tion werden die Produkte bei beiden Verfahren entsprechend ihrer Qualitat in zwei Klasseneingeteilt. In der ersten Technologie besteht die Wahrscheinlichkeit 0.9, dass ein Produkt in dieerste Qualitatsklasse kommt, in der zweiten 0.8. Bestimmen Sie, welche dieser Technologiendie grossere Wahrscheinlichkeit fur die Herstellung von Produkten der ersten Qualitatsklassebesitzt.

Aufgabe 4.4.16. Die Wahrscheinlichkeit dafur, dass ein beliebiges Werkstuck nach mecha-nischer (resp. thermischer) Bearbeitung Ausschuss ergibt, ist pmech (resp. ptherm). Die Wahr-scheinlichkeit dafur, dass sich die Mangel dieser fehlerhaften Stucke nicht beseitigen lassen,betrage qmech (resp. qtherm).

a. Welche Anzahl von Werkstucken mussen nach der mechanischen Bearbeitung mindes-tens zur Verfugung sein, damit mit der Wahrscheinlichkeit 0.9 nach der thermischenBearbeitung mindestens eines noch einwandfrei ist? Zahlen Sie unter die einwandfreienauch die fehlerhaften Stucke, die sich weiter verarbeiten lassen.

b. Wie gross ist die Wahrscheinlichkeit dafur, dass mindestens eines von drei Produktennach Durchlaufen der mechanischen und thermischen Bearbeitung Ausschuss ist?

Losungen

Losung 4.4.1.

a. 0.0156

b. 0.3349

c. 0.0878

Losung 4.4.2. 0.3056

Page 49: Wahrscheinlichkeitstheorie undStatistik (wst)web.fhnw.ch/.../Stochastik-Skript-Marcel-Steiner.pdf · Anwendungsgebiete der Statistik sind die Industrie und die Technik, die Physik,

4.4. Wahrscheinlichkeit von zusammengesetzten Ereignissen 41

Losung 4.4.3. 0.0123

Losung 4.4.4.

a. 0.5

b. 0.25

c. 0.75

Losung 4.4.5. 0.6513

Losung 4.4.6. 0.1977

Losung 4.4.7. 0.0016

Losung 4.4.8. n ≥ 7

Losung 4.4.9. 0.9583

Losung 4.4.10.

a. P (Gerat arbeitet) = (1− q)3

b. P (Gerat arbeitet) = 1− q3

Losung 4.4.11.

a. P (Absturz) = q4 + 4(1− q)q3

b. P (Absturz) = q4 + 4(1− q)q3 + 2(1− q)2q2

Losung 4.4.12. 0.9131

Losung 4.4.13. 0.2262

Losung 4.4.14. 0.512

Losung 4.4.15. Die erste Technologie besitzt die grossere Wahrscheinlichkeit fur die Her-stellung von Produkten der ersten Qualitatsklasse.

Losung 4.4.16.

a. n ≥ log(0.1)

log(ptherm · qtherm)

b. P (mindestens eines Ausschuss) = 1− (1− pmech · qmech)3(1− ptherm · qtherm)3

Page 50: Wahrscheinlichkeitstheorie undStatistik (wst)web.fhnw.ch/.../Stochastik-Skript-Marcel-Steiner.pdf · Anwendungsgebiete der Statistik sind die Industrie und die Technik, die Physik,

42 Kapitel 4. Wahrscheinlichkeit

Page 51: Wahrscheinlichkeitstheorie undStatistik (wst)web.fhnw.ch/.../Stochastik-Skript-Marcel-Steiner.pdf · Anwendungsgebiete der Statistik sind die Industrie und die Technik, die Physik,

Kapitel 5

Zufallsgrossen undWahrscheinlichkeitsverteilungen

5.1 Diskrete und stetige Zufallsgrossen

Betrachten wir einen Versuch mit dem Stichprobenraum S = s1, s2, . . . , sn. Jedem Ausfallsi von S sei eine reelle Zahl zugeordnet

X : S −→ R

si 7−→ X(si) = xi

Die Zuordnung X wird in der Statistik Zufallsgrosse oder Zufallsvariable genannt.1

Die Zufallsgrosse X, die die Werte x1, . . . , xn annehmen kann, wird durch die Angabe ih-rer Wahrscheinlichkeiten charakterisiert. Jedem Ausfall si aus dem Stichprobenraum S,respektive X(si) = xi, entspricht eine Wahrscheinlichkeit

pi = P (X = xi) ∈ [0, 1]

als Funktion2 von xi aufgefasst. Dabei gilt

n∑

i=1

pi =

n∑

i=1

P (X = xi) = 1.

Die Funktion

xi 7−→ P (X = xi)

heisst Wahrscheinlichkeitsverteilung der Zufallsgrosse X.

Beispiel 5.1.1. Wir betrachten den symmetrischen Wurfel mit dem Stichprobenraum S =1, 2, 3, 4, 5, 6. In diesem Fall setzen wir

X(si) = i fur alle i ∈ 1, 2, 3, 4, 5, 6.1Zufallsgrossen werden im Allgemeinen mit grossen lateinischen Buchstaben X,Y, Z, . . . und die Werte, die

sie annehmen, mit kleinen lateinischen Buchstaben x, y, z, . . . bezeichnet.2Die Symbolik pi = P (X = xi) wird wie folgt gelesen: Die Wahrscheinlichkeit, dass die Zufallsgrosse X den

Wert xi annimmt, ist pi.

43

Page 52: Wahrscheinlichkeitstheorie undStatistik (wst)web.fhnw.ch/.../Stochastik-Skript-Marcel-Steiner.pdf · Anwendungsgebiete der Statistik sind die Industrie und die Technik, die Physik,

44 Kapitel 5. Zufallsgrossen und Wahrscheinlichkeitsverteilungen

Die Wahrscheinlichkeitsverteilung ist bekanntlich

pi = P (X = xi) =1

6fur alle i ∈ 1, 2, 3, 4, 5, 6.

Als Tabelle dargestellt ergibt sich:

xi 1 2 3 4 5 6

P (X = xi)16

16

16

16

16

16

Beispiel 5.1.2. Wir betrachten einen unsymmetrischen Wurfel mit dem StichprobenraumS = 1, 2, 3, 4, 5, 6. In diesem Fall setzen wir wiederum

X(si) = i fur alle i ∈ 1, 2, 3, 4, 5, 6.

Damit folgt

xi 1 2 3 4 5 6

P (X = xi) p1 p2 p3 p4 p5 p6

mit6∑

i=1

pi = 1

(vgl. Abbildung 5.1.i).

Oft geben wir nicht die Wahrscheinlichkeiten einzeln an, sondern die Wahrscheinlichkeits-summe von links. Damit erhalten wir die so genannte Verteilungsfunktion oder Summen-funktion (vgl. Abbildung 5.1.ii).

xi 1 2 3 4 5 6

P (X ≤ xi) =

k∑

i=1

P (X = xi) p1 p1 + p2 · · ·6∑

i=1

pi = 1

Allgemein ist die Verteilungsfunktion F einer diskreten Zufallsgrosse durch

F (x) = P (X ≤ x) =∑

alle i mit xi≤x

pi

gegeben. Die Summation erfolgt uber alle pi, fur die xi hochstens gleich x ist. Die Verteilungs-funktion F ist eine monoton wachsende Treppenfunktion mit Sprungen der Hohe pi an denStellen xi.

Die soweit beschriebenen Zufallsgrossen heissen diskrete Zufallsgrossen, da sie nur ganz be-stimmte, getrennt liegende Zahlenwerte annehmen. Eine andere Art Zufallsgrossen sind dieso genannt stetigen Zufallsgrossen. Diese nehmen als Werte samtliche Zahlen eines ganzenIntervalls an, zum Beispiel alle reellen Zahlen R oder alle Zahlen im Intervall [−1, 1]. Die ste-tigen Zufallsgrossen haben vor allem bei Messprozessen eine grosse Bedeutung. Die diskretentreten vorwiegend bei Zahlprozessen auf.

An Hand eines einpragsamen Beispiels betrachten wir den Unterschied zwischen einer diskre-ten und einer stetigen Zufallsgrosse:

Page 53: Wahrscheinlichkeitstheorie undStatistik (wst)web.fhnw.ch/.../Stochastik-Skript-Marcel-Steiner.pdf · Anwendungsgebiete der Statistik sind die Industrie und die Technik, die Physik,

5.1. Diskrete und stetige Zufallsgrossen 45

1

p1

2

p2

3

p3

4

p4

5

p5

6

p6

1

pi = P (X = xi)

xi

Abbildung 5.1.i: Wahrscheinlichkeitsver-teilung des unsymmetrischen Wurfels

1

p1

2

p1 + p2

3

p1 + p2 + p3

4

p1 + p2 + p3 + p4

5

p1 + p2 + p3 + p4 + p5

6

p1 + p2 + p3 + p4 + p5 + p61

F (x) = P (X ≤ xi)

xi

Abbildung 5.1.ii: Verteilungsfunktion desunsymmetrischen Wurfels

Beispiel 5.1.3. Der Stichprobenraum S sei die Menge aller Studierenden einer Klasse. DieZufallsgrosse X ordne nun jedem Studierenden s die Lange in Zentimeter seines rechten Fusseszu. In diesem Fall konnen alle Fusslangen im Intervall [5 cm, 40 cm] angenommen werden. DieFusslangen einer Klasse sind also stetig verteilt.

Fragen wir nun aber nach der jeweiligen Schuhnummer der Studierenden, so erhalten wir nurWerte in der Menge 34, 341

2 , 36, . . . , 46, 4612. Die Schuhnummern einer Klasse sind somit

diskret verteilt. Je nach dem welches Merkmal betrachtet wird, kann also eine Stichprobe zueiner diskreten oder stetigen Verteilung fuhren.

Aufgaben

Aufgabe 5.1.1. Die Trefferwahrscheinlichkeit fur einen Basketball in den gegnerischen Korbsei bei jedem Wurf 0.3. Bestimmen Sie die Wahrscheinlichkeitsverteilung und die Verteilungs-funktion der zufalligen Trefferzahl X bei zwei Wurfen.

Aufgabe 5.1.2. Ein Versuch bestehe aus drei unabhangigen Munzwurfen. Bei jedem Wurfliegt Kopf mit der Wahrscheinlichkeit 0.5 oben. Bestimmen Sie Wahrscheinlichkeitsverteilungund die Verteilungsfunktion fur die Anzahl X des Eintreffens von Kopf.

Aufgabe 5.1.3. Eine Familie habe funf Kinder. Wir betrachten die Anzahl der Madchen alsWerte einer Zufallsgrosse X, dabei sind Zwillinge ausgeschlossen. Bestimmen Sie die Wahr-scheinlichkeitsverteilung von X.

Aufgabe 5.1.4. Eine Zielscheibe bestehe aus einem Kreis Nr. 1 und zwei Ringen mit denNummern 2 und 3. Ein Treffer in den Kreis Nr. 1 liefert 10 Punkte, ein Treffer in den Ring Nr.

Page 54: Wahrscheinlichkeitstheorie undStatistik (wst)web.fhnw.ch/.../Stochastik-Skript-Marcel-Steiner.pdf · Anwendungsgebiete der Statistik sind die Industrie und die Technik, die Physik,

46 Kapitel 5. Zufallsgrossen und Wahrscheinlichkeitsverteilungen

2 liefert 5 Punkte und ein Treffer in den Ring Nr. 3 liefert 1 Punkt. Die Trefferwahrschein-lichkeiten betragen entsprechend 50%, 30% und 20%. Bestimmen Sie die Wahrscheinlich-keitsverteilung fur die Summe X der erreichten Punkte bei drei Treffern auf die Zielscheibe.Fehlschusse werden nicht berucksichtigt.

Aufgabe 5.1.5. Die Zuverlassigkeit von funf Geraten wird hintereinander uberpruft. JedesGerat wird nur dann gepruft, wenn das vorhergehende zuverlassig war. Bestimmen Sie dieWahrscheinlichkeitsverteilung der Anzahl X der gepruften Gerate, wenn jedes Gerat mit derWahrscheinlichkeit 0.9 die Prufung besteht.

Aufgabe 5.1.6. Es seien n Rohlinge fur ein Werkstuck vorhanden. Die Wahrscheinlichkeitfur die Herstellung eines brauchbaren Stucks aus einem Rohling betrage p.

a. Bestimmen Sie die Wahrscheinlichkeitsverteilung der Anzahl X Rohlinge, die nach derHerstellung des ersten brauchbaren Werkstucks ubrig bleiben.

b. Bestimmen Sie die Wahrscheinlichkeitsverteilung fur die Anzahl X der verwendetenRohlinge.

Losungen

Losung 5.1.1. Wahrscheinlichkeitsverteilung und die Verteilungsfunktion der zufalligen Tref-ferzahl X = k.

k 0 1 2

P (X = k) 0.49 0.42 0.09k∑

i=0

P (X = i) 0.49 0.91 1.00

Losung 5.1.2. Wahrscheinlichkeitsverteilung und die Verteilungsfunktion fur die AnzahlX = k.

k 0 1 2 3

P (X = k) 0.125 0.375 0.375 0.125k∑

i=0

P (X = i) 0.125 0.500 0.875 1.000

Losung 5.1.3. Wahrscheinlichkeitsverteilung der Anzahl Madchen X = k.

k 0 1 2 3 4 5

P (X = k) 0.03125 0.15625 0.31250 0.31250 0.15625 0.03125

Losung 5.1.4. Wahrscheinlichkeitsverteilung fur die Summe X = n der erreichten Punktebei drei Treffern.

n Punkte 3 7 11 12 15 16 20 21 25 30

P (X = n) 0.008 0.036 0.054 0.060 0.027 0.180 0.135 0.150 0.225 0.125

Beachten Sie, dass zur Kontrolle∑

n∈mogliche Punkte bei 3 WurfenP (X = n) = 1

gelten muss.

Page 55: Wahrscheinlichkeitstheorie undStatistik (wst)web.fhnw.ch/.../Stochastik-Skript-Marcel-Steiner.pdf · Anwendungsgebiete der Statistik sind die Industrie und die Technik, die Physik,

5.1. Diskrete und stetige Zufallsgrossen 47

Losung 5.1.5. Wahrscheinlichkeitsverteilung der Anzahl X = x der gepruften Gerate.

x 1 2 3 4 5

p 0.1000 0.0900 0.0810 0.0729 0.6561

Losung 5.1.6.

a. Es bezeichne X = k die Anzahl Rohlinge, die ubrig bleiben.

k 0 1 2 . . . n− 1

P (X = k) (1− p)n−1 (1− p)n−2p (1− p)n−3p · · · p

Zur Kontrolle muss

n−1∑

k=0

P (X = k) = (1− p)n−1 + (1− p)n−2p+ (1− p)n−3p+ · · ·+ p

= (1− p)n−1 + p((1− p)n−2 + (1− p)n−3 + · · ·+ 1

)

= (1− p)n−1 + p1− (1− p)n−1

1− (1− p)

= 1

gelten.

b. Es bezeichne X = k die Anzahl der verwendeten Rohlinge.

k 1 2 3 4 . . . n

P (X = k) p p(1− p) p(1− p)2 p(1− p)3 · · · (1− p)n−1

Zur Kontrolle muss

n∑

k=1

P (X = k) = p+ p(1− p) + p(1− p)2 + · · ·+ (1− p)n−1

= p(1 + (1− p)1 + (1− p)2 + · · ·+ (1− p)n−2

)+ (1− p)n−1

= p1− (1− p)n−1

1− (1− p)+ (1− p)n−1

= 1

gelten.

Page 56: Wahrscheinlichkeitstheorie undStatistik (wst)web.fhnw.ch/.../Stochastik-Skript-Marcel-Steiner.pdf · Anwendungsgebiete der Statistik sind die Industrie und die Technik, die Physik,

48 Kapitel 5. Zufallsgrossen und Wahrscheinlichkeitsverteilungen

Page 57: Wahrscheinlichkeitstheorie undStatistik (wst)web.fhnw.ch/.../Stochastik-Skript-Marcel-Steiner.pdf · Anwendungsgebiete der Statistik sind die Industrie und die Technik, die Physik,

Kapitel 6

Diskrete Zufallsgrossen undVerteilungen

6.1 Erwartungswert und Varianz

Analog wie bei empirischen Stichproben konnen wir bei Wahrscheinlichkeitsverteilungen cha-rakterisierende Parameter definieren.

Es seien x1, . . . , xn die Werte einer Zufallsgrosse X, die mit den entsprechenden Wahrschein-lichkeiten p1, . . . , pn auftreten. Dann konnen wir eine Art Lageparameter fur die Werte derdiskreten Zufallsgrosse definieren, den so genannten Erwartungswert der diskreten Zufalls-grosse

µ = E(X) =

n∑

i=1

xipi. (6.1.a)

Analog lasst sich ein Formparameter definieren, die so genannte Varianz oder Streuung derdiskreten Zufallsgrosse

σ2 = Var(X) = E((X − µ)2

)=

n∑

i=1

(xi − µ)2pi =

n∑

i=1

x2i pi − µ2. (6.1.b)

Die (positive) Quadratwurzel σ heisst Standardabweichung.

Beispiel 6.1.1. Betrachten wir wieder einmal den symmetrischen Wurfel. Es ist bekanntlichxi = i und pi =

16 fur alle i ∈ 1, 2, 3, 4, 5, 6. Dann erhalten wir fur den Erwartungswert

E(X) =

6∑

i=1

i · 16

= 1 · 16+ 2 · 1

6+ 3 · 1

6+ 4 · 1

6+ 5 · 1

6+ 6 · 1

6=

21

6= 3.5

49

Page 58: Wahrscheinlichkeitstheorie undStatistik (wst)web.fhnw.ch/.../Stochastik-Skript-Marcel-Steiner.pdf · Anwendungsgebiete der Statistik sind die Industrie und die Technik, die Physik,

50 Kapitel 6. Diskrete Zufallsgrossen und Verteilungen

und fur die Varianz

Var(X) =n∑

i=1

(i− µ)21

6

= (1− 3.5)21

6+ (2− 3.5)2

1

6+ (3− 3.5)2

1

6+ (4− 3.5)2

1

6+ (5− 3.5)2

1

6+ (6− 3.5)2

1

6= 2.92.

Die Standardabweichung betragt demzufolge σ = 1.71.

Aufgaben

Aufgabe 6.1.1. Eine Klasse habe die folgende Altersgliederung:

Alter xi 15 16 17 18

Haufigkeit hi 2 7 13 3

Bestimmen Sie die Wahrscheinlichkeitsverteilung fur das Alter X eines zufallig heraus gegrif-fenen Schulers und berechnen Sie den Erwartungswert und die Varianz.

Aufgabe 6.1.2. Eine Munze werde vier mal geworfen, und es bezeichne X die Anzahl Kopf.Bestimmen Sie die Wahrscheinlichkeitsverteilung vonX, den Erwartungswert und die Varianz.

Aufgabe 6.1.3. Bestimmen Sie den Erwartungswert und die Varianz der Aufgabe 5.1.4.

Aufgabe 6.1.4. Eine Zufallsgrosse X habe die folgende Verteilung (Gleichverteilung):

xi 1 2 · · · n

pi1n

1n

· · · 1n

Bestimmen Sie den Erwartungswert und die Varianz.

Aufgabe 6.1.5. Es sei X die Augenzahl eines symmetrischen Wurfels und Y = X2. Bestim-men Sie E(Y ). Welche Bedeutung hat die Zufallsgrosse Y .

Aufgabe 6.1.6. Es sei X eine Zufallsgrosse auf dem Stichprobenraum S. Ihre Wahrschein-lichkeitsverteilung sei P (X = xi) = pi. Ferner seien a und b zwei reelle Parameter. BeweisenSie die Beziehung

E(aX + b) = aE(X) + b.

Losungen

Losung 6.1.1. Wahrscheinlichkeitsverteilung fur das Alter X = xi eines zufallig herausgegriffenen Schulers.

Alter xi 15 16 17 18

P (X = xi) 0.08 0.28 0.52 .12

E(X) = 16.68 und Var(X) = 0.618

Losung 6.1.2. Wahrscheinlichkeitsverteilung der Anzahl Kopf X = k.

Page 59: Wahrscheinlichkeitstheorie undStatistik (wst)web.fhnw.ch/.../Stochastik-Skript-Marcel-Steiner.pdf · Anwendungsgebiete der Statistik sind die Industrie und die Technik, die Physik,

6.2. Die Binomialverteilung 51

k 0 1 2 3 4

P (X = k) 0.0625 0.2500 0.3750 0.2500 0.0625

E(X) = 2.0 und Var(X) = 1.0

Losung 6.1.3. E(X) = 20.1 und Var(X) = 38.4

Losung 6.1.4. Benutzen Sie Beispiel A.1.1(b) und (c), dann folgt E(X) = n+12 und Var(X) =

n2−112 .

Losung 6.1.5. E(Y ) = 15.1667

Losung 6.1.6. Setzen Sie aX + b direkt in die Definition des Erwartungswertes ein.

6.2 Die Binomialverteilung

6.2.1 Definition und Eigenschaften der Binomialverteilung

Die Binomialverteilung beschaftigt sich mit Ereignissen, bei denen zwei alternative Ausgangeauftreten konnen, wie zum Beispiel Munzwurf (Kopf oder Zahl, gleich wahrscheinlich) oderbeim Werfen eines Wurfels (6 oder keine 6 geworfen, ungleich wahrscheinlich). Wir betrachtenalso einen Versuch mit zwei moglichen Ausfallen:

• Erfolg mit Wahrscheinlichkeit p ∈ [0, 1].

• Misserfolg mit Wahrscheinlichkeit q = 1− p ∈ [0, 1].

Dieser Versuch werde n mal durchgefuhrt. Es sei X die Zufallsgrosse, deren Werte x ∈ N0 dieAnzahl Erfolge bei n Versuchen bedeute. Wir bestimmen nun die Wahrscheinlichkeitsvertei-lung von X.Bei n Versuchen gibt es genau

(nx

)Anordnungen mit x Erfolgen und n−x Misserfolgen. Damit

erhalten wir die Binomialverteilung

P (X = x) =

(n

x

)pxqn−x =

(n

x

)px(1− p)n−x,

wobei x ∈ N0. Die Wahrscheinlichkeitsverteilung hat somit folgende Form:

x 0 1 · · · x · · · n− 1 n

P (X = x) qn npqn−1 · · ·(n

x

)pxqn−x · · · npn−1q pn

Um die zum Teil recht muhsamen Berechnungen der Wahrscheinlichkeiten zu vereinfachen, be-dienen wir uns Taschenrechner oder Computer, in Excel benutzen wir den Befehl BINOMVERT.

Beispiel 6.2.1. Zwei Spieler A und B spielen Tischtennis. Der bessere Spieler A gewinntmit der Wahrscheinlichkeit von 60%. Unentschieden sei ausgeschlossen. Sieger des Turniers(3 Spiele) ist der Spieler, der die Mehrzahl der Spiele gewonnen hat.

Es bezeichne X die Zufallsgrosse, die als Werte die Anzahl der von A gewonnenen Spiele habe.Dann ist n = 3, p = 0.6 und q = 0.4.

Page 60: Wahrscheinlichkeitstheorie undStatistik (wst)web.fhnw.ch/.../Stochastik-Skript-Marcel-Steiner.pdf · Anwendungsgebiete der Statistik sind die Industrie und die Technik, die Physik,

52 Kapitel 6. Diskrete Zufallsgrossen und Verteilungen

p

p

pnp

pn−1qq

pn−2q2p

pn−3q3q

q

p

q

pxqn−xp

px−1qn−x+1q

q

p

p

q

p

q

q

p3qn−3p

p2qn−2q

pqn−1p

qnq

1 2 · · · n − 1 n

Abbildung 6.2.i: Wahrscheinlichkeitsbaum der Binomialverteilung mit Erfolgs- p und Misser-folgswahrscheinlichkeit q = 1− p.

Wie gross ist die Wahrscheinlichkeit, dass der schlechtere Spieler das Turnier gewinnt? Wirerhalten

P (X ≤ 1) = P (X = 0) + P (X = 1)

=

(3

0

)p0q3 +

(3

1

)p1q2

= 0.43 + 3 · 0.6 · 0.42 = 0.352.

Folgerung: In diesem Fall werden etwa 35% aller Turniere vom schlechteren Spieler gewonnen.

Bemerkung 6.2.1. Aus der Analysis (vgl. Analysis I, [14]) kennen wir den BinomischenSatz

(a+ b)n =

n∑

x=0

(n

x

)axbn−x. (6.2.a)

Wir berechnen die Wahrscheinlichkeit, dass X die Werte 0, 1, 2, . . . , n− 1 oder n (das sichere

Page 61: Wahrscheinlichkeitstheorie undStatistik (wst)web.fhnw.ch/.../Stochastik-Skript-Marcel-Steiner.pdf · Anwendungsgebiete der Statistik sind die Industrie und die Technik, die Physik,

6.2. Die Binomialverteilung 53

Ereignis) annimmt

P (X ≤ n) =

n∑

x=0

P (X = x)

=n∑

x=0

(n

x

)px(1− p)n−x

= (p+ (1− p))n = 1.

Die Polynome fx,n(p) =(nx

)px(1−p)n−x in der Variablen p werden Bernstein Polynome ge-

nannt und finden bei den so genannten Bezier-Kurven ihre Anwendung (vgl. Aufgabe 6.2.4).

Aufgaben

Aufgabe 6.2.1. Machen Sie sich die Verhaltnisse bei der Binomialverteilung klar, indem SieHistogramme fur die folgenden speziellen Falle zeichnen:

a. n = 3 und p = q

b. n = 3 und p = 0.25

c. n = 8 und p = 0.9

Aufgabe 6.2.2. Jedes Mitglied eines Komitees mit 9 Mitgliedern kommt mit der Wahrschein-lichkeit von 0.5 zur Versammlung. Wie gross ist die Wahrscheinlichkeit, dass eine zweidrittels-Mehrheit, d.h. sechs oder mehr, anwesend ist

Aufgabe 6.2.3. Jemand wettet, dass er bei 12 Wurfen einer Munze genau 6 mal Zahl erziele.Wie gross ist die Gewinnwahrscheinlichkeit?

Aufgabe 6.2.4. Betrachten Sie in der Formel

fx,n(p) =

(n

x

)px(1− p)n−x

die Wahrscheinlichkeit p als Variable und x und n als Konstanten.

a. Stellen Sie die Bernstein-Polynome f0,4, f1,4, . . . , f4,4 im Intervall [0, 1] grafisch dar.

b. Stellen Sie die Bernstein-Polynome f0,10, f1,10, . . . , f10,10 im Intervall [0, 1] grafisch dar.

c. Fur welchen Wert von p besitzt das Bernstein Polynom fx,n den grossten Wert? Ver-gleichen Sie mit Ihren Grafiken.

Losungen

Losung 6.2.1. Siehe Excelfile: Losung 6.2.1 Binomialverteilung.xls

Losung 6.2.2. P (X ≥ 6) = 0.2539

Losung 6.2.3. P (X = 6) = 0.2256

Page 62: Wahrscheinlichkeitstheorie undStatistik (wst)web.fhnw.ch/.../Stochastik-Skript-Marcel-Steiner.pdf · Anwendungsgebiete der Statistik sind die Industrie und die Technik, die Physik,

54 Kapitel 6. Diskrete Zufallsgrossen und Verteilungen

Losung 6.2.4.

a. Abbildung 6.2.ii

b. Abbildung 6.2.iii

c. p = xn

0.0 0.2 0.4 0.6 0.8 1.0

0.0

0.2

0.4

0.6

0.8

1.0

Abbildung 6.2.ii: Bernstein-Polynomef0,4, f1,4, . . . , f4,4

0.0 0.2 0.4 0.6 0.8 1.0

0.0

0.2

0.4

0.6

0.8

1.0

Abbildung 6.2.iii: Bernstein-Polynomef0,10, f1,10, . . . , f10,10

6.2.2 Erwartungswert und Varianz der Binomialverteilung

Diese beiden Parameter, und das gilt fur alle Wahrscheinlichkeitsverteilungen, konnen wirmit Hilfe der so genannten erzeugenden Funktion relativ einfach herleiten, ohne die ganzeSummationen zur Berechnung des Erwartungswertes und der Varianz ausfuhren zu mussen.Es bleibt allerdings das Problem offen, wie wir im Allgemeinen die entsprechende erzeugendeFunktion finden, die zu einer bestimmten Wahrscheinlichkeitsverteilung gehort.Nach dem Binomischen Satz (vgl. Analysis I, [14]) gilt

f(t) = (q + pt)n =n∑

x=0

(n

x

)qn−xpxtx.

Die Funktion f ist die erzeugende Funktion der Binomialverteilung.Wir berechnen die erste Ableitung der erzeugenden Funktion

f ′(t) = n(q + pt)n−1p =

n∑

x=0

x

(n

x

)qn−xpxtx−1

und werten sie an der Stelle t = 1 aus

f ′(1) = n(q + p)n−1p =n∑

x=0

x

(n

x

)qn−xpx =

n∑

x=0

xP (X = x) =n∑

x=0

xpx

Page 63: Wahrscheinlichkeitstheorie undStatistik (wst)web.fhnw.ch/.../Stochastik-Skript-Marcel-Steiner.pdf · Anwendungsgebiete der Statistik sind die Industrie und die Technik, die Physik,

6.2. Die Binomialverteilung 55

Da p+q = 1 ist, folgt mit Hilfe der Definition des Erwartungswertes fur diskrete Verteilungen(Formel 6.1.a), dass der Erwartungswert einer binomialverteilten Zufallsgrosse

µ = E(X) = np

betragt.Analog bestimmen wir die Varianz. Wir berechnen die zweite Ableitung der erzeugendenFunktion

f ′′(t) = n(n− 1)(q + pt)n−2p2 =n∑

x=0

x(x− 1)

(n

x

)qn−xpxtx−2

und werten sie wieder an der Stelle t = 1 aus

f ′′(1) = n(n− 1)(q + p)n−2p2 =n∑

x=0

x(x− 1)

(n

x

)qn−xpx

=

n∑

x=0

x(x− 1)P (X = x)

=n∑

x=0

x2P (X = x)−n∑

x=0

xP (X = x)

=

n∑

x=0

x2px −n∑

x=0

xpx

=

n∑

x=0

x2px − µ.

Da p+ q = 1 und mit µ = np folgt fur die linke Seite der obigen Gleichung, dass

n(n− 1)p2 = n2p2 − np2 = µ2 − µp.

Mit Hilfe der Definition der Varianz fur diskrete Verteilungen (Formel 6.1.b), i.e. σ2 =∑ni=1 x

2i pi − µ2, folgern wir fur die rechte Seite

n∑

x=0

x2px − µ = σ2 + µ2 − µ.

Da die linke und rechte Seite gleich sein mussen, erhalten wir die Gleichung

µ2 − µp = σ2 + µ2 − µ,

die wir nach

σ2 = µ− µp = µ(1− p) = np(1− p)

umformen.Die Varianz oder Streuung einer binomialverteilten Zufallsgrosse X betragt demzufolge

σ2 = Var(X) = npq.

Die Standardabweichung der binomialverteilten Zufallsgrosse betragt σ =√npq.

Page 64: Wahrscheinlichkeitstheorie undStatistik (wst)web.fhnw.ch/.../Stochastik-Skript-Marcel-Steiner.pdf · Anwendungsgebiete der Statistik sind die Industrie und die Technik, die Physik,

56 Kapitel 6. Diskrete Zufallsgrossen und Verteilungen

6.2.3 Die Binomialverteilung beim Testen von Hypothesen

Das folgende Beispiel zeigt eine Anwendung der Binomialverteilung beim Testen einer Hy-pothese. Dabei ist das Vorgehen typisch fur die Ausfuhrung eines statistischen Tests. Beianderen Verteilungen wurde nur die Verteilung andern, nicht aber das grundsatzliche Vorge-hen (vgl. Vorlesung uber Statistische Datenanalyse, [15]). Wir konnen deshalb dieses Beispielals Prototyp fur einen statistischen Test uberhaupt sehen.

Beispiel 6.2.2. Wir befassen uns mit der Frage: Kann ein neugeborenes Huhn Formen un-terscheiden?Um dies zu entscheiden, werden dem Huhn “Korner” aus Papier vorgelegt. Die Halfte derKorner (aus Papier) besteht aus kleinen Dreiecken und die andere Halfte aus Kreisen. Nunlassen wir das Huhn 20 mal picken. Das Ergebnis unserer Untersuchung zeigt, dass das Huhn5 mal ein Dreieck und 15 mal ein Kreis gepickt hat. Wir hegen deshalb die Vermutung, dassdas Huhn Kreise bevorzugt. Diese Vermutung wollen wir nun testen.Die Aufgabenstellung ist eine Fragestellung der Binomialverteilung, wenn wir Kreis als Erfolgmit Wahrscheinlichkeit p und Dreieck als Misserfolg mit Wahrscheinlichkeit q auffassen.Es sei X die Zufallsgrosse, deren Werte x ∈ N0 die Anzahl der Kreise bei n = 20 “Kornern”sind. Die Wahrscheinlichkeitsverteilung der Zufallsgrosse X ist dann durch die folgende Ta-belle gegeben:

x 0 1 2 3 · · · 19 20

px q20(20

1

)pq19

(20

2

)p2q18

(20

3

)p3q17 · · ·

(20

19

)p19q p20

Zur Durchfuhrung des Tests formulieren wir zwei sich ausschliessende Hypothesen:

Nullhypothese H0 : p = q = 12 , d.h., das Huhn unterscheidet keine Formen.

Das Histogramm in Abbildung 6.2.iv ist richtig.

Alternativhypothese H1 : p > q, d.h., das Huhn zieht Kreise vor.

Das Histogramm in Abbildung 6.2.v ist moglich.

Das Anliegen eines statistischen Tests zur Prufung von H0 gegen H1 ist es, eine Entscheidungdaruber zu treffen, ob die aus einer konkreten Stichprobe entnommenen Angaben zur auf-gestellten Hypothese H0 im Widerspruch stehen oder nicht, d.h., ob H0 abzulehnen ist odernicht. Falls die Hypothese H1 gilt, so muss nicht unbedingt das wirkliche Histogramm, d.h.die Wahrscheinlichkeitsverteilung, so einseitig sein wie in Abbildung 6.2.v. Es konnte p auchnur wenig grosser sein als q.Nun berechnen wir unter der Voraussetzung der Nullhypothese H0, d.h. p = q = 1

2 , dieWahrscheinlichkeit

P (15 ≤ X ≤ 20) =

20∑

x=15

(20

x

)pxq20−x

=

20∑

x=15

(20

x

)1

220

= 0.021.

Page 65: Wahrscheinlichkeitstheorie undStatistik (wst)web.fhnw.ch/.../Stochastik-Skript-Marcel-Steiner.pdf · Anwendungsgebiete der Statistik sind die Industrie und die Technik, die Physik,

6.2. Die Binomialverteilung 57

1 2 3 4 5 6 7 8 9 10 11 12 13 14 15 16 17 18 19 20

0.05

0.10

0.15

0.20

P (X = x)

x

Abbildung 6.2.iv: Histogramm der Vertei-lung der Anzahl Kreise, sofern das Huhnkeine Formen unterscheiden kann, also beip = q = 0.5.

1 2 3 4 5 6 7 8 9 10 11 12 13 14 15 16 17 18 19 20

0.05

0.10

0.15

0.20

P (X = x)

x

Abbildung 6.2.v: Histogramm der Vertei-lung der Anzahl Kreise, sofern das HuhnKreise mit einer Wahrscheinlichkeit vonp = 0.75 den Dreiecken vorzieht.

Somit besteht unter der Voraussetzung der Nullhypothese H0 nur eine Wahrscheinlichkeitvon etwa 2.1%, dass das Huhn 15 oder mehr Kreise nimmt. Die Wahrscheinlichkeit fur diesesEreignis ist unter der Voraussetzung der Nullhypothese sehr klein.

Hat aber das Histogramm das Aussehen wie in Abbildung 6.2.v oder auch weniger asym-metrisch, so wird die Wahrscheinlichkeit fur ein solches Ereignis wesentlich grosser und diesin Abhangigkeit von p. Wir folgern deshalb: Die Nullhypothese ist zu verwerfen und H1 istanzunehmen. Die berechnete Wahrscheinlichkeit von P (15 ≤ X ≤ 20) = 2.1% stellt dann beidiesem Schluss die Irrtumswahrscheinlichkeit dar.

Im Allgemeinen mussen wir uns entscheiden, wann eine berechnete Abweichung zur Ableh-nung der Nullhypothese fuhren soll. Dazu wird eine Schranke α ∈ ]0, 1[, das so genannte Si-gnifikanzniveau, gewahlt. Ist die berechnete Wahrscheinlichkeit der Abweichung kleiner alsdas Signifikanzniveau, so wird die Nullhypothese abgelehnt, sonst angenommen. Die zulassigeGrosse des Signifikanzniveaus α hangt stark vom Fachgebiet ab und ist eine Vereinbarungs-sache. Haufig verwendete Niveaus sind α = 0.01, 0.05 und 0.1.

Das Prinzip eines statistischen Tests oder Signifikanztest lasst sich in folgenden Schrittenzusammenfassen:

1. Aufstellen der Nullhypothese H0 und der Alternativhypothese H1 und Vorgabedes Signifikanzniveaus α.

2. Berechnung der Wahrscheinlichkeit des Ereignisses unter der Voraussetzung derNullhypothese H0.

3. Statistischer Schluss: Ist die berechnete Wahrscheinlichkeit kleiner als das Signifi-kanzniveau α, so wird H0 abgelehnt, sonst wird H0 angenommen.

Wir sagen dann: Die Nullhypothese wird auf dem Signifikanzniveau α verworfen (resp. ange-nommen). Nur wenn wir die Nullhypothese verwerfen, geben wir die Irrtumswahrscheinlichkeitan.

Page 66: Wahrscheinlichkeitstheorie undStatistik (wst)web.fhnw.ch/.../Stochastik-Skript-Marcel-Steiner.pdf · Anwendungsgebiete der Statistik sind die Industrie und die Technik, die Physik,

58 Kapitel 6. Diskrete Zufallsgrossen und Verteilungen

Aufgaben

Formulieren Sie jeweils die Null- und Alternativhypothese und den problemorientierten sta-tistischen Schluss in Worten.

Aufgabe 6.2.5. Bei einem Wurfel ist die Wahrscheinlichkeit fur eine Sechs p = 16 . Fritz hat

den Verdacht, dass bei Hans’ Wurfel p > 16 ist. Er pruft ihn und erhalt

a. zwei Sechsen in drei Wurfen,

b. drei Sechsen in funf Wurfen.

Kann er die Nullhypothese H0: p = 16 auf dem Signifikanzniveau 5% verwerfen?

Aufgabe 6.2.6. Jemand behauptet, aussersinnliche Wahrnehmungen zu besitzen. Um dieseBehauptung zu uberprufen, wird das Glucksrad (siehe Abbildung 6.2.vi) zehn mal gedreht.Die Versuchsperson errat sieben Ausfalle richtig. Formulieren Sie eine Nullhypothese und

A

p =1

3

B

p =1

3

C

p =1

3

Abbildung 6.2.vi: Glucksrad

untersuchen Sie, ob sie auf dem Signifikanzniveau 5% zu verwerfen ist?

Aufgabe 6.2.7. Ein Arzt behauptet, dass er eine alternative Methode besitze, mit der er mit80%Wahrscheinlichkeit, das Geschlecht eines Kindes Monate vor der Geburt bestimmen kann.Um seine Behauptung zu testen, wird folgende Entscheidungsregel verwendet: Wir lassen denArzt 14 Voraussagen treffen. Wenn die Anzahl der Erfolge X grosser oder gleich 11 ist, wollenwir seine Behauptung akzeptieren. Wenn hingegen X < 11 ist, wird sie verworfen. Wie grossist die Wahrscheinlichkeit, dass

a. seine Methode verworfen wird, falls sie wertlos ist, d.h. wenn H0: p = 12?

b. seine Methode verworfen wird, obwohl er recht, d.h. wenn H0: p = 45?

Das Signifikanzniveau betrage 5%.

Losungen

Losung 6.2.5.

a. Unter der Voraussetzung der Nullhypothese H0 : p = 16 besteht nur eine Wahrschein-

lichkeit von etwa 7.4%, dass der Wurfel in drei Versuchen 2 oder 3 Sechsen zeigt: Null-hypothese annehmen.

Page 67: Wahrscheinlichkeitstheorie undStatistik (wst)web.fhnw.ch/.../Stochastik-Skript-Marcel-Steiner.pdf · Anwendungsgebiete der Statistik sind die Industrie und die Technik, die Physik,

6.3. Die Poissonverteilung 59

b. Unter der Voraussetzung der Nullhypothese H0 : p = 16 besteht nur eine Wahrschein-

lichkeit von etwa 3.5%, dass der Wurfel in funf Versuchen 3, 4 oder 5 Sechsen zeigt:Nullhypothese ablehnen.

Losung 6.2.6. Unter der Voraussetzung der Nullhypothese H0 : p = 13 besteht nur eine

Wahrscheinlichkeit von etwa 2.0%, dass die Person sieben oder mehr Mal richtig tippt: Null-hypothese ablehnen.

Losung 6.2.7.

a. Unter der Voraussetzung der Nullhypothese H0 : p = 12 besteht nur eine Wahrschein-

lichkeit von etwa 2.9%, dass er recht hat: Nullhypothese ablehnen.

b. Unter der Voraussetzung der Nullhypothese H0 : p = 45 besteht nur eine Wahrschein-

lichkeit von etwa 69.8%, dass er recht hat: Nullhypothese annehmen. Obwohl er rechthat, besteht die Wahrscheinlichkeit von 30.2%, ihn als Lugner abzustempeln.

6.3 Die Poissonverteilung

Bei vielen Anwendungen, die eigentlich mit der Binomialverteilung zusammenhangen, ist dieErfolgswahrscheinlichkeit p beim einzelnen Experiment klein, das heisst, der Erfolg ist einseltenes Ereignis. Gleichzeitig ist die Anzahl n der Ausfuhrungen sehr gross. In einem solchenFall approximieren wir die Binomialverteilung durch die Poissonverteilung.

6.3.1 Poissonverteilung als Grenzfall der Binomialverteilung

Die Poissonverteilung ergibt sich, wenn n so gegen unendlich strebt, dass der Erwartungs-wert

µ = np

gegen einen endlichen Wert strebt. Das heisst wir konnen in der Binomialverteilung p = µn

und q = 1− µnsetzen. Dann erhalten wir

P (X = x) =

(n

x

)pxqn−x

=n(n− 1)(n − 2) · · · (n− x+ 1)

x!

(µn

)x (1− µ

n

)n−x

=µx

x!

(1− µ

n

)n· n(n− 1)(n − 2) · · · (n− x+ 1)

nx

(1− µ

n

)−x

=µx

x!

(1− µ

n

)n·((

1− 1

n

)(1− 2

n

)· · ·(1− x− 1

n

))·(1− µ

n

)−x

.

Denken wir daran, dass hier µ und x feste gegebene Zahlen sind, wahrend dem n gegenunendlich strebt. Wir erhalten einerseits

limn→+∞

(1− µ

n

)n= e−µ

und andererseits

limn→+∞

((1− 1

n

)(1− 2

n

)· · ·(1− x− 1

n

))·(1− µ

n

)−x

= (1 · 1 · · · 1) · 1 = 1

Page 68: Wahrscheinlichkeitstheorie undStatistik (wst)web.fhnw.ch/.../Stochastik-Skript-Marcel-Steiner.pdf · Anwendungsgebiete der Statistik sind die Industrie und die Technik, die Physik,

60 Kapitel 6. Diskrete Zufallsgrossen und Verteilungen

und damit die Grenzverteilung

P (X = x) =µx

x!e−µ.

Die entstandene Verteilung heisst Poissonverteilung mit dem positiven Parameter µ undx ∈ N0.Da die Poissonverteilung ein Grenzfall der Binomialverteilung ist, kann je nach Genauigkeits-anspruchen fur etwa p ≤ 0.1 und n ≥ 100 statt der Binomialverteilung auch die Poissonver-teilung verwendet werden.Es gibt aber auch Aufgaben, die typische Aufgaben der Poissonverteilung selbst sind. Immerdann, wenn ein Versuch zwei Ausfallsmoglichkeiten hat und keine endliche Anzahl n vonVersuchen gegeben ist, ist die Poissonverteilung anzuwenden. Eine durchschnittliche AnzahlErfolge, der Erwartungswert µ, muss dann gegeben sein.In der praktischen Anwendung finden sich zahlreiche Beispiele fur das Auftreten poisson-verteilter Zufallsgrossen. So kann die Anzahl der auf einer Kreuzung innerhalb einer festenZeitspanne (eine Minute) vorbeifahrender Fahrradfahrer als poissonverteilt angesehen wer-den. Die Wahrscheinlichkeit dafur, dass in einer Minute genau x Fahrradfahrer vorbeifahren,wenn die Anzahl der vorbeifahrenden Fahrradfahrer je Minute im Durchschnitt µ betragt, istdann durch µx

x! e−µ gegeben. Weitere Beispiele fur poissonverteilte Zufallsgrossen sind:

• Die Anzahl der innerhalb einer kurzen Zeitspanne zerfallenden Atome eines radioaktivenPraparats.

• In einer Spinnerei die Anzahl der Fadenbruche innerhalb einer vorgegebenen Zeitspannebei einer bestimmten Garnsorte.

• Die Anzahl der wahrend einer festen Zeit beobachteten Sternschnuppen.

Dass die Poissonverteilung eine Wahrscheinlichkeitsverteilung ist, lasst sich mit Hilfe derReihenentwicklung der Exponentialfunktion einsehen.

∞∑

x=0

P (X = x) =

∞∑

x=0

µx

x!e−µ = e−µ

∞∑

x=0

µx

x!= e−µeµ = 1

Der Stichprobenraum der Poissonverteilung ist die Menge S = N0.Um die zum Teil recht muhsamen Berechnungen der Wahrscheinlichkeiten zu vereinfachen,bedienen wir uns Taschenrechner oder Computer, in Excel benutzen wir den Befehl POISSON.

6.3.2 Erwartungswert und Varianz der Poissonverteilung

Der Erwartungswert der Poissonverteilung ergibt einerseits sich aus der Konstruktion alsGrenzfall der Binomialverteilung zu µ und andererseits durch direkte Berechnung mit Hilfevon Formel 6.1.a

E(X) =

∞∑

x=0

x · P (X = x) = e−µµ

∞∑

x=1

µx−1

(x− 1)!= e−µµ

∞∑

x=0

µx

x!= e−µeµµ = µ.

Die Varianz oder Streuung der Poissonverteilung folgt aus folgender Uberlegung: Bekannt-lich ist die Varianz der Binomialverteilung Var(X) = npq. Zur Herleitung der Poissonvertei-lung betrachteten wir den Grenzubergang n → +∞ unter Berucksichtigung, dass das Produkt

Page 69: Wahrscheinlichkeitstheorie undStatistik (wst)web.fhnw.ch/.../Stochastik-Skript-Marcel-Steiner.pdf · Anwendungsgebiete der Statistik sind die Industrie und die Technik, die Physik,

6.3. Die Poissonverteilung 61

µ = np gegen einen endlichen Wert strebte. Damit folgt im Grenzubergang

Var(X) = limn→+∞

npq = limn→+∞

µ(1− µ

n

)= µ.

Somit folgt fur die Standardabweichung der Poissonverteilung

σ =√µ.

Bei der Poissonverteilung sind Erwartungswert und Varianz gleich dem Parameter µ.

Beispiel 6.3.1. Die Wahrscheinlichkeit dafur, dass ein Produkt einem Qualitatstest nichtgenugt, betrage p = 0.001. Wir bestimmen die Wahrscheinlichkeit, dass von 5000 Produktenmindestens zwei die Prufung nicht uberstehen.

Diese Aufgabe ist eigentlich eine Aufgabe der Binomialverteilung. Sie kann aber wegen demkleinen p und dem grossen n naherungsweise als Aufgabe der Poissonverteilung betrachtetwerden.

Es sei n = 5000, p = 0.001 also µ = np = 5, und X bezeichne die Anzahl Waren, die diePrufung nicht bestehen. Dann gilt

P (X < 2) = P (X = 0) + P (X = 1) =50

0!e−5 +

51

1!e−5 = 6e−5 = 0.04,

also P (X ≥ 2) = 1− P (X < 2) = 0.96.

Aufgaben

Aufgabe 6.3.1. Zeichnen Sie je ein Histogramm fur die Poissonverteilung mit µ = 2 undµ = 8.

Aufgabe 6.3.2. Die Wahrscheinlichkeit dafur, dass ein Teilnehmer im Verlaufe einer Stundebei einer Telefonzentrale anruft, betrage 1%. Die Zentrale bedient 300 Teilnehmer. Wie grossist die Wahrscheinlichkeit, dass wahrend einer Stunde genau vier Teilnehmer anrufen?

Aufgabe 6.3.3. Eine Firma stellt gleichartige Zubehorteile fur Kraftfahrzeuge in grosser Zahlher. Wir wissen, dass im Mittel 0.5% fehlerhaft sind. Wie gross ist die Wahrscheinlichkeit,dass die Lieferung von 1000 genau 10 schadhafte Stucke enthalt?

Aufgabe 6.3.4. Wie gross ist die Wahrscheinlichkeit, dass von einer Gruppe mit 730 Per-sonen wenigstens drei am Oster- oder Pfingstsonntag geboren sind, wenn die Geburtstagezufallig verteilt sind?

Aufgabe 6.3.5. Eine Stadt hat durchschnittlich zwei schwere Unfalle pro Woche. Wie grossist die Wahrscheinlichkeit fur mehr als funf Unfalle in einer Woche?

Aufgabe 6.3.6. In einem Hafen laufen wochentlich im Mittel funf Lastschiffe ein. Wie grossist die Wahrscheinlichkeit, dass dieser Mittelwert nachste Woche ubertroffen wird?

Aufgabe 6.3.7. Dozent Steiner macht durchschnittlich zwei Fehler pro getippte LATEX-Seite.Wie gross ist die Wahrscheinlichkeit, dass eine Seite fehlerfrei ist?

Page 70: Wahrscheinlichkeitstheorie undStatistik (wst)web.fhnw.ch/.../Stochastik-Skript-Marcel-Steiner.pdf · Anwendungsgebiete der Statistik sind die Industrie und die Technik, die Physik,

62 Kapitel 6. Diskrete Zufallsgrossen und Verteilungen

Aufgabe 6.3.8. Ein Prozent der Bevolkerung ist farbenblind. Welchen Umfang muss eineStichprobe mindestens haben, damit sie mit 95%-iger Wahrscheinlichkeit mindestens einefarbenblinde Person hat?

Aufgabe 6.3.9. Die Wahrscheinlichkeit, dass ein Fluggast, der einen Platz reserviert hat,nicht zum Flug erscheint, betragt vier Prozent. Die Fluggesellschaft weiss dies und verkauft75 fur 73 verfugbare Platze.Wie gross ist die Wahrscheinlichkeit, dass alle Platze besetzt sind und niemand auf dennachsten Flug warten muss?

a. Losen Sie die Aufgabe exakt mit einer Binomialverteilung.

b. Losen Sie die Aufgabe mit einer Poissonnaherung.

Aufgabe 6.3.10. Nach der Beobachtung von Rutherford und Geiger gibt eine radioaktiveSubstanz im Verlaufe von 7.5 s im Mittel 3.87 α-Teilchen ab. Bestimmen Sie die Wahrschein-lichkeit dafur, dass diese Substanz wahrend einer Sekunde mindestens ein α-Teilchen emittiert.

Aufgabe 6.3.11. Eine Firma verkauft Saatgut in Packchen von 1000 Samenkornern verpackt.Die Firma gibt an, dass durchschnittlich zwei Korner, die nicht der Sorte des Saatgutesangehoren, in einem Packchen zu erwarten sind. Eine Stichprobe ergibt aber 6 Fremdkorner.Formulieren Sie eine problembezogene Nullhypothese und untersuchen Sie, ob sie auf demSignifikanzniveau 5% abgelehnt werden kann.

Losungen

Losung 6.3.1. Siehe Excelfile: Losung 6.3.1 Poissonverteilung.xls

Losung 6.3.2. P (X = 4) = 0.1680

Losung 6.3.3. P (X = 10) = 0.0181

Losung 6.3.4. P (X ≥ 3) = 0.7619

Losung 6.3.5. P (X > 5) = 0.0166

Losung 6.3.6. P (X > 5) = 0.3840

Losung 6.3.7. Die Antwort “unmoglich” ist frech! P (X = 0) = 0.1353.

Losung 6.3.8. n ≥ 300

Losung 6.3.9.

a. P (X = 73) = 0.2255.

b. P (X = 73) = 0.2240.

Losung 6.3.10. P (X ≥ 1) = 0.4031

Losung 6.3.11. Unter der Voraussetzung der Nullhypothese H0 : µ = 2 besteht nur eineWahrscheinlichkeit von etwa 1.7%, dass ein Packchen 6 oder mehr Fremdkorner enthalt: Daeine solche Abweichung von der Nullhypothese nicht mit Zufall erklart werden kann, wird dieNullhypothese auf dem Niveau von 5% abgelehnt.

Page 71: Wahrscheinlichkeitstheorie undStatistik (wst)web.fhnw.ch/.../Stochastik-Skript-Marcel-Steiner.pdf · Anwendungsgebiete der Statistik sind die Industrie und die Technik, die Physik,

Kapitel 7

Stetige Zufallsgrossen undVerteilungen

Die Zahlenwerte, die eine stetige Zufallsgrosse X annehmen kann, sind uber ein ganzes In-tervall I verteilt, welches beschrankt oder unbeschrankt sein kann. Die Wahrscheinlichkeits-verteilung kann nun nicht mehr als Liste angegeben werden. Zu jedem moglichen Wert x ∈ I

wird der Wert f(x) Wahrscheinlichkeitsdichte f angegeben. Es kann nur davon gesprochenwerden, dass x mit einer bestimmten Wahrscheinlichkeit in einem gewissen Teilintervall desIntervalls I liegt.

7.1 Stetige Zufallsgrossen und Wahrscheinlichkeitsdichten

Beispiele von stetigen Zufallsgrossen sind viele physikalische Messungen. Wir konnen uns dieVerhaltnisse bei stetigen Zufallsgrossen plausibel machen, wenn wir die Werte der diskretenZufallsgrosse beliebig zusammenrucken lassen. Aus den einzelnen Strecken, die im Histogrammeiner solchen auftreten, wird bei einer stetigen Zufallsgrosse eine Flache, die durch die x-Achse und die Kurve y = f(x) der so genannten Wahrscheinlichkeitsdichte f begrenztwird (siehe Abbildung 7.1.i). Dabei entspricht die Wahrscheinlichkeit, dass die Zufallsgrosse

x1 x2

P

y = f(x)

a b

y

x

Abbildung 7.1.i: Wahrscheinlichkeitsdichte

X einen Wert zwischen x1 und x2 gerade der Masszahl der Flache unter der Kurve y = f(x)

63

Page 72: Wahrscheinlichkeitstheorie undStatistik (wst)web.fhnw.ch/.../Stochastik-Skript-Marcel-Steiner.pdf · Anwendungsgebiete der Statistik sind die Industrie und die Technik, die Physik,

64 Kapitel 7. Stetige Zufallsgrossen und Verteilungen

uber dem Intervall [x1, x2] ⊆ [a, b], das heisst

P (x1 ≤ X ≤ x2) =

∫ x2

x1

f(x) dx.

Fur die Verteilungsfunktion F einer stetigen Zufallsgrosse X, die jeden Wert in einembestimmten Intervall annehmen kann, gilt

F (x) = P (X ≤ x) =

∫ x

a

f(x) dx =

∫ x

a

f(t) dt.

Die nichtnegative Funktion f heisst Wahrscheinlichkeitsdichte von X. Wir stellen diewichtigsten Eigenschaften der Funktionen F und f zusammen. Im Folgenden sei das Inter-vall I = ]−∞,∞[. Bei einem (einseitig) beschrankten Intervall sind die Integrationsgrenzenentsprechend abzuandern.

1. Die Funktion F ist stetig, monoton wachsend mit F (−∞) = 0 und F (+∞) = 1.

2. Der Gesamtflacheninhalt unter der Wahrscheinlichkeitsdichtekurve ist gleich 1, d.h.

∫ ∞

−∞f(x) dx = 1.

3. Es gilt ddxF (x) = F ′(x) = f(x) fur alle x ∈ R.

4. Die Wahrscheinlichkeit ein Ereignis zwischen x1 und x2 zu erhalten, betragt

P (x1 ≤ X ≤ x2) =

∫ x2

x1

f(x) dx = F (x2)− F (x1).

x

f(x)

1F (x)

y

x

Abbildung 7.1.ii: Verteilungsfunktion F und Wahrscheinlichkeitsdichte F ′ = f .

7.1.1 Die Gleichverteilung

Wir betrachten die Wahrscheinlichkeitsdichte

f(x) =

1 wenn 0 ≤ x ≤ 10 sonst.

Page 73: Wahrscheinlichkeitstheorie undStatistik (wst)web.fhnw.ch/.../Stochastik-Skript-Marcel-Steiner.pdf · Anwendungsgebiete der Statistik sind die Industrie und die Technik, die Physik,

7.1. Stetige Zufallsgrossen und Wahrscheinlichkeitsdichten 65

x1 x2

P

1y = f(x)

1

y

x

Abbildung 7.1.iii: Die Wahrscheinlich-keitsdichte der Gleichverteilung

y = F (x)

1

1

y

x

Abbildung 7.1.iv: Die Verteilungsfunktionder Gleichverteilung

Hierbei handelt es sich um die so genannte Gleichverteilung im Intervall I = [0, 1] (sieheAbbildung 7.1.iii).Naturlich gilt auch bei der Gleichverteilung die Normierungsbedingung (2) aus Kapitel 7.1,d.h. ∫ ∞

−∞f(x) dx =

∫ 1

01 dx = 1.

Die Wahrscheinlichkeit, dass die Zufallsgrosse X einen Wert zwischen x1 und x2 annimmtbetragt

P (x1 ≤ X ≤ x2) =

∫ x2

x1

f(x) dx =

∫ x2

x1

1 dx = x2 − x1.

Die Verteilungsfunktion ergibt sich hier zu

F (x) =

0 wenn x < 0x wenn 0 ≤ x ≤ 11 wenn 1 < x

(vgl. Abbildung 7.1.iv).

Aufgaben

Aufgabe 7.1.1. Es sei die Funktion

f(x) =

ax wenn 0 ≤ x ≤ 10 sonst

gegeben, wobei a ein positiver Parameter bezeichnet.

a. Bestimmen Sie den Parameter a so, dass f auf dem Intervall I = [0, 1] eine Wahrschein-lichkeitsdichte ist.

b. Berechnen Sie die Verteilungsfunktion F zu f und veranschaulichen Sie sich diese Funk-tionen in einem Grafen.

Page 74: Wahrscheinlichkeitstheorie undStatistik (wst)web.fhnw.ch/.../Stochastik-Skript-Marcel-Steiner.pdf · Anwendungsgebiete der Statistik sind die Industrie und die Technik, die Physik,

66 Kapitel 7. Stetige Zufallsgrossen und Verteilungen

c. Berechnen Sie die folgenden Wahrscheinlichkeiten fur eine stetige Zufallsgrosse, diegemass dem obigen f verteilt ist und interpretieren Sie die Resultate geometrisch:

P

(1

3≤ X ≤ 3

4

), P

(X ≤ 1

2

)und P

(3

4≤ X

).

Aufgabe 7.1.2. Es sei die Funktion

f(x) =a

1 + x2

gegeben, wobei a ein positiver Parameter bezeichnet.

a. Bestimmen Sie den Parameter a so, dass f auf dem Intervall I = R eine Wahrschein-lichkeitsdichte ist.

b. Berechnen Sie die Verteilungsfunktion F zu f und veranschaulichen Sie sich diese Funk-tionen in einem Grafen.

c. Berechnen Sie die folgenden Wahrscheinlichkeiten fur eine stetige Zufallsgrosse, diegemass dem obigen f verteilt ist:

P

(0 ≤ X ≤ 1

2

), P

(X ≤ −1

4

)und P

(3

2≤ X

).

Losungen

Losung 7.1.1.

a. a = 2

b.

F (x) =

0 wenn x < 0x2 wenn 0 ≤ x ≤ 11 wenn 1 < x

c. P(13 ≤ X ≤ 3

4

)= 0.451, P

(X ≤ 1

2

)= 0.25 und P

(34 ≤ X

)= 0.438

Losung 7.1.2.

a. a = 1π

b. F (x) = 12 + 1

πarctan(x)

c. P(0 ≤ X ≤ 1

2

)= 0.148, P

(X ≤ −1

4

)= 0.422 und P

(32 ≤ X

)= 0.187

Page 75: Wahrscheinlichkeitstheorie undStatistik (wst)web.fhnw.ch/.../Stochastik-Skript-Marcel-Steiner.pdf · Anwendungsgebiete der Statistik sind die Industrie und die Technik, die Physik,

7.1. Stetige Zufallsgrossen und Wahrscheinlichkeitsdichten 67

7.1.2 Erwartungswert und Varianz

Mit Hilfe der Analogie, dass der Summe bei einer diskreten Zufallsgrosse ein Integral bei einerstetigen Zufallsgrosse entspricht erhalten wir die Beziehung fur den Erwartungswert und dieVarianz.

Im Folgenden sei das Intervall I = ]−∞,∞[. Bei einem (einseitig) beschrankten Intervallsind die Integrationsgrenzen entsprechend abzuandern. Der Erwartungswert der stetigenZufallsgrosse X betragt

µ = E(X) =

∫ ∞

−∞xf(x) dx,

die Varianz oder Streuung ist

σ2 = Var(X) = E((X − µ)2

)=

∫ ∞

−∞(x− µ)2f(x) dx =

∫ ∞

−∞x2f(x) dx− µ2.

Beispiel 7.1.1. Wir betrachten noch einmal die Gleichverteilung aus Kapitel 7.1.1. DerErwartungswert ergibt sich folgendermassen

µ = E(X) =

∫ ∞

−∞xf(x) dx =

∫ 1

0x · 1 dx =

x2

2

∣∣∣∣1

0

=1

2

und die Varianz

σ2 =

∫ ∞

−∞(x− µ)2f(x) dx =

∫ 1

0

(x− 1

2

)2

· 1 dx =1

3

(x− 1

2

)3∣∣∣∣∣

1

0

=1

12.

Die Standardabweichung ist also σ = 12√3.

Aufgaben

Aufgabe 7.1.3. Es sei die Funktion

f(x) =

2x wenn 0 ≤ x ≤ 10 sonst

gegeben. Berechnen Sie den Erwartungswert und die Varianz.

Aufgabe 7.1.4. Es sei die Funktion

f(x) =2

π

1

1 + x2

fur x ∈ [−1, 1] gegeben. Berechnen Sie den Erwartungswert und die Varianz.

Losungen

Losung 7.1.3. µ = 23 und σ2 = 1

18

Losung 7.1.4. µ = 0 und σ2 = 4π− 1

Page 76: Wahrscheinlichkeitstheorie undStatistik (wst)web.fhnw.ch/.../Stochastik-Skript-Marcel-Steiner.pdf · Anwendungsgebiete der Statistik sind die Industrie und die Technik, die Physik,

68 Kapitel 7. Stetige Zufallsgrossen und Verteilungen

7.2 Die Normalverteilung

Die bekannteste und bei allen Problemen der Statistik am haufigsten verwendete Verteilungeiner stetigen Zufallsgrosse ist die Gausssche Normalverteilung.

7.2.1 Die standardisierte Normalverteilung

Die standardisierte Normalverteilung besitzt eine Dichtefunktion der Form

f(z) = ϕ(z, 0, 1) =1√2π

e−z2

2 fur −∞ < z < ∞.

Es sei Z eine standardnormalverteilte Zufallsgrosse, dann schreiben wir Z ∼ N (0, 1).

1−1

1 1

z0

ϕ(z, 0, 1)

Abbildung 7.2.i: Die Wahrscheinlichkeitsdichte ϕ(z, 0, 1) der standardisierten Normalvertei-lung

Naturlich gilt auch bei der standardisierten Normalverteilung die Normierungsbedingung (2)aus Kapitel 7.1.In der Tat: Aus Grunden der Symmetrie der Wahrscheinlichkeitsdichte gilt

∫ ∞

−∞f(z) dz =

1√2π

∫ ∞

−∞e−

z2

2 dz =2√2π

∫ ∞

0e−

z2

2 dz.

Wir substituieren u2 = z2

2 also dz =√2 du. Es folgt1

2√2π

∫ ∞

0e−

z2

2 dz =√2

2√2π

∫ ∞

0e−u2

du =√2

2√2π

√π

2= 1.

Da der Integrand keine elementare Stammfunktion besitzt, benutzen wir zur expliziten Be-rechnung solcher Integrale meistens Tabellen oder einen Computer, z.B. in Excel mit demBefehl NORMVERT oder STANDNORMVERT. Mit Tafel T.1 lassen sich alle Aufgabenstellungenzur standardisierten Normalverteilung losen. Sie enthalt die Werte fur das bestimmte Integral

P (Z ≤ z) = Φ(z, 0, 1) =1√2π

∫ z

−∞e−

z2

2 dz

in Funktion von z ∈ R (siehe Abbildung 7.2.ii). Ferner gilt, wie entsprechend bei allen Wahr-

1Das folgende bestimmte Integral, dessen Berechnung schwierig ist, da es keine elementare Stammfunktionbesitzt, kann in Kapitel A.3 gefunden werden

0

e−u2

du =

√π

2.

Es dient zur weiteren Berechnung der auftretenden Integrale.

Page 77: Wahrscheinlichkeitstheorie undStatistik (wst)web.fhnw.ch/.../Stochastik-Skript-Marcel-Steiner.pdf · Anwendungsgebiete der Statistik sind die Industrie und die Technik, die Physik,

7.2. Die Normalverteilung 69

0 z

Φ(z, 0, 1)

z

ϕ(z, 0, 1)

Abbildung 7.2.ii: Verteilung Φ(z, 0, 1) der standardisierten Normalverteilung N (0, 1)

scheinlichkeitsverteilungen: Die Wahrscheinlichkeit dafur, dass die standardnormalverteilteZufallsgrosse Z ∼ N (0, 1) Werte zwischen z1 und z2 annimmt, betragt

P (z1 ≤ Z ≤ z2) =1√2π

∫ z2

z1

e−z2

2 dz = Φ(z2, 0, 1) − Φ(z1, 0, 1).

Beispiel 7.2.1. Es sei Z ∼ N (0, 1) eine standardnormalverteilte Zufallsgrosse. Wir berechnendie Wahrscheinlichkeit

P (1 ≤ Z ≤ 2.45) =1√2π

∫ 2.45

1e−

z2

2 dz = Φ(2.45, 0, 1) − Φ(1, 0, 1).

In der Tafel T.1 finden wir Φ(1, 0, 1) = 0.8413 und Φ(2.45, 0, 1) = 0.9929. Also folgt durchSubtraktion die gesuchte Wahrscheinlichkeit

P (1 ≤ Z ≤ 2.45) = 0.9929 − 0.8413 = 0.1516.

Bemerkung 7.2.1. Naturlich gilt Φ(−∞, 0, 1) = 0 und Φ(+∞, 0, 1) = 1. Wieso?

Aufgaben

Aufgabe 7.2.1. Bestimmen Sie die folgenden Wahrscheinlichkeiten einer standardisiertenNormalverteilung, d.h. Z ∼ N (0, 1).

a. Ein-σ-Bereich: P (−1 ≤ Z ≤ 1)

b. Zwei-σ-Bereich: P (−2 ≤ Z ≤ 2)

c. Drei-σ-Bereich: P (−3 ≤ Z ≤ 3)

d. P (Z ≤ 1)

e. P (|Z| ≥ 12)

f. P (−3 ≤ Z ≤ 1)

Aufgabe 7.2.2. Beweisen Sie, dass die standardisierte Normalverteilung den Erwartungswertµ = 0 und die Varianz σ2 = 1 besitzt.

Page 78: Wahrscheinlichkeitstheorie undStatistik (wst)web.fhnw.ch/.../Stochastik-Skript-Marcel-Steiner.pdf · Anwendungsgebiete der Statistik sind die Industrie und die Technik, die Physik,

70 Kapitel 7. Stetige Zufallsgrossen und Verteilungen

Losungen

Losung 7.2.1.

a. P (−1 ≤ Z ≤ 1) = 0.6827

b. P (−2 ≤ Z ≤ 2) = 0.9545

c. P (−3 ≤ Z ≤ 3) = 0.9973

d. P (Z ≤ 1) = 0.8413

e. P (|Z| ≥ 12) = 0.6171

f. P (−3 ≤ Z ≤ 1) = 0.8400

Losung 7.2.2. Benutzen Sie das bestimmte Integral∫∞0 e−u2

du =√π2 .

7.2.2 Die Normalverteilung mit den Parametern µ und σ2

Die stetige Zufallsgrosse X, die alle reellen Werte zwischen −∞ und +∞ annehmen kann,besitzt eine Normalverteilung mit den Parametern µ und σ2, wenn ihre Dichte durch

f(x) = ϕ(x, µ, σ2) =1√2πσ2

e−(x−µ)2

2σ2 fur −∞ < x < ∞

gegeben ist. Symbolisch schreiben wir X ∼ N (µ, σ2). Bei bekannten Werten µ und σ ist dieGestalt der Dichtefunktion vollig bestimmt (siehe Abbildung 7.2.iii).

µ + σµ − σ

σ σ

ϕ(x, µ, σ2)

Abbildung 7.2.iii: Die Wahrscheinlichkeitsdichte ϕ(x, µ, σ2) der Normalverteilung mit denParametern µ und σ2

Tragen wir die Dichte ϕ in Abhangigkeit von x in ein kartesisches Koordinatensystem ein,so ergibt sich die bekannte Gestalt der Gaussschen Glockenkurve. Das Maximum von ϕ

liegt bei x = µ und betragt 1√2πσ2

. Die Dichte ist symmetrisch bezuglich x = µ und nahert

sich fur x → ±∞ asymptotisch der x-Achse. Die Wendepunkte liegen bei x = µ ± σ. Damitist die Glockenkurve um so hoher und steiler, je kleiner σ ist.Die Verteilungsfunktion erhalten wir durch Integration der Dichte

F (x) = Φ(x, µ, σ2) =1√2πσ2

∫ x

−∞e−

(x−µ)2

2σ2 dx.

Page 79: Wahrscheinlichkeitstheorie undStatistik (wst)web.fhnw.ch/.../Stochastik-Skript-Marcel-Steiner.pdf · Anwendungsgebiete der Statistik sind die Industrie und die Technik, die Physik,

7.2. Die Normalverteilung 71

Fur obiges Integral gibt es keine elementare Stammfunktion, dies ist aber kein Problem, dafur die Anwendungen stets Tafeln oder Computerprogramme (z.B. Excel) verwendet werdenkonnen.

Bemerkung 7.2.2. Naturlich gilt Φ(−∞, µ, σ2) = 0 und Φ(+∞, µ, σ2) = 1. Wieso?

Durch eine Massstabsanderung auf der Koordinatenachse und einer Nullpunktverschiebungauf der x-Achse

z =x− µ

σ

kann von der Normalverteilung mit den Parametern µ und σ2 zur standardisierte Nor-malverteilung mit den Parametern µ = 0 und σ2 = 1 ubergegangen werden.Der Erwartungswert und die Varianz oder Streuung der normalverteilten ZufallsgrosseX betragt

E(X) = µ und Var(X) = σ2.

Die Parameter der Normalverteilung lassen sich damit leicht deuten: µ ist der Erwartungswertder Zufallsgrosse X und σ2 die Varianz.

7.2.3 Transformation auf die standardisierte Normalverteilung

Zur konkreten Berechnung von Wahrscheinlichkeiten bei der Normalverteilung mit den Pa-rametern µ und σ2 benutzen wir einen Computer oder die Tafel T.1 fur die standardisierteNormalverteilung, nachdem wir die Grenzen des interessierenden Intervalls transformierthaben. Der ganze Rechnungsablauf stellt nichts anderes dar, als eine Substitution eines Inte-grals. Es gilt namlich mit der Substitution

z =x− µ

σ

die Beziehung (siehe Abbildung 7.2.iv)

P (x1 ≤ X ≤ x2) =1√2πσ2

∫ x2

x1

e−(x−µ)2

2σ2 dx = Φ(x2, µ, σ2)− Φ(x1, µ, σ

2)

=1√2π

∫ x2−µ

σ

x1−µ

σ

e−z2

2 dz = Φ(x2−µσ

, 0, 1)− Φ

(x1−µσ

, 0, 1)

und im Falle von unbeschrankten Intervallen (siehe Abbildungen 7.2.v und 7.2.vi)

P (X ≤ x2) = Φ(x2, µ, σ2) = Φ

(x2−µσ

, 0, 1),

P (x1 ≤ X) = 1− P (X < x1) = 1− Φ(x1, µ, σ2) = 1− Φ

(x1−µσ

, 0, 1).

Die letzte Beziehung folgt direkt aus der Symmetrie der Normalverteilung bezuglich x = µ

und der Normierung des gesamten Flacheninhalts unter der Glockenkurve auf Eins. Die expli-zite Berechnung der gesuchten Wahrscheinlichkeiten wird also so durchgefuhrt, dass aus denGrenzen x1 und x2 (diese konnen x1 = −∞ und/oder x2 = ∞ betragen) die entsprechenden

z1 =x1 − µ

σund z2 =

x2 − µ

σ

berechnet werden und dann mit Hilfe von Tafel T.1 wie in Kapitel 7.2.1 vorgegangen wird.

Page 80: Wahrscheinlichkeitstheorie undStatistik (wst)web.fhnw.ch/.../Stochastik-Skript-Marcel-Steiner.pdf · Anwendungsgebiete der Statistik sind die Industrie und die Technik, die Physik,

72 Kapitel 7. Stetige Zufallsgrossen und Verteilungen

x2x1

P (x1 ≤ X ≤ x2)

x

ϕ(x, µ, σ2)

Abbildung 7.2.iv: Die Wahrscheinlichkeit P (x1 ≤ X ≤ x2) = Φ(x2, µ, σ2)− Φ(x1, µ, σ

2)

x2

P (X ≤ x2)

x

ϕ(x, µ, σ2)

Abbildung 7.2.v: Die WahrscheinlichkeitP (X ≤ x2) = Φ(x2, µ, σ

2).

x1

P (x1 ≤ X)

x

ϕ(x, µ, σ2)

Abbildung 7.2.vi: Die WahrscheinlichkeitP (x1 ≤ X) = 1− Φ(x1, µ, σ

2)

Beispiel 7.2.2. Es sei X ∼ N (2, 4) eine normalverteilte Zufallsgrosse mit den Parameternµ = 2 und σ2 = 4. Wir berechnen die Wahrscheinlichkeit

P (1 ≤ X ≤ 2.45) =1√2π · 4

∫ 2.45

1e−

(x−2)2

2·4 dx.

Also folgt mit der Transformation

z1 =1− 2

2= −0.5 und z2 =

2.45− 2

2= 0.225,

dass

P (1 ≤ X ≤ 2.45) = P (−0.5 ≤ Z ≤ 0.225) = Φ(0.225, 0, 1) − Φ(−0.5, 0, 1).

In der Tafel T.1 finden wir Φ(−0.5, 0, 1) = 1−Φ(0.5, 0, 1) = 0.3085 und Φ(0.225, 0, 1) = 0.5890.Also folgt durch Subtraktion die gesuchte Wahrscheinlichkeit

P (1 ≤ X ≤ 2.45) = 0.5890 − 0.3085 = 0.2805.

7.2.4 Quantile der standardisierten Normalverteilung

Wichtig im Hinblick auf statistische Hypothesentests ist die umgekehrte Frage nach denSchranken, bis zu denen die von der Zufallsgrosse angenommenen Werte mit einer vorge-gebenen Wahrscheinlichkeit liegen mussen.

Page 81: Wahrscheinlichkeitstheorie undStatistik (wst)web.fhnw.ch/.../Stochastik-Skript-Marcel-Steiner.pdf · Anwendungsgebiete der Statistik sind die Industrie und die Technik, die Physik,

7.2. Die Normalverteilung 73

Es sei q ∈ ]0, 1[ und Z ∼ N (0, 1) standardnormalverteilt. Dann suchen wir zq ∈ R so, dass

P (Z ≤ zq) = q

gilt (siehe Abbildung 7.2.vii). Wegen der Symmetrie der Dichte der standardisierten Normal-verteilung befindet sich eine gleich grosse Flache links von zq wie rechts von z1−q, so dass

z1−q = −zq

gilt. Wir nennen zq das q-Quantil oder q-Perzentil der standardisierten Normalverteilung.

0 zq

q

1 − q

z

ϕ(z, 0, 1)

Abbildung 7.2.vii: Das q-Quantil zq der standardisierten Normalverteilung N (0, 1). Es giltz1−q = −zq.

Je nach Problemstellung betrachten wir dabei

1. einseitige Quantile: Bis zu welchem Wert zα sind alle Werte mit Wahrscheinlichkeitα zu erwarten? Hier wird nach dem Argument zα gefragt, fur das die Flache unter derDichtekurve von −∞ bis zα gerade gleich α wird, d.h.

P (Z ≤ zα) = α

(siehe Abbildung 7.2.viii).

2. zweiseitige Quantile: Innerhalb von welchem Intervall[zα

2, z1−α

2

]sind alle Werte mit

Wahrscheinlichkeit 1− α zu erwarten, d.h.

P(Z ≤ zα

2und z1−α

2≤ Z

)= 1− α

(siehe Abbildung 7.2.ix).

Einige haufig benutzte Quantile finden sich in Tafel T.2. Diese lassen sich auch mit einemComputerprogramm leicht berechnen, z.B. in Excel mit dem Befehl NORMINV.

7.2.5 Anwendung der Normalverteilung in der Fehlerrechnung

Die Normalverteilung wurde von Carl Friedrich Gauss (siehe Abbildung 7.2.x) im Jahre 1809im Zusammenhang mit seiner Theorie der Beobachtungsfehler entdeckt. Sie wird daher auchFehlerkurve genannt. Fuhren wir in der Praxis wiederholt Messungen an ein und demselbenGegenstand, etwa der Lange eines Stabes oder Durchmessers einer Welle durch, so ergibtbekanntlich nicht jede Messung den gleichen Wert. Die erhaltenen Werte weisen kleinere oder

Page 82: Wahrscheinlichkeitstheorie undStatistik (wst)web.fhnw.ch/.../Stochastik-Skript-Marcel-Steiner.pdf · Anwendungsgebiete der Statistik sind die Industrie und die Technik, die Physik,

74 Kapitel 7. Stetige Zufallsgrossen und Verteilungen

0zα

α

1 − α

z

ϕ(z, 0, 1)

Abbildung 7.2.viii: Einseitiges Quantil zαder Standardnormalverteilung

0 z1−α

2

α2

1 − α

2

α2

z

ϕ(z, 0, 1)

Abbildung 7.2.ix: Zweiseitige Quantile zα2

und z1−α2der Standardnormalverteilung

Abbildung 7.2.x: Carl Friedrich Gauss, 1777-1855

grossere Abweichungen voneinander und von einem bestimmten “wahren” Wert, dem Mit-telwert, auf. Diese Abweichungen oder Beobachtungsfehler haben verschiedene Ursachen, wiezum Beispiel Schwankungen der Raumtemperatur, Einflusse der Umgebung auf das Mess-gerat, Ungenauigkeiten der Messskala, Wechsel im Prufpersonal usw. Nach ihrer Herkunftunterscheiden wir zwischen systematischen und zufalligen Fehlern. Die groben Fehler (z.B.Ablesefehler oder defekte Instrumente) schliessen wir von vornherein aus, da sie im Prinzipvermeidbar sind.

1. Systematische Fehler: Systematische Fehler sind oft nicht vermeidbar. Zu ihnengehoren Nullpunktsverschiebungen oder Skalenfehler. Die Ursache kann bei Mangelnan den Instrumenten liegen. Systematische Fehler konnen meistens beseitigt werden.

2. Zufallige Fehler: Sie ergeben sich aus dem Zusammenwirken zahlreicher Fehlerursa-chen, die vom Beobachter nicht erfasst oder beseitigt werden konnen. Zufallige Fehlersind unvermeidbar. Sie entstehen zum Beispiel aus Mangeln des Beobachters, Witte-rungseinflussen, Erschutterungen, Ablagerung von Staub usw. Solche Einflusse ergebenZufalligkeiten. Solche zufallige Fehler, Zufallsgrossen, verursachen bei Messungen Ab-weichungen nach beiden Seiten vom wahren Wert. Sprechweise: Die Messwerte streuen.

Mit Hilfe der Theorie der Grenzwertsatze konnen Verteilungen fur diese zufalligen Fehlergefunden werden. In den meisten Fallen, wo durch additive Uberlagerung einer grossen Anzahlvoneinander unabhangiger, zufalliger Effekte entstehen, wobei jeder dieser Effekte nur einenunbedeutenden Einfluss auf den zufalligen Gesamtfehler hat, ergibt sich als Grenzverteilung

Page 83: Wahrscheinlichkeitstheorie undStatistik (wst)web.fhnw.ch/.../Stochastik-Skript-Marcel-Steiner.pdf · Anwendungsgebiete der Statistik sind die Industrie und die Technik, die Physik,

7.2. Die Normalverteilung 75

die Normalverteilung. Diesem Sachverhalt liegt der Zentrale Grenzwertsatz zu Grunde,der aussagt, dass unter bestimmten Bedingungen jede Summe (unabhangiger) Zufallsgrossennaherungsweise normalverteilt ist. Aus diesem Grund konnen in den meisten praktischenAnwendungsfalle die Beobachtungsfehler bei Messvorgangen wenigstens naherungsweise alsnormalverteilt angesehen werden.

Aufgaben

Aufgabe 7.2.3. Berechnen Sie

a. die Wahrscheinlichkeit P (−4 ≤ X ≤ 8) wenn X ∼ N (2, 4);

b. die Wahrscheinlichkeit P (2 ≤ X) wenn X ∼ N (1, 9);

c. die Wahrscheinlichkeit P (|X| ≤ 1) wenn X ∼ N (−1, 16).

Aufgabe 7.2.4. Ein Werkstuck besitze die gewunschte Qualitat, wenn die Abweichung sei-ner Masse von den Nennwerten dem Absolutbetrag nach 3.45mm nicht uberschreiten. Diezufalligen Abweichungen der Abmessungen von ihren Nennwerten seien normalverteilt mitder Standardabweichung σ = 3mm. Systematische Abweichungen liegen nicht vor. Bestim-men Sie die mittlere Anzahl Werkstucke mit der gewunschten Qualitat, wenn 24 Werkstuckehergestellt werden.

Aufgabe 7.2.5. Eine Maschine stellt Metallplatten mit der mittleren Dicke µ = 8.00mmund der Standardabweichung σ = 0.05mm her. Fur die leicht schwankende Plattendicke geltedie Normalverteilung.

a. Mit wie viel Prozent Ausschuss ist zu rechnen, wenn die Dicke nicht uber 8.10mm liegensoll?

b. Mit wie viel Prozent Ausschuss ist zu rechnen, wenn die Dicke zwischen 7.92mm und8.08mm liegen soll?

c. Welche Abweichung von 8.00mm ist noch erlaubt bei hochstens 5% Ausschuss?

Aufgabe 7.2.6. Eine Maschine stellt Metallplatten mit der mittleren Dicke µ = 8.00mmund der Standardabweichung σ = 0.05mm her. Fur die leicht schwankende Plattendicke geltedie Normalverteilung.

a. Welche beidseitige Abweichung ∆x vom Mittelwert µ = 8.00mm musste zugelassenwerden, damit der Ausschuss nicht mehr als 4% betragt?

b. Wie viel Prozent Ausschuss sind zu erwarten, wenn die Platten nicht dunner als 7.95mmund nicht dicker als 8.12mm sein sollen?

Aufgabe 7.2.7. In einer Ebene liegen im Abstand L = 10 cm zwei parallele Geraden g1und g2, dabei ist g2 die y-Achse. Auf diese Ebene werfen wir Kreisscheiben mit dem RadiusR = 8cm. Die Lage der Kreismittelpunkte kann, wenn wir von der y-Richtung absehen, alsnormalverteilt in x-Richtung gelten, wobei das Streuzentrum auf der Geraden h liegt unddie Standardabweichung σ = 10 cm betragt. Der Abstand der beiden Geraden h und g2 seib = 5cm. Die Anordnung der Geraden sei von links nach rechts, zuerst g1, dann g2 und h.Bestimmen Sie die Wahrscheinlichkeit, dass bei einem Wurf der Kreis

Page 84: Wahrscheinlichkeitstheorie undStatistik (wst)web.fhnw.ch/.../Stochastik-Skript-Marcel-Steiner.pdf · Anwendungsgebiete der Statistik sind die Industrie und die Technik, die Physik,

76 Kapitel 7. Stetige Zufallsgrossen und Verteilungen

a. mindestens eine der Geraden g1 und g2 teilweise uberdeckt.

b. beide Geraden teilweise uberdeckt.

Aufgabe 7.2.8. Eine Abfullmaschine fullt ein Erzeugnis in Dosen. Das Nettogewicht ei-ner Dose ist eine normalverteilte Zufallsgrosse. Die Standardabweichung, als Mass fur diePrazision mit der die Maschine arbeitet, sei 8 g. Auf welchen Mittelwert ist die Maschineeinzustellen, wenn hochstens 5% aller Dosen weniger als 250 g enthalten sollen?

Aufgabe 7.2.9. Bei der Herstellung von Kondensatoren sei die Kapazitat eine normalver-teilte Zufallsgrosse mit µ = 5µF und σ = 0.02µF. Welcher Ausschussanteil ist zu erwarten,wenn die Kapazitat

a. mindestens 4.98µF betragen soll?

b. hochstens 5.05µF betragen soll?

c. um maximal 0.03µF vom Sollwert µ = 5µF abweichen darf?

Aufgabe 7.2.10. Der DurchmesserX von serienmassig gefertigten Kugeln sei normalverteilt.Von zwei Sieben weist das eine Locher mit einem Durchmesser von 10mm auf, das anderesolche mit einem Durchmesser von 13mm. Damit werden die Wahrscheinlichkeiten

P (X ≤ 10mm) = 0.1736 und P (X ≥ 13mm) = 0.1446

bestimmt. Wie lauten die Parameter µ und σ der Verteilung von X?

Aufgabe 7.2.11. Es sei X ∼ N (µ, σ2)-verteilt mit µ = 8 und σ = 3. Bestimmen Sie x so,dass P (x ≤ X ≤ 10) = 0.7 ist.

Losungen

Losung 7.2.3.

a. P (−4 ≤ X ≤ 8) = 0.9973

b. P (2 ≤ X) = 0.3694

c. P (|X| ≤ 1) = 0.1915

Losung 7.2.4. n = 18 Stuck

Losung 7.2.5.

a. P (8.10mm ≤ X) = 2.3%

b. P (X ≤ 7.92mm oder 8.08mm ≤ X) = 11.0%

c. ±0.098mm

Losung 7.2.6.

a. ∆x = ±0.1027mm

Page 85: Wahrscheinlichkeitstheorie undStatistik (wst)web.fhnw.ch/.../Stochastik-Skript-Marcel-Steiner.pdf · Anwendungsgebiete der Statistik sind die Industrie und die Technik, die Physik,

7.3. Normalverteilung als Grenzfall der Binomialverteilung 77

b. P (X ≤ 7.95mm oder 8.12mm ≤ X) = 16.7%

Losung 7.2.7.

a. P (−18 cm ≤ X ≤ 8 cm) = 0.6072

b. P (−8 cm ≤ X ≤ −2 cm) = 0.1452

Losung 7.2.8. µ = 263.2 g

Losung 7.2.9.

a. 1− P (4.98µF ≤ X) = 15.9%

b. 1− P (X ≤ 5.05µF) = 0.6%

c. P (|X − 5.00µF| ≥ 0.03µF) = 13.4%

Losung 7.2.10. Beachten Sie, dass der Durchmesser zwischen 10mm und 13mm liegen muss.Dann folgt µ = 11.41mm und σ = 1.5mm.

Losung 7.2.11. x = 2.9915

7.3 Normalverteilung als Grenzfall der Binomialverteilung

Die Binomialverteilung kann gemass Kapitel 6.3.1 fur kleine Erfolgswahrscheinlichkeiten p undgrosse Versuchsanzahl n durch die Poissonverteilung angenahert werden. Ist p nicht klein, sokonnen wir eine andere Naherung benutzen: die Normalverteilung.Es handelt sich dabei um einen Spezialfall des Zentralen Grenzwertsatzes, dem so genann-ten Grenzwertsatz von de Moivre und Laplace. Dieser besagt, dass eine binomialver-teilte Zufallsgrosse X mit Erwartungswert E(X) = np und Varianz Var(X) = np(1 − p),naherungsweise normalverteilt mit den Parametern

µ = np und σ2 = np(1− p)

ist. Danach konnen wir fur eine binomialverteilte Zufallsgrosse X fur grosses n die Naherungs-formel

P (x1 ≤ X ≤ x2) ≈ Φ

(x2 − np√np(1− p)

, 0, 1

)− Φ

(x1 − np√np(1− p)

, 0, 1

)(7.3.a)

verwenden. In der Literatur wird diese Naherung als Faustregel fur

n >9

p(1− p)

empfohlen2.Dieser Grenzubergang bedeutet geometrisch, dass sich dass Histogramm der Binomialvertei-lung immer mehr der Normalverteilung annahert (siehe Abbildungen 7.3.i und 7.3.ii). DerBeweis dieser Aussage finden Sie in jedem anstandigen Lehrbuch uber mathematische Statis-tik, zum Beispiel in [1].

2Wenn die Faustregel nicht erfullt ist, dann konnen wir mit einer so genannten Stetigkeitskorrektur

arbeiten, d.h.

P (x1 ≤ X ≤ x2) ≈ Φ

(

x2 +12− np

np(1− p), 0, 1

)

−Φ

(

x1 − 12− np

np(1− p), 0, 1

)

Die Korrekturen ± 12in den Transformationsformeln mussen vor allem bei kleinen Werten von σ2 = np(1− p)

verwendet werden. Sie stammen aus der Annaherung des Flacheninhalts durch Rechtecke.

Page 86: Wahrscheinlichkeitstheorie undStatistik (wst)web.fhnw.ch/.../Stochastik-Skript-Marcel-Steiner.pdf · Anwendungsgebiete der Statistik sind die Industrie und die Technik, die Physik,

78 Kapitel 7. Stetige Zufallsgrossen und Verteilungen

5 10 15 20 25 30 35 40 45 50

0.05

0.10

0.15

P (X = x)

x

Abbildung 7.3.i: Histogramm der Binomi-alverteilung bei p = q = 0.5 und n =50 und die Approximation mit der Nor-malverteilung mit den Parametern µ =25 und σ = 3.536. Die Faustregel n >

9p(1−p) = 36 ist erfullt.

5 10 15 20 25 30 35 40 45 50

0.05

0.10

0.15

P (X = x)

x

Abbildung 7.3.ii: Histogramm der Bino-mialverteilung bei p = 0.75, q = 0.25und n = 50 und die Approximation mitder Normalverteilung mit den Parameternµ = 37.5 und σ = 3.062. Die Faustregeln > 9

p(1−p) = 48 ist erfullt.

Beispiel 7.3.1 (Galtonsches Brett). Mit Hilfe eines Zufallsapparates von Francis Galton(siehe Abbildung 7.3.iii), des so genannten Galtonschen Brettes, konnen wir die Normal-verteilung experimentell erzeugen.

Wir lassen kleine Kugeln durch ein System von Nageln, die gemass der Abbildung 7.3.iv an-geordnet sind, hindurch rollen und in n+1 Facher, die nach der n-ten Nagelreihe angebrachtsind, fallen.

Abbildung 7.3.iii: Francis Galton, 1822-1911

Abbildung 7.3.iv: Galtonsches Brett, Bildaus [6]

Wir erhalten stets eine Verteilung der Kugeln auf die Facher, die sich durch eine Glockenkur-ve darstellen lasst. Wieso? Wir verfolgen eine Kugel auf ihrem Weg. Sie moge nach anstossenan einem Nagel der ersten Reihe den Weg links wahlen, nach dem Anstossen an einem Nagelder zweiten Reihe nochmals links, dann rechts, links, usw. Das Fach, das sie zuunterst errei-chen wird, ist nun offenbar durch diese Aufeinanderfolge von links und rechts, total n Mal,bestimmt. Diesen Vorgang konnen wir auch so charakterisieren: Wenn die Kugel den Weglinks einschlagt, notieren wir 0, wenn sie den Weg rechts einschlagt, notieren wir 1. Dann ist

Page 87: Wahrscheinlichkeitstheorie undStatistik (wst)web.fhnw.ch/.../Stochastik-Skript-Marcel-Steiner.pdf · Anwendungsgebiete der Statistik sind die Industrie und die Technik, die Physik,

7.3. Normalverteilung als Grenzfall der Binomialverteilung 79

ihr Fach durch 0 + 0 + 1 + 0 + · · · , also durch eine Summe von n zufalligen Grossen, die allenur die Werte Null oder Eins - jedes Mal mit der Wahrscheinlichkeit 1

2 - annehmen konnen.Diese Summen, und damit die Kugeln in den Fachern, sind nach dem Grenzwertsatz von deMoivre und Laplace normalverteilt (siehe Abbildung 7.3.iv).

Aufgaben

Aufgabe 7.3.1. Eine Fabrik produziert Prazisonswerkstucke, die mit einer Wahrscheinlich-keit p = 0.001 defekt sind. Wie gross ist die Wahrscheinlichkeit, dass eine Lieferung vonn = 500 Werkstucken mindestens 2 unbrauchbare Werkstucke enthalt.

a. Rechnen Sie genau mit einer Binomialverteilung.

b. Approximieren Sie mit einer Poissonverteilung.

c. Approximieren Sie mit einer Normalverteilung.

Aufgabe 7.3.2. In einem Land werden stundlich im Mittel 25 Kinder geboren. Wie gross istdie Wahrscheinlichkeit, dass die Zahl unter 20 sinkt.

Losungen

Losung 7.3.1.

a. PBinom(2 ≤ X) = 0.09013

b. PPoisson(2 ≤ X) ≈ 0.09020

c. Da 500 > 9p(1−p) = 9009 nicht erfullt ist, mussen wir die Stetigkeitskorrektur benutzen

und erhalten PNorm(2 ≤ X) ≈ 0.07855. (Im Gegensatz dazu wurden wir das unbrauch-bare Resultat PNorm(2 ≤ X) ≈ 0.016903 ohne Stetigkeitskorrektur erhalten.)

Losung 7.3.2. Die Anzahl der stundlichen Geburten besitzt Poissonverteilung mit µ = 25und σ = 5. Es folgt PPoisson(X ≤ 19) = 0.1336.Wegen dem relativ grossen Erwartungswert kann die Normalverteilung als Naherung verwen-det werden. Es folgt PNorm(X ≤ 19) ≈ 0.1357 (mit Stetigkeitskorrektur).

Page 88: Wahrscheinlichkeitstheorie undStatistik (wst)web.fhnw.ch/.../Stochastik-Skript-Marcel-Steiner.pdf · Anwendungsgebiete der Statistik sind die Industrie und die Technik, die Physik,

80 Kapitel 7. Stetige Zufallsgrossen und Verteilungen

Page 89: Wahrscheinlichkeitstheorie undStatistik (wst)web.fhnw.ch/.../Stochastik-Skript-Marcel-Steiner.pdf · Anwendungsgebiete der Statistik sind die Industrie und die Technik, die Physik,

Kapitel 8

Statistische Tests

Statistische Tests dienen dazu, an Hand von Stichproben Annahmen (sog. Hypothesen1)uber das Verteilungsgesetz in der Grundgesamtheit zu uberprufen. Oft existiert uber dieunbekannte Verteilungsfunktion F oder ihre unbekannten Parameter, wie zum Beispiel µ undσ2 bei der Normalverteilung oder p bei der Binomialverteilung, eine bestimmte Vorstellung.Diese wird in Gestalt einer Nullhypothese, die mit H0 bezeichnet wird, ausgedruckt. Wirdauch eine die Nullhypothese ausschliessende Alternative betrachtet, so bezeichnen wir dieseAlternativhypothese mit H1. Das Anliegen eines statistischen Tests zur Prufung von H0

gegen H1 ist es, eine Entscheidung daruber zu treffen, ob die aus einer konkreten Stichprobeentnommenen Angaben zur aufgestellten Hypothese H0 im Widerspruch stehen oder nicht,d.h., ob H0 abzulehnen ist oder nicht.

8.1 Das Prinzip des statistischen Tests

Wir beginnen mit einem ausfuhrlichen Einfuhrungsbeispiel, bei dem wir den Parameter p derBinomialverteilung einem statistischen Test unterziehen.

Beispiel 8.1.1. Bei 12000 Wurfen eines Wurfels wurden x = 2107 Sechsen gezahlt. Ist dieserWurfel unsymmetrisch, d.h. werden Sechsen bevorzugt gewurfelt?Uns scheint die Anzahl Sechsen ein bisschen zu gross. Da es aber in den modernen Natur- undIngenieurwissenschaften nicht erlaubt ist, aus dem Bauch Gefuhle zu aussern, benotigen wireinen rigorosen statistischen Test, um unseren Eindruck wissenschaftlich zu belegen. Dazubezeichnen wir mit X die Zufallsgrosse der Anzahl “Sechsen” unter n = 12000 Wurfen,und mit p die Wahrscheinlichkeit mit dem betrachteten Wurfel eine “Sechs” zu wurfeln.Anschliessend formulieren wir zwei sich ausschliessende Hypothesen:

Nullhypothese H0 : p ≤ 16 , d.h., der Wurfel ist symmetrisch.

Alternativhypothese H1 : p > 16 , d.h., es werden Sechsen bevorzugt gewurfelt.

Beachten Sie, dass sich die Hypothesen gegenseitig ausschliessen und sich nur auf die Ereig-nisse “Sechs” oder “nicht Sechs” beziehen. Wie die anderen Augenzahlen ausfallen ist nichtvon Belang.Unter der Voraussetzung der Nullhypothese H0 berechnen wir nun den Erwartungswert

E(X) = np = 12000 · 16 = 2000.

1Hypothese enthalt die Worter hypo-thesis = das Unter-Gestellte.

81

Page 90: Wahrscheinlichkeitstheorie undStatistik (wst)web.fhnw.ch/.../Stochastik-Skript-Marcel-Steiner.pdf · Anwendungsgebiete der Statistik sind die Industrie und die Technik, die Physik,

82 Kapitel 8. Statistische Tests

In unserem Experiment stellen wir eine Abweichung vom Erwartungswert

x− E(X) = 2107 − 2000 = 107

fest. Nun bestimmen wir die Wahrscheinlichkeit P (2107 ≤ X) einer so grossen oder grosserenAbweichung vom Erwartungswert. Dazu benutzen wir den Grenzwertsatz von de Moivreund Laplace (vgl. Gleichung (7.3.a)), d.h., wir approximieren2 die Binomialverteilung miteiner Normalverteilung mit den Parametern

µ = np = 12000 · 16 = 2000 und σ2 = np(1− p) = 12000 · 1

6 · 56 = 16662

3 .

Wir erhalten nun mit Tafel T.1

P (2107 ≤ X) ≈ 1− Φ

2107 − 2000√

166623

, 0, 1

= 1− Φ (2.621, 0, 1)

= 1− 0.9956 = 0.0044.

Die Wahrscheinlichkeit unter der Voraussetzung der Nullhypothese mindestens so viel Abwei-chung vom Erwartungswert zu erhalten, ist somit ausserordentlich klein. Dies erlaubt uns,die Nullhypothese abzulehnen. Die Irrtumswahrscheinlichkeit dieses Schlusses entsprichtdem berechneten Wert P (2107 ≤ X) ≈ 0.0044.

Im Allgemeinen mussen wir uns entscheiden, wann eine berechnete Abweichung zur Ableh-nung der Nullhypothese fuhren soll. Dazu wird eine Schranke α ∈ ]0, 1[, das so genannte Si-gnifikanzniveau, gewahlt. Ist die berechnete Wahrscheinlichkeit der Abweichung kleiner alsdas Signifikanzniveau, so wird die Nullhypothese abgelehnt, sonst angenommen. Die zulassigeGrosse des Signifikanzniveaus α hangt stark vom Fachgebiet ab und ist eine Vereinbarungs-sache. Haufig verwendete Niveaus sind α = 0.01, 0.05 und 0.1.

Das Prinzip eines statistischen Tests oder Signifikanztest lasst sich in folgenden Schrittenzusammenfassen:

1. Aufstellen der Nullhypothese H0 und der Alternativhypothese H1 und Vorgabedes Signifikanzniveaus α.

2. Bestimmen einesAblehnungsbereichs in Abhangigkeit von α, fur den die Wahrschein-lichkeit, dass die Stichprobenfunktion Werte aus dem Ablehnungsbereich annimmt,hochstens gleich α ist.

3. Berechnung der Testgrosse aus der vorliegenden konkreten Stichprobe.

4. Statistischer Schluss: Liegt die Testgrosse im Ablehnungsbereichs, so wird H0 abge-lehnt, sonst wird H0 angenommen.

8.2 Einseitiger und zweiseitiger Test

In Beispiel 8.1.1 haben wir uns fur die Abweichung vom Erwartungswert (nach einer Seite)interessiert. Im Gegensatz dazu konnen wir auch Abweichungen nach beiden Seiten Beachtungschenken. Fur die Nullhypothese H0 und die Alternativhypothese H1 bieten sich nun dienachfolgenden Moglichkeiten an, die je nach Aufgabenstellung angewandt werden mussen.

2Die Approximation ist erlaubt, da die Faustregel np(1− p) = 1666 23> 9 erfullt ist.

Page 91: Wahrscheinlichkeitstheorie undStatistik (wst)web.fhnw.ch/.../Stochastik-Skript-Marcel-Steiner.pdf · Anwendungsgebiete der Statistik sind die Industrie und die Technik, die Physik,

8.2. Einseitiger und zweiseitiger Test 83

1. Zweiseitiger Test H0 : µ = µ0 und H1 : µ 6= µ0. Zur Konstruktion des Ablehnungs-bereiches wird der Flacheninhalt α symmetrisch auf beiden Seiten der Kurve aufge-teilt, und es ergibt sich einen zweiseitigen Ablehnungsbereich mit den beiden kritischenGrossen zα

2und z1−α

2. Die Abweichung zwischen dem Stichprobenparameter und dem

µ0 z1−α

2

α2

1 − α

2

α2

Annahmebereich AblehnungsbereichAblehnungsbereich

z

Abbildung 8.2.i: H1 : µ 6= µ0, zweiseitigeFragestellung mit den kritischen Grossenzα

2und z1−α

2.

hypothetischen Wert µ0 wird nur dem Absolutbetrag nach beurteilt.

2. Einseitiger Test H0 : µ ≤ µ0 und H1 : µ > µ0 (resp. H0 : µ ≥ µ0 und H1 : µ < µ0).Zur Konstruktion des Ablehnungsbereiches wird der Flacheninhalt α nur auf einer Seiteder Kurve abgeschnitten, und es ergibt sich einen einseitigen Ablehnungsbereich mit derkritischen Grosse zα (resp. z1−α). Die damit verbundene einseitige Fragestellung liegt

µ0zα

α

1 − α

Ablehnungsbereich Annahmebereich

z

Abbildung 8.2.ii: H1 : µ < µ0, einseiti-ge untere Fragestellung mit der kritischenGrosse zα.

µ0 z1−α

α

1− α

Annahmebereich Ablehnungsbereich

z

Abbildung 8.2.iii: H1 : µ > µ0, einseiti-ge obere Fragestellung mit der kritischenGrosse z1−α.

dann vor, wenn nur Abweichungen nach einer Seite interessieren, d.h., wenn es zumBeispiel darauf ankommt zu beurteilen, ob ein Stichprobenparameter nicht zu gross ist,wahrend einem zu kleinen Stichprobenparameter keine Bedeutung beigemessen wird.Hier mussen also grosse positive (resp. negative) Abweichungen zu einer Ablehnung derNullhypothese fuhren.

Die Nullhypothese und die Alternativhypothese sind jeweils disjunkt.

Page 92: Wahrscheinlichkeitstheorie undStatistik (wst)web.fhnw.ch/.../Stochastik-Skript-Marcel-Steiner.pdf · Anwendungsgebiete der Statistik sind die Industrie und die Technik, die Physik,

84 Kapitel 8. Statistische Tests

Ob eine Hypothese mit einem zweiseitigen oder einseitigen Test zu prufen ist, hangt vompraktischen Problem ab und wird vor Testbeginn festgelegt. Sind keine Vorkenntnisse uberdie Richtung der moglichen Abweichungen vorhanden, so wird ein zweiseitiger Test verwendet.Ist von vornherein einer der Falle µ > µ0 oder µ < µ0 ausgeschlossen, so wird ein einseitigerTest zur Anwendung kommen.

Beispiel 8.2.1 (z-Test). Letztes Jahr waren 75% der SBB-Fahrgaste Inhaber von Halbtax-abonnementen. Bei einer kurzlich durchgefuhrten Fahrgastbefragung gaben 270 von 350 Be-fragten an, dass sie ein Halbtaxabonnement besitzen. Hat sich der Anteil der Besitzer vonHalbtaxabonnementen wesentlich verandert? Das Signifikanzniveau sei α = 10%.Um diese Frage zu beantworten, fuhren wir einen statistischen Test nach obigem Prinzipdurch: Es sei p = 0.75 der relative Anteil von Halbtaxabonnementbesitzer im letzten Jahr.Nun formulieren wir die Null- und Alternativhypothesen fur einen statistischen Test:

H0 : p = 0.75, d.h., die Anzahl Halbtaxabonnementbesitzer ist gleich wie letztes Jahr.

H1 : p 6= 0.75, d.h., die Anzahl Halbtaxabonnementbesitzer hat sich verandert.

Es handelt sich hier um einen so genannten zweiseitigen Test, da hier die Alternativhy-pothese nur Werte p 6= 0.75 zulasst. Weiter beschreibe die Zufallsgrosse X die Anzahl derHalbtaxabonnementbesitzer unter den n = 350 befragten Fahrgasten. Die Zufallsgrosse X

ist binomialverteilt. Wir berechnen den Erwartungswert und die Varianz unter Annahme derNullhypothese H0

E(X) = np = 262.5 und Var(X) = np(1− p) = 65.625.

Es stellt sich also die Frage, ob sich die Zahl der gezahlten 270 Halbtaxabonnementbesitzersignifikant vom Erwartungswert E(X) = 262.5 unterscheidet.Weil np(1 − p) = 65.625 > 9 konnen wir die Binomialverteilung mit einer Normalverteilungmit den Parametern µ = 262.5 und σ2 = 65.625 approximieren (vgl. Grenzwertsatz von deMoivre und Laplace, Gleichung (7.3.a)).Durch eine Massstabsanderung auf der Koordinatenachse und einer Nullpunktverschiebungauf der x-Achse

z =x− µ

σ

kann von der Normalverteilung mit den Parametern µ und σ2 zur standardisierte Normalver-teilung mit den Parametern µ = 0 und σ2 = 1 ubergegangen werden.Da es sich hier um einen zweiseitigen Test handelt, verteilen wir α = 0.10 = 0.05 + 0.05gleichmassig auf beiden Seiten der Standardnormalverteilung (vgl. Abbildung 8.2.iv). Ausder Beziehung

P (Z ≤ z0.05) = Φ (z0.05, 0, 1) = 0.05

bestimmen wir mit Tafel T.2 oder einem Computerprogramm (z.B. Excel) die untere kritischeGrosse z0.05 = −1.645, d.h. das 0.05-Quantil z0.05 der Standardnormalverteilung; und aus derBeziehung

P (z0.95 ≤ Z) = 1− Φ (z0.95, 0, 1) = 0.05

bestimmen wir die obere kritische Grosse z0.95 = 1.645, d.h. das 0.95-Quantil z0.95 der Stan-dardnormalverteilung. Da die Standardnormalverteilung symmetrisch bezuglich µ = 0 ist,folgt z0.95 = −z0.05.

Page 93: Wahrscheinlichkeitstheorie undStatistik (wst)web.fhnw.ch/.../Stochastik-Skript-Marcel-Steiner.pdf · Anwendungsgebiete der Statistik sind die Industrie und die Technik, die Physik,

8.3. Mogliche Fehler bei statistischen Tests 85

0 z0.95

0.050.90

zz0.05

0.05

Annahmebereich AblehnungsbereichAblehnungsbereich

z

Abbildung 8.2.iv: Bestimmung des 0.05-Quantils z0.05 = −1.645 und des 0.95-Quantils z0.95 =1.645 bei der Standardnormalverteilung bei einer zweiseitigen Fragestellung. Die Testgrossez = 0.926 liegt im Annahmebereich, damit wird die Nullhypothese angenommen.

Die Berechnung der Testgrosse aus den vorliegenden Angaben ergibt

z =270− µ

σ=

270− 262.5√65.625

= 0.926.

Es gilt nun z0.05 = −1.645 < z = 0.926 < z0.95 = 1.645, d.h., die Testgrosse z liegt imAnnahmebereich und somit lautet der statistische Schluss:Wir nehmen die Nullhypothese auf dem Niveau 10% an. Der Anteil der Besitzer von Halbtax-abonnementen hat sich nicht signifikant verandert.

Hierbei handelt es sich um einen so genannten z-Test.

8.3 Mogliche Fehler bei statistischen Tests

Am Ende eines statistischen Tests fallen wir immer einen statistischen Schluss, der dabeizugunsten der Nullhypothese H0 oder der Alternativhypothese H1 ausfallt. In beiden Fallenwerden gewisse Ruckschlusse von einer Stichprobe auf die entsprechende Grundgesamtheit ge-zogen. Dabei mussen wir unbedingt bedenken, dass es absolut sichere Schlusse grundsatzlichnicht gibt. Bei einer Testentscheidung besteht immer eine bestimmteWahrscheinlichkeit dafur,dass die getroffene Entscheidung falsch ist. Dabei werden zwei Arten von Fehlern unterschie-den:

Definition 8.3.1.

a. Ein Fehler 1. Art liegt vor, wenn eine richtige Nullhypothese H0 abgelehnt wird.

b. Ein Fehler 2. Art liegt vor, wenn eine falsche Nullhypothese H0 nicht abgelehnt wird.

Die Wahrscheinlichkeit fur einen Fehler 1. Art entspricht der Irrtumswahrscheinlichkeit, furwelche wir das Signifikanzniveau α vorgegeben haben. Die Wahrscheinlichkeit fur einen Fehler2. Art wird mit β bezeichnet.Wir erlautern nun die moglichen Falle an Hand eines einseitigen Tests, bei dem die Nullhypo-these H0 : µ = µ0 gegen die Alternativhypothese H1 : µ > µ0 getestet wird. Dabei bezeichnetµ den zu testenden unbekannten Parameter der Verteilung.

Page 94: Wahrscheinlichkeitstheorie undStatistik (wst)web.fhnw.ch/.../Stochastik-Skript-Marcel-Steiner.pdf · Anwendungsgebiete der Statistik sind die Industrie und die Technik, die Physik,

86 Kapitel 8. Statistische Tests

µ0 z1−α

α

1 − α

z

Annahmebereich Ablehnungsbereich

z

Abbildung 8.3.i: Es sei H0 richtig: Daz < z1−α wird die Nullhypothese ange-nommen. Dies ist die richtige Entschei-dung, welche mit einer Wahrscheinlichkeitvon 1− α getroffen wird.

µ0 z1−α

α

1 − α

z

Annahmebereich Ablehnungsbereich

z

Abbildung 8.3.ii: Es sei H0 richtig: Daz ≥ z1−α wird die Nullhypothese abge-lehnt. Dies ist die falsche Entscheidung(Fehler 1. Art), welche mit einer Wahr-scheinlichkeit von α getroffen wird.

µ0 µ1z1−α

α

1 − α

β

1 − β

z

Annahmebereich Ablehnungsbereich

z

Abbildung 8.3.iii: Es sei H0 falsch, H1

richtig, d.h., die gestrichelte Dichte ist dierichtige: Da z < z1−α wird die Nullhy-pothese angenommen. Dies ist die falscheEntscheidung (Fehler 2. Art), welchemit einer Wahrscheinlichkeit von β getrof-fen wird.

µ0 µ1z1−α

α

1 − α

β

1 − β

z

Annahmebereich Ablehnungsbereich

z

Abbildung 8.3.iv: Es sei H0 falsch, H1

richtig, d.h., die gestrichelte Dichte ist dierichtige: Da z ≥ z1−α wird die Nullhypo-these abgelehnt. Dies ist die richtige Ent-scheidung, welche mit einer Wahrschein-lichkeit (so genannte Trennscharfe) von1− β getroffen wird.

AlsTrennscharfe oderMacht eines Tests bezeichnen wir die Wahrscheinlichkeit 1−β mit derdie Nullhypothese abgelehnt wird, wenn sie tatsachlich nicht stimmt (vgl. Abbildung 8.3.iv).

Trennscharfe = P (Entscheidung H0 nicht anzunehmen |H1 sei richtig) = 1− β

In der Praxis sind wir bestrebt, die Fehler 1. und 2. Art (d.h. gleichzeitig α und β) moglichstklein zu halten. Dazu betrachten wir die Abbildungen 8.3.iv und 8.3.iii und stellen fest, dasseine Verkleinerung von α (Verschiebung der kritischen Grosse z1−α nach rechts) automatischeine Vergrosserung von β nach sich zieht und umgekehrt. Entscheiden wir uns im konkretenFall fur ein kleines α und damit fur ein kleines Risiko eine an sich richtige Nullhypotheseabzulehnen, dann nehmen wir gleichzeitig ein deutlich erhohtes Risiko fur einen Fehler 2. Artin Kauf. Wir mussen also von Fall zu Fall entscheiden, welcher der beiden Fehler letztendlichdie grosseren Konsequenzen hat. Soll gleichwohl das Risiko fur einen Fehler 2. Art, d.h. β, ver-ringert werden, ohne gleichzeitig die Wahrscheinlichkeit α fur einen Fehler 1. Art vergrossern

Page 95: Wahrscheinlichkeitstheorie undStatistik (wst)web.fhnw.ch/.../Stochastik-Skript-Marcel-Steiner.pdf · Anwendungsgebiete der Statistik sind die Industrie und die Technik, die Physik,

8.3. Mogliche Fehler bei statistischen Tests 87

zu mussen, so bleibt uns nur die Vergrosserung des Stichprobenumfangs3 (Verbesserung derTrennscharfe des Tests).

In Abbildung 8.3.iii sehen wir zusatzlich, dass die Wahrscheinlichkeit β einen Fehler 2. Artzu begehen, wesentlich von der Alternativhypothese H1, d.h. Lage von µ1, abhangt. Die-sen funktionalen Zusammenhang zwischen β und µ1 wird als Operationscharakteristikbezeichnet.

Aufgaben

Formulieren Sie jeweils die Null- und Alternativhypothese und den problemorientierten sta-tistischen Schluss in Worten.

Aufgabe 8.3.1. Wir wurfeln mit einem Wurfel. Bei 20 Wurfen erhalten wir 9 Sechsen. Istder Wurfel gezinkt, d.h., werden bevorzugt Sechsen gewurfelt? Das Signifikanzniveau ist 5%.

Aufgabe 8.3.2. In 10000 Wurfen zeigte eine Munze 5150 mal Zahl. Mit welcher Wahrschein-lichkeit konnen wir behaupten, dass sie unsymmetrisch ist, d.h. bevorzugt Zahl geworfen wird?Das Signifikanzniveau ist 5%.

Aufgabe 8.3.3. Bei einer Umfrage vor einer Wahl sagten 285 der 2000 befragten Personen,sie wurden nicht zur Wahl gehen. Nachdem in der Zwischenzeit ein medienintensiver Wahl-kampf stattfand, betrug die tatsachliche Wahlbeteiligung 88.5%. Kann daraus mit 99%-igerSicherheit geschlossen werden, dass in der Zwischenzeit Personen, die ursprunglich nicht zurWahl gehen wollten, umgestimmt wurden?

Aufgabe 8.3.4. Unter 3000 in einer Klinik neugeborenen Kindern befanden sich 1578 Kna-ben. Testen Sie mit einer Irrtumswahrscheinlichkeit α = 0.01 die Hypothesen

H0: P (Knabengeburt) = 0.5

H1: P (Knabengeburt) 6= 0.5

Formulieren Sie den entsprechenden statistischen Schluss.

Aufgabe 8.3.5. Eine Multiple-Choice-Prufung bestehe aus 100 Einzelfragen, wobei bei jederFrage in zufalliger Reihenfolge 4 Antworten angegeben sind, wovon genau eine richtig ist. DerPrufling darf jeweils nur eine Antwort ankreuzen. Wieviel richtig angekreuzte Antwortenmussen zum Bestehen der Prufung mindestens verlangt werden, damit die Prufung durch(zufalliges Ankreuzen) hochstens mit Wahrscheinlichkeit

a. 0.05

b. 0.01

c. 0.001

d. 0.0001

bestanden werden kann?

3Dabei werden die Verteilungen schlanker und somit α und β gleichzeitig kleiner.

Page 96: Wahrscheinlichkeitstheorie undStatistik (wst)web.fhnw.ch/.../Stochastik-Skript-Marcel-Steiner.pdf · Anwendungsgebiete der Statistik sind die Industrie und die Technik, die Physik,

88 Kapitel 8. Statistische Tests

Losungen

Losung 8.3.1. Eine Approximation mit der Normalverteilung ist wegen np(1−p) = 2.778 ≤ 9nicht erlaubt. Wir berechnen mit der Binomialverteilung exakt die IrrtumswahrscheinlichkeitP (9 ≤ X) = 1− P (X ≤ 8) = 0.00284.

Losung 8.3.2. Die Wahrscheinlichkeit einer so grossen oder grosseren Abweichung nach obenist P (5150 ≤ X) = 0.0013. Also ist die Munze unsymmetrisch.

Losung 8.3.3. Es hatten hochstens x1−α = 264 Personen nicht zur Wahl gehen durfen, damitdie Nullhypothese angenommen werden konnte.

Losung 8.3.4. Der Annahmebereich ist [1430, 1570], also H0 ablehnen.

Losung 8.3.5. Die Ablehnungsgrenzen sind:

a. n1−0.05 = 33

b. n1−0.01 = 36

c. n1−0.001 = 39

d. n1−0.0001 = 42

Page 97: Wahrscheinlichkeitstheorie undStatistik (wst)web.fhnw.ch/.../Stochastik-Skript-Marcel-Steiner.pdf · Anwendungsgebiete der Statistik sind die Industrie und die Technik, die Physik,

Kapitel 9

Prufen von Erwartungswerten(Parametertests)

9.1 Problemstellung der technischen Statistik

Die meisten Fragestellungen der angewandten Statistik fuhren auf den Vergleich von zwei odermehreren normalverteilten Grundgesamtheiten. Da eine Normalverteilung durch die beidenParameter µ und σ2 vollstandig definiert ist, bedeutet dies, dass wir herausfinden mussen, obdie entsprechenden Parameter bei zwei normalverteilten Grundgesamtheiten ubereinstimmenoder nicht.

Beispiel 9.1.1. Gegeben seien zwei Maschinen des gleichen Typs. Beide produzieren Pro-dukte von einer bestimmten gleichen Art. Produzieren sie Produkte aus der gleichen Grund-gesamtheit oder nicht? Das heisst, arbeiten sie gleich genau, ist also die Varianz identischund sind sie auf den gleichen Sollwert eingestellt, d.h., ist der Erwartungswert der beidenGrundgesamtheiten gleich?

Fragestellung der Statistik

• Gegeben: Zwei normalverteilte Grundgesamtheiten mit den Parametern µ1, σ21 und

µ2, σ22 .

• Frage: Sind die beiden Grundgesamtheiten identisch, d.h., gilt µ1 = µ2 und σ21 = σ2

2?

• Vorgehen der Statistik zur Beantwortung dieser Frage: Jeder Grundgesamtheit ent-nehmen wir je eine Stichprobe S1 und S2, dann berechnen wir aus diesen Stichprobendie geschatzten Parameter x1, s

22 und s21, x2, vergleichen sie in einem statistischen Test

und schliessen die Grundgesamtheiten sind gleich oder nicht.

In einigen Fallen kann bereits vorausgesetzt werden, dass zum Beispiel µ1 = µ2 ist, so dassein Test auf σ2

1 = σ22 durchzufuhren ist. Oder die Varianzen sind gleich, und die Gleichheit

der Erwartungswerte wird getestet. Je nach Problemstellung ist ein bestimmter Test durch-zufuren.In den bisherigen Betrachtungen in Kapitel 8 war zur Prufung der Hypothese H0 : µ = µ0

fur die Berechnung der Testgrosse die Kenntnis der Varianz σ2 in der Grundgesamtheit erfor-derlich. Dazu sind umfangreiche Voruntersuchungen notwendig, oder wir ersetzen fur einengrossen Stichprobenumfang σ2 durch die geschatzte Stichprobenvarianz s2. Welche Testgrosse

89

Page 98: Wahrscheinlichkeitstheorie undStatistik (wst)web.fhnw.ch/.../Stochastik-Skript-Marcel-Steiner.pdf · Anwendungsgebiete der Statistik sind die Industrie und die Technik, die Physik,

90 Kapitel 9. Prufen von Erwartungswerten (Parametertests)

konnen wir nun bei kleinen Stichprobenumfangen zur Prufung der Hypothese H0 : µ = µ0

heranziehen, wenn σ2 nicht als Erfahrungswert vorliegt?

Beim Prufen von Erwartungswerten gibt es mehrere verschieden Problemstellungen. Diesewollen wir nun untersuchen.

9.2 Einstichproben-t-Test, Student-t-Test

Beim Einstichproben-t-Test oder Student-t-Test ist der Erwartungswert µ der Grundge-samtheit G bekannt und es sind folgende Voraussetzungen zu beachten:

1. Die normalverteilte Grundgesamtheit G hat den bekannten Erwartungswert µ und dieunbekannte Varianz σ2.

2. Es sind zufallig N Stichprobenwerte x1, . . . , xN aus einer normalverteilten Grundge-samtheit gewahlt.

Wir wollen nun wissen, ob die gewahlte Stichprobe der N Werte x1, . . . , xN aus der Grund-gesamtheit G mit dem Erwartungswert µ stammt. Dazu berechnen wir den geschatztenMittelwert

x =1

N

N∑

i=1

xi

und vergleichen ihn mit dem bekannten Erwartungswert µ der Grundgesamtheit G, indemwir folgende alternative Hypothese aufstellen.

H0 : µ = x, d.h., Stichprobe stammt aus der Grundgesamtheit G mit Erwartungswert µ.

H1 : µ 6= x, d.h., Stichprobe stammt aus einer anderen Grundgesamtheit.

Zur Beantwortung dieser Fragestellung machen wir nun folgende gedankliche Konstruktion,die typisch ist fur die statistische Denkweise: Wir betrachten die Gesamtheit aller zufalligenStichproben mit N Werten x1, . . . , xN aus einer normalen Grundgesamtheit mit Erwar-tungswert µ und unbekannter Varianz. Zu jeder Stichprobe berechnen wir aus den Wertenx1, . . . , xN den geschatzten Mittelwert x und die geschatzte Varianz

s2 =1

N − 1

N∑

i=1

(xi − x)2

und daraus die Testgrosse

t =x− µ

s

√N. (9.2.a)

Der Wert der Testgrosse t wird umso grosser,

• je grosser die Abweichung des geschatzten Mittelwerts x vom Erwartungswert µ ist,

• je grosser der Stichprobenumfang N gewahlt ist und

• je kleiner die geschatzte Varianz s2 ist, d.h., je weniger die Stichprobenwerte um denMittelwert streuen.

Page 99: Wahrscheinlichkeitstheorie undStatistik (wst)web.fhnw.ch/.../Stochastik-Skript-Marcel-Steiner.pdf · Anwendungsgebiete der Statistik sind die Industrie und die Technik, die Physik,

9.2. Einstichproben-t-Test, Student-t-Test 91

Fur jede Stichprobe erhalten wir nun einen anderen Wert fur t und demzufolge wieder eineWahrscheinlichkeitsverteilung. Ist die Zufallsgrosse X normalverteilt, so gehorcht die neueZufallsgrosse

T =X − µ

s

√N

einer sogenannten Student-t-Verteilung mit n = N − 1 Freiheitsgraden, die nicht mehrder Normalverteilung entspricht1.

Der Ablehnungsbereich fur die Nullhypothese H0 bei einem gegebenen Signifikanzniveau α

ist fur die zweiseitige Fragestellung durch die kritischen Grossen tn,1−α2und tn,α

2= −tn,1−α

2

gegeben. Die kritischen Grossen lassen sich fur die zweiseitige Fragestellung aus der Beziehung

P(|T | ≥ tn,1−α

2

)= α

mit Hilfe von Tafel T.3 oder einem Computerprogramm (z.B. Excel) ermitteln. Jetzt ziehen

tn,1−α

2

α2

1 − α

tn, α

2

= −tn,1−α

2

α2

Annahmebereich AblehnungsbereichAblehnungsbereich

t

fn(t)

Abbildung 9.2.i: Kritische Grossen tn,α2und tn,1−α

2beim Student-t-Test, mit tn,α

2= −tn,1−α

2.

wir den statistischen Schluss (hier fur die zweiseitige Fragestellung):

• Ist die Testgrosse |t| < tn,1−α2, dann wird die Nullhypothese H0 angenommen, d.h.,

Abweichungen vom idealen Wert t = 0 sind zufalliger Natur. Die Stichprobe stammtsomit mit einer Irrtumswahrscheinlichkeit von 1−α aus der Grundgesamtheit mit demErwartungswert µ.

• Ist die Testgrosse |t| ≥ tn,1−α2, dann wird die Nullhypothese H0 auf dem Signifikanzni-

veau α abgelehnt. Die Stichprobe stammt demnach aus einer anderen Grundgesamtheit.

Der Student-t-Test ist gegenuber Abweichungen von der Voraussetzung (1), dass die Grund-gesamtheit G normalverteilt sein muss, ziemlich unempfindlich. Der Student-t-Test ist ein sogenannt robuster Test.

Beispiel 9.2.1. Es sei die folgende Stichprobe mit zehn Werten gegeben:

5 -5 7 4 15 -7 5 10 18 16

1Dass die SummeX+Y zweier gleich verteilter Zufallsvariablen X und Y nicht mehr der gleichen Verteilungwie die der Summanden gehorchen muss, sehen wir an folgendem Beispiel: Ein einzelner Wurfel hat fur jedeAugenzahl die gleiche Wahrscheinlichkeit. Betrachten wir nun die Summe der Augenzahlen zweier Wurfel, sostellen wir fest, dass die Summe 7 viel haufiger ist als 2 oder 12. Dies wird zum Beispiel beim beruhmtenGesellschaftsspiel Die Siedler von Catan von Klaus Teuber ausgenutzt.

Page 100: Wahrscheinlichkeitstheorie undStatistik (wst)web.fhnw.ch/.../Stochastik-Skript-Marcel-Steiner.pdf · Anwendungsgebiete der Statistik sind die Industrie und die Technik, die Physik,

92 Kapitel 9. Prufen von Erwartungswerten (Parametertests)

Uns interessiert nun, ob die Stichprobe aus einer Grundgesamtheit mit Erwartungswert µ = 0und unbekannter Varianz stammt oder nicht. Es handelt sich dabei um einen zweiseitigenStudent-t-Test, da wir nur wissen wollen, ob der Mittelwert x gleich oder ungleich von µ = 0ist. Dazu wollen wir fur die zweiseitige Fragestellung die folgende Nullhypothese gegen dieAlternative testen:

H0 : µ = 0, d.h., Stichprobe stammt aus Grundgesamtheit mit Erwartungswert µ = 0.

H1 : µ 6= 0, d.h., Stichprobe stammt aus anderer Grundgesamtheit.

Wir identifizieren den Stichprobenumfang mit N = 10 und berechnen x = 6.80 und s2 =70.18. Die Nullhypothese besagt in diesem Fall, dass das Mittel x = 6.80 rein zufallig, aus-wahlbedingt, vom erwarteten theoretischen Wert µ = 0 abweicht. Da hier der Erwartungs-wert µ = 0 der Grundgesamtheit bekannt und die Varianz unbekannt ist, benutzen wir einenStudent-t-Test mit n = N − 1 = 9 Freiheitsgraden, um obige Hypothese zu untersuchen. Wirberechnen die Testgrosse

t =x− µ

s

√N =

6.80 − 0√70.18

√10 = 2.567. (9.2.b)

Zum Signifikanzniveau α = 0.05 bestimmen wir nun die kritische Grosse t9,1−0.025 = 2.262 furdie zweiseitige Fragestellung.Nun fuhren wir den statistischen Schluss durch: Es gilt |t| = 2.567 ≥ t9,1−0.025 = 2.262,also wird die Nullhypothese H0 abgelehnt. Das Mittel x = 6.80 weicht somit wesentlich vomtheoretischen Wert µ = 0 ab.

Bemerkungen:

• Falls das Signifikanzniveau kleiner gewahlt wurde, z.B. α = 0.01, dann ergabe sich einkritischer Wert von t9,1−0.005 = 3.250 und es ergabe keine signifikante Abweichung mehr.Es ist deshalb wichtig, dass immer bei einem statistischen Schluss das Signifikanzniveauangegeben wird, damit alle wissen, was von der Aussage zu halten ist.

• Auch wenn die Nullhypothese abgelehnt werden kann, besteht, wie wir bereits in Kapi-tel 8.3 gesehen haben, eine gewisse Wahrscheinlichkeit, dass wir einen falschen Schlussziehen. Die Wahrscheinlichkeit, dass ein berechneter Wert von t unter der Voraussetzungder Nullhypothese so extrem wird, ist bekanntlich nicht null, sondern nur klein. BeimAblehnen der Nullhypothese mussen wir also auch eine Irrtumswahrscheinlichkeit an-geben. Sie entspricht der Wahrscheinlichkeit, die besteht, dass der Wert der Testgrosset rein zufallig so extrem herauskommt. Sie ist also kleiner als das gewahlte Signifikanz-niveau.

• Kann hingegen die Nullhypothese nicht abgelehnt werden, so besteht trotzdem einegewisse Wahrscheinlichkeit, dass eine signifikante Abweichung vorliegt. Wir sprechenvom Risiko fur einen Fehler 2. Art.

Die Student-t-Verteilung

Die von W. S. Gosset (1876-1937) (vgl. Abbildung 9.2.iii) gefundene Student-t-Verteilungmit n = N − 1 Freiheitsgraden hat die Wahrscheinlichkeitsdichte

fn(t) = cn

(1 +

t2

n

)−n+12

fur t ∈ R,

Page 101: Wahrscheinlichkeitstheorie undStatistik (wst)web.fhnw.ch/.../Stochastik-Skript-Marcel-Steiner.pdf · Anwendungsgebiete der Statistik sind die Industrie und die Technik, die Physik,

9.2. Einstichproben-t-Test, Student-t-Test 93

wobei cn =Γ(n+1

2)√

nπ Γ(n2)eine nur von der Anzahl Freiheitsgraden n abhangige Konstante2 ist. Der

Erwartungswert und die Varianz oder Streuung einer mit n Freiheitsgraden Student-t-verteilten Zufallsgrosse T betragt

E(T ) = 0 fur n > 1 und Var(T ) =n

n− 2fur n > 2.

Wie bei jeder Verteilung gilt auch hier∫∞−∞ fn(t) dt = 1.

0

0.1

0.2

0.3

0.4

–6 –4 –2 2 4 6

x

Abbildung 9.2.ii: Die Student-t-Verteilung (schwarze Kurven) fur verschiedene Freiheitsgraden. Die Kurven ahneln denen der standardisierten Normalverteilung (graue Kurve), stimmenaber erst fur grosse n einigermassen uberein.

Die Dichte der Student-t-Verteilung ist symmetrisch bezuglich des Nullpunktes t = 0. Siehat einen um so flacheren Verlauf, je kleiner n ist, und strebt fur n → ∞ gegen die Dichteder standardisierten Normalverteilung N (0, 1). Das q-Quantil der Student-t-Verteilung mitn Freiheitsgraden wird mit tn,q bezeichnet und ist vertafelt (vgl. Tafel T.3). Das q-Quantiltn,q kann aus der Beziehung

P (T ≤ tn,q) = q

bestimmt werden. Da die Dichte symmetrisch ist, gilt fur das (1− q)-Quantil tn,1−q = −tn,q.

Aufgaben

Aufgabe 9.2.1. Durch Messung wurden die Langen von funf Wellen bestimmt. Es wurden8, 9, 11, 10, 10 Einheiten gemessen. Weicht der Mittelwert signifikant von µ = 10E ab? DasSignifikanzniveau ist 1%.

Aufgabe 9.2.2. Bei einem Spannvorgang wurde bisher mit einem Vorgabewert von 135sgerechnet. Eine Zeitaufnahme lieferte bei N = 32 aufgenommenen Zeiten fur diesen Teilvor-gang einen mittleren Zeitbedarf in der Hohe von x = 128s bei einer Standardabweichung von

2Es ist Γ(x) =∫

0tx−1e−t dt die Gammafunktion, die die Fakultat auf reelle Zahlen verallgemeinert. Es

gilt Γ(1) = 1 und Γ(n) = (n− 1)! fur alle n ∈ N.

Page 102: Wahrscheinlichkeitstheorie undStatistik (wst)web.fhnw.ch/.../Stochastik-Skript-Marcel-Steiner.pdf · Anwendungsgebiete der Statistik sind die Industrie und die Technik, die Physik,

94 Kapitel 9. Prufen von Erwartungswerten (Parametertests)

Abbildung 9.2.iii: William Sealey Gosset (1876-1937), der bei Guinness als Bierbrauerbeschaftigt war, veroffentlichte im Jahr 1908 die t-Verteilung zum Mittelwertsvergleich. Da erdamit den Malzgehalt verschiedener Getreidesorten untersuchte, war seine Firma von einerVeroffentlichung nicht begeistert. Daher publizierte er seinen t-Test (t vom engl. test) unterdem Pseudonym “Student”, was dem Test den Namen “Student-t-Test” eingebracht hat.

s = 4.7s. Kann aus dem Unterschied zwischen 135s und 128s darauf geschlossen werden, dassder wahre jedoch unbekannte mittlere Zeitbedarf fur diesen Teilvorgang generell nicht bei135s liegt? Das Signifikanzniveau ist 1%.

Aufgabe 9.2.3. Auf vier Ackern von je 40 Aren konnte der Ertrag von Kartoffeln durchneuartige Behandlung um 0.55, 0.30, 1.52, 0.68 Tonnen gesteigert werden. Ist diese Behand-lungsmethode wirksamer als fruhere? Das Signifikanzniveau ist 1%.

Aufgabe 9.2.4. Es werden die Zugriffszeiten bei einem bestimmten Produktionsprozess un-tersucht. Folgende Stichprobe [in Sekunden] wurde ermittelt. Sind diese Zeiten wirklich von0.4 Sekunden verschieden? Das Signifikanzniveau ist 1%.

0.23 0.23 0.23 0.30 0.32 0.32 0.34 0.34 0.340.43 0.43 0.43 0.43 0.45 0.45 0.45 0.45 0.450.54 0.54 0.54 0.54 0.54 0.54 0.56 0.56 0.560.62 0.65 0.65 0.65 0.67 0.67 0.68 0.76 0.76

9.2.1 Vertrauensintervall fur den Erwartungswert

Aus einer gegebenen Stichprobe konnen wir gewisse Parameter wie Mittelwert oder Vari-anz schatzen (berechnen). Hierbei fehlen aber noch Genauigkeitsangaben zu den berechnetenWerten, und vielleicht mochten wir auch wissen, wie die Genauigkeit vom Stichprobenumfangabhangt. Dazu dienen die Intervallschatzungen, das sind aus der Stichprobe berechnete In-tervalle, in denen der wahre, aber unbekannt Wert mit grosser Wahrscheinlichkeit zu erwartenist. Solche Intervalle heissen Vertrauens- oder Konfidenzintervalle.

Haben wir eine Stichprobe x1, . . . , xN vom Umfang N aus einer normalverteilten Grund-gesamtheit genommen, so interessieren wir uns fur ein Vertrauensintervall des unbekanntenErwartungswertes µ der normalverteilten Grundgesamtheit. Dazu schatzen (berechnen) wirvorerst den Mittelwert

x =1

N

N∑

i=1

xi

Page 103: Wahrscheinlichkeitstheorie undStatistik (wst)web.fhnw.ch/.../Stochastik-Skript-Marcel-Steiner.pdf · Anwendungsgebiete der Statistik sind die Industrie und die Technik, die Physik,

9.2. Einstichproben-t-Test, Student-t-Test 95

und die Varianz

s2 =1

N − 1

N∑

i=1

(xi − x)2

aus der Stichprobe. Nun konnen wir Vertrauens- oder Konfidenzgrenzen fur den unbe-kannten Wert µ angeben, innerhalb welchen der wahre Erwartungswert mit einer gewissenvorgegebenen Wahrscheinlichkeit γ liegt. Aus der vorgegebenen Vertrauenswahrschein-lichkeit γ = 1 − α bestimmen wir die kritische Grosse tn,1−α

2der Student-t-Verteilung mit

n = N − 1 Freiheitsgraden. Beachten Sie, dass wir den Flacheninhalt α auf beiden Seiten un-ter der Student-t-Verteilung gleichmassig verteilen, d.h. zweiseitige Fragestellung. Wir suchenalso alle moglichen µ, so dass

P(|T | < tn,1−α

2

)= P

(∣∣∣ X−µs

√N∣∣∣ < tn,1−α

2

)= 1− α

gilt. Nun konnen wir die Ungleichung∣∣∣ x−µ

s

√N∣∣∣ < tn,1−α

2nach µ umformen und erhalten das

Vertrauensintervall fur den unbekannten Erwartungswert µ durch

x−tn,1−α

2√N

s ≤ µ ≤ x+tn,1−α

2√N

s.

Je grosser der Stichprobenumfang N ist, desto kleiner wird das Vertrauensintervall. Wollenwir also genaue Aussagen uber den unbekannten Mittelwert machen, so sind wir gezwungenden Stichprobenumfang N moglichst gross zu wahlen.Das Vertrauensintervall kann wie folgt interpretiert werden: von 100 aus Stichproben derselbenGrundgesamtheit mit dem unbekannten Erwartungswert µ berechneten Vertrauensintervallenuberdecken im Mittel γ · 100 = (1− α) · 100 den wahren Erwartungswert µ.

Beispiel 9.2.2. Es sei eine Stichprobe vom Umfang N = 10 mit geschatztem Mittelwertx = 5 und Standardabweichung s = 0.2 gegeben. In welchem Intervall liegt nun der wahreaber unbekannte Erwartungswert µ der normalverteilten Grundgesamtheit? Dazu berechnenwir zur Vertrauenswahrscheinlichkeit γ = 0.95 die kritische Grosse t9,1−0.025 = 2.262 derStudent-t-Verteilung. Damit ergibt sich das gesuchte Vertrauensintervall

5− 2.262√10

0.2 ≤ µ ≤ 5 +2.262√

100.2,

also 4.86 ≤ µ ≤ 5.14 mit 95% Wahrscheinlichkeit.

Aufgaben

Aufgabe 9.2.5. An Hand einer Stichprobe von 10 auf einem Drehautomaten bearbeitetenWellen soll ein Vertrauensintervall zur Vertrauenswahrscheinlichkeit 0.99 fur den Erwartungs-wert µ der Grundgesamtheit der Abweichungen des Wellendurchmessers von der Mitte desToleranzfeldes bestimmt werden. Folgende Abweichungen [in Mikrometer] der ist-Masse vonder Mitte des Toleranzfeldes sind festgestellt worden:

2 1 -2 3 2 4 -2 5 3 4

a. Verwenden Sie die Normalverteilung!

Page 104: Wahrscheinlichkeitstheorie undStatistik (wst)web.fhnw.ch/.../Stochastik-Skript-Marcel-Steiner.pdf · Anwendungsgebiete der Statistik sind die Industrie und die Technik, die Physik,

96 Kapitel 9. Prufen von Erwartungswerten (Parametertests)

b. Verwenden Sie die Student-t-Verteilung!

Aufgabe 9.2.6. Gegeben sei wieder die Stichprobe aus Aufgabe 9.2.4 der Zugriffszeiten. Inwelchem Vertrauensintervall zur Vertrauenswahrscheinlichkeit 0.99 liegt der wirkliche Wertdes Mittels der Zugriffszeiten?

9.2.2 Ungefahr erforderlicher Stichprobenumfang

Wirtschaftliche und rationelle Arbeitsweise erfordern die Angabe des Arbeitsaufwandes, umbestimmte Genauigkeiten bei Mess- und Analysenergebnissen zu erziehlen. So ist es etwawichtig, abzuschatzen, wie gross der Stichprobenumfang bei einem statistischen Test ungefahrsein muss, um eine bestimmte Zuverlassigkeit der Aussage zu erhalten.

Mit Hilfe der Testgrosse t (vgl. Gleichung (9.2.a)) lasst sich eine solche ungefahre Abschatzungmachen. Formen wir die Gleichung (9.2.a) nach N um, so erhalten wir

N =t2s2

(x− µ)2. (9.2.c)

Wir geben uns einen bestimmten Toleranzbereich ∆µ = |x−µ| vor. Ist zusatzlich die Varianzs2 aus Voruntersuchungen etwa in Form einer oberen Schranke bekannt, so konnen wir fur denStichprobenumfang N einen ungefahren Wert abschatzen, indem wir einen Durchschnittswertfur t = tn,1−α

2≈ 2 bei einer Vertrauenswahrscheinlichkeit γ = 1 − α einsetzen. Wir erhalten

damit einen ungefahren Stichprobenumfang

N ≈ 4s2

∆µ2.

Es sei hier ausdrucklich gesagt, dass diese Abschatzung nur einen ungefahren3 Wert fur denStichprobenumfang liefert.

Aufgabe

Aufgabe 9.2.7. Der Kupfergehalt einer Partie Schwefelkies-Abbrande (Fe3O3 Huttruckstan-de) soll auf ∆µ = ±0.05% Cu genau bestimmt werden. Zur Bestimmung von s wurden 24Proben genommen und getrennt analysiert. Es ergaben sich x = 2.034% Cu und s = 0.271%Cu. Wie viele Proben sind etwa zu nehmen?

9.3 Vergleich zweier Mittelwerte unverbundener Stichproben

Im Folgenden wollen wir den Vergleich zweier Mittelwerte aus normalverteilten Grundge-samtheiten anstellen. Wir unterscheiden die beiden Falle, wenn die unbekannten Varianzender normalverteilten Grundgesamtheit gleich oder ungleich sind.

3Die rechte Seite der Gleichung (9.2.c) hangt via der kritischen Grosse tn,1−α

2auch noch von n = N − 1

ab. Somit liesse sich der Stichprobenumfang N nur iterativ bestimmen. Wir umgehen das Problem indem wireinen Durchschnittswert fur t einsetzten.

Page 105: Wahrscheinlichkeitstheorie undStatistik (wst)web.fhnw.ch/.../Stochastik-Skript-Marcel-Steiner.pdf · Anwendungsgebiete der Statistik sind die Industrie und die Technik, die Physik,

9.3. Vergleich zweier Mittelwerte unverbundener Stichproben 97

9.3.1 Zweistichproben-t-Test bei unbekannten aber gleichen Varianzen

Beispiel 9.3.1 (Parallelklassen). An einer Fachhochschule werden eine Klasse A von 25Studierenden und eine Parallelklasse B von 28 Studierenden vom gleichen Dozenten in Ma-thematik unterrichtet. Der Dozent gestaltet jeweils den Unterricht in beiden Klassen gleich.Demzufolge wurden die beiden Klassen gleichzeitig zur gleichen Klausur aufgeboten. Die er-reichten Notendurchschnitte waren xA = 3.9 und xB = 4.2 und die Standardabweichungenbetrugen je sA = sB = 1. Der Dozent stellt sich nun sofort die Frage, ob die B-Klassesignifikant besser als die A-Klasse sei. Was denken Sie?

Beim Zweistichproben-t-Test sind folgende Voraussetzungen zu beachten:

1. Die normalverteilten Grundgesamtheiten G1 und G2 haben die unbekannten Erwar-tungswerte µ1 und µ2 und die unbekannten aber gleichen4 Varianzen σ2

1 = σ22 = σ2,

so genannt homoskedastischer Fall. Der Wert von σ2 braucht jedoch nicht bekanntzu sein.

2. Es sind zufallig zwei Stichproben x1, . . . , xN1 und y1, . . . , yN2 aus den normalverteiltenGrundgesamtheiten G1 und G2 gewahlt.

Wir wollen nun wissen, ob sich die Mittelwerte x und y der gewahlten Stichproben signi-fikant voneinander unterscheiden um herauszufinden, ob die Stichproben aus der gleichenGrundgesamtheit stammen. Dazu formulieren wir die beiden alternativen Hypothesen

H0 : µ1 = µ2, d.h., Stichproben stammen aus der gleichen Grundgesamtheit.

H1 : µ1 6= µ2, d.h., Stichproben stammen aus unterschiedlichen Grundgesamtheiten.

Um diese Frage zu beantworten berechnen wir die geschatzten Mittelwerte

x =1

N1

N1∑

i=1

xi und y =1

N2

N2∑

i=1

yi

und die geschatzten Varianzen

s21 =1

N1 − 1

N1∑

i=1

(xi − x)2 und s22 =1

N2 − 1

N2∑

i=1

(yi − y)2

und daraus das gewogene Mittel der Varianzen

s2 =(N1 − 1)s21 + (N2 − 1)s22

N1 +N2 − 2.

Mit diesen Werten berechnen wir nun die Testgrosse

t =x− y

s

√N1N2

N1 +N2, (9.3.a)

welche unter den obigen Voraussetzungen und der Nullhypothese einer Student-t-Verteilungmit n = N1 + N2 − 2 Freiheitsgraden genugt. Damit konnen wir nun nach Vorgabe einesSignifikanzniveaus α die kritische Grosse tn,1−α

2fur die zweiseitige Fragestellung bestimmen.

Danach ziehen wir wieder den statistischen Schluss:4Die Gleichheit der Varianzen in den Grundgesamtheiten kann mit einem χ2-Test uberpruft werden. Ist

diese Gleichheit nicht erfullt, so haben wir ein so genanntes Behrens-Fisher-Problem (vgl. Kapitel 9.3.2).

Page 106: Wahrscheinlichkeitstheorie undStatistik (wst)web.fhnw.ch/.../Stochastik-Skript-Marcel-Steiner.pdf · Anwendungsgebiete der Statistik sind die Industrie und die Technik, die Physik,

98 Kapitel 9. Prufen von Erwartungswerten (Parametertests)

• Ist die Testgrosse |t| < tn,1−α2, dann wird die Nullhypothese H0 angenommen, d.h., die

Unterschiede zwischen x und y sind zufalliger Natur.

• Ist die Testgrosse |t| ≥ tn,1−α2, dann wird die Nullhypothese H0 auf dem Signifikanzni-

veau α abgelehnt.

Sind im Falle unabhangiger Stichproben ihre Umfange gleich, gilt also N1 = N2 = N , sovereinfacht sich die Testgrosse (9.3.a) zu

t =x− y√s21 + s22

√N.

Der Zweistichproben-t-Test ist auch dann anwendbar, wenn die Grundgesamtheiten nichtnormalverteilt, ihre Verteilungen aber nicht allzu unsymmetrisch sind.

Beispiel 9.3.2. Gegeben seien die Messreihen x = 1, 2, 3, 2, 1 und y = 2, 2, 4, 1 aus nor-malverteilten Grundgesamtheiten. Wir testen unter der Voraussetzung σ2

1 = σ22 die Gleich-

heit der Erwartungswerte. Dazu stellen wir die geeigneten Hypothesen auf und berechnendie geschatzten Mittelwerte x = 1.8 und y = 2.25 und das gewogene Mittel der Varianzens2 = 1.079. Die Anzahl Freiheitsgrade ist n = 5 + 4 − 2 = 7. Daraus berechnen wir dieTestgrosse t = 0.646. Zum Signifikanzniveau α = 0.05 bestimmen wir fur die zweiseitige Fra-gestellung den kritischen Wert t7,1−0.025 = 2.365.Nun fuhren wir den statistischen Schluss durch: Es gilt |t| = 0.646 < t7,1−0.025 = 2.365,also wird die Nullhypothese H0 angenommen. Die Mittelwerte x = 1.8 und y = 2.25 weichendemzufolge nur unwesentlich, zufallsbedingt, voneinander ab. Dieses eventuell uberraschendeResultat ist erstens auf die grossen Varianzen und zweitens auf die sehr kleinen Stichproben-umfange zuruckzufuhren.

Aufgaben

Aufgabe 9.3.1. Stammen die drei Messreihen x, y und z unter der Voraussetzung σ2x = σ2

y =

σ2z aus der gleichen normalverteilten Grundgesamtheit? Das Signifikanzniveau ist α = 1%.

x 18.0 14.5 13.5 12.5 23.0 24.0 21.0 17.0 18.5 9.5 14.0y 27.0 34.0 20.5 29.5 20.0 28.0 20.0 26.5 22.0 24.5 34.0 35.5 19.0z 21.5 20.5 19.0 24.5 16.0 13.0 20.0 16.5 17.5 19.0

Aufgabe 9.3.2. Der durchschnittliche Verbrauch eines bestimmten Hilfsstoffes in zwei ver-gleichbaren Filialen einer Unternehmung soll gepruft werden. Dazu wurde der Verbrauchwahrend einer Anzahl Tage bei beiden Filialen ermittelt. Es ergaben sich die folgenden Stich-proben. Kann statistisch erhartet werden, dass die eine Filiale signifikant mehr von dementsprechenden Hilfsstoff verbraucht? Das Signifikanzniveau sei α = 1%.

1.5 3.5 2.6 4.9 6.8 4.9 6.2 5.8 6.4x 8.8 4.5 7.1 8.3 3.6 7.6 7.7 4.9 5.9

3.4 4.5 6.6 5.5 7.7 8.8 4.7 6.35.5 6.6 5.6 6.6 6.1 8.3 5.5 7.3 8.8

y 6.6 6.4 7.8 6.6 7.7 5.5 7.7 6.6 6.65.5 5.4 6.6 5.5 8.8 8.8 9.9 7.7

Page 107: Wahrscheinlichkeitstheorie undStatistik (wst)web.fhnw.ch/.../Stochastik-Skript-Marcel-Steiner.pdf · Anwendungsgebiete der Statistik sind die Industrie und die Technik, die Physik,

9.3. Vergleich zweier Mittelwerte unverbundener Stichproben 99

9.3.2 Zweistichproben-t-Test bei unbekannten Varianzen

Im Falle ungleicher Varianzen der Grundgesamtheiten σ21 6= σ2

2 , oder bei Ablehnung derHypothese uber die Gleichheit der Varianzen σ2

1 und σ22 durch einen geeigneten Test kann

zur Prufung der Hypothese H0 : µ1 = µ2 ein von B. L. Welch (1947) vorgeschlagenerNaherungstest verwendet werden. Diese Aufgabenstellung wird alsBehrens-Fisher-Problembezeichnet.

Beispiel 9.3.3 (Markenjeans). Wir haben zwei Lieferungen von Markenjeans vom gleichenImporteur. Dieser behauptet, beide Lieferungen seien in den USA hergestellt worden. Wirvermuten aber, dass eine Lieferung aus Fernost mit schlechter Qualitat stammt. Wie konnenwir unsere Vermutung uberprufen? Zum Beispiel, indem wir ein Qualitatsmerkmal (z.B.Reisfestigkeit der Jeans nach 20 Mal waschen) bestimmen und von diesem Merkmal denMittelwert fur beide Lieferungen bilden und diese Mittelwerte mit einem unverbundenenZweistichproben-t-Test untersuchen. Da wir vermuten, dass die Lieferungen aus zwei ver-schiedenen Fabriken stammen, mussen wir davon ausgehen, dass die Varianzen verschiedensein konnten.

Beim Behrens-Fisher-Problem sind folgende Voraussetzungen zu beachten:

1. Die normalverteilten Grundgesamtheiten G1 und G2 haben die unbekannten Erwar-tungswerte µ1 und µ2 und die unbekannten Varianzen σ2

1 und σ22 , so genannt hetero-

skedastischer Fall.

2. Es sind zufallig zwei Stichproben x1, . . . , xN1 und y1, . . . , yN2 aus den normalverteiltenGrundgesamtheiten G1 und G2 gewahlt.

Wir wollen nun wissen, ob sich die Mittelwerte x und y der gewahlten Stichproben signi-fikant voneinander unterscheiden um herauszufinden, ob die Stichproben aus der gleichenGrundgesamtheit stammen. Dazu formulieren wir die beiden alternativen Hypothesen:

H0 : µ1 = µ2, d.h., Stichproben stammen aus der gleichen Grundgesamtheit.

H1 : µ1 6= µ2, d.h., Stichproben stammen aus unterschiedlichen Grundgesamtheiten.

Um diese Frage zu beantworten berechnen wir die geschatzten Mittelwerte

x =1

N1

N1∑

i=1

xi und y =1

N2

N2∑

i=1

yi

und die geschatzten Varianzen

s21 =1

N1 − 1

N1∑

i=1

(xi − x)2 und s22 =1

N2 − 1

N2∑

i=1

(yi − y)2

und daraus die gewogene Varianz

s2 =s21N1

+s22N2

.

Wir stellen fest, dass die gewogene Varianz s2 anders berechnet wird, als im Fall gleicherVarianzen (vgl. Gleichung (9.3.a)). Mit diesen Werten berechnen wir nun die Testgrosse

t =x− y

s. (9.3.b)

Page 108: Wahrscheinlichkeitstheorie undStatistik (wst)web.fhnw.ch/.../Stochastik-Skript-Marcel-Steiner.pdf · Anwendungsgebiete der Statistik sind die Industrie und die Technik, die Physik,

100 Kapitel 9. Prufen von Erwartungswerten (Parametertests)

Die Testgrosse t gehorcht wiederum einer Student-t-Verteilung mit

n =

⌊1

c2

N1−1 +(1−c)2

N2−1

⌋mit c =

s21N1

s21N1

+s22N2

Freiheitsgraden, wobei ⌊ . ⌋ die Abrundungsfunktion bezeichnet. Damit konnen wir nun nachVorgabe eines Signifikanzniveaus α die kritische Grosse tn,1−α

2fur die zweiseitige Fragestellung

bestimmen und den statistischen Schluss ziehen:

• Ist die Testgrosse |t| < tn,1−α2, dann wird die Nullhypothese H0 angenommen.

• Ist die Testgrosse |t| ≥ tn,1−α2, dann wird die Nullhypothese H0 auf dem Signifikanzni-

veau α abgelehnt.

Aufgabe

Aufgabe 9.3.3. Mit zwei verschiedenen Holzwerkstoffbindemitteln A und B werden Span-platten hergestellt. Mit dem Bindemittel A erhalten wir 10 Prufkorper, mit dem Mittel Bderen 12. Alle Prufkorper werden einem Querzugfestigkeitstest unterworfen. Folgende Wertewurden gemessen:

A 0.745 0.824 0.804 0.863 0.873 0.814 0.804 0.794 0.804 0.745B 0.745 0.686 1.049 1.059 0.873 0.834 0.735 0.971 0.932 0.932 0.843 0.873

Sind die beiden Bindemittel gleichwertig? Das Signifikanzniveau ist α = 1%.

9.4 Paarweiser Vergleich bei verbundenen Stichproben

Oft stehen wir in der Praxis vor der Aufgabe, Unterschiede zwischen zwei verschiedenenProduktionsverfahren, Behandlungsmethoden, Messgeraten, Messmethoden oder Laborantenmiteinander zu vergleichen. Zu diesem Zweck werden mit beiden Verfahren an denselbenEinheiten Messungen des Merkmals durchgefuhrt und paarweise verglichen. Folgendes Beispielsoll zur naheren Erlauterung dienen.

Beispiel 9.4.1. Zwei verschiedene Messmethoden fur Widerstande sollen miteinander vergli-chen werden. Vergleichsmessungen an funf Widerstanden ergaben das folgende Messprotokoll:

i 1 2 3 4 51. Methode: xi [in Ω] 100.5 102.4 104.3 101.5 98.42. Methode: yi [in Ω] 98.2 99.1 102.4 101.1 96.2

Wir wollen wissen, ob beide Messmethoden als gleichwertig angesehen werden konnen oderob die beobachteten Abweichungen signifikant sind.

Um diese Aufgabe zu bewaltigen, verwenden wir einen paarweisen Vergleichstest, bei demfolgende Voraussetzungen zu beachten sind:

1. Die normalverteilten Grundgesamtheiten G1 und G2 haben die unbekannten Erwar-tungswerte µ1 und µ2 und die unbekannten aber gleichen Varianzen σ2

1 = σ22 = σ2. Der

Wert von σ2 braucht jedoch nicht bekannt zu sein.

Page 109: Wahrscheinlichkeitstheorie undStatistik (wst)web.fhnw.ch/.../Stochastik-Skript-Marcel-Steiner.pdf · Anwendungsgebiete der Statistik sind die Industrie und die Technik, die Physik,

9.4. Paarweiser Vergleich bei verbundenen Stichproben 101

2. Es sind zufallig zwei verbundene Stichproben x1, . . . , xN und y1, . . . , yN aus den normal-verteilten Grundgesamtheiten G1 und G2 gewahlt, d.h., xi lasst sich mit yi vergleichen.

Wir wollen nun wissen, ob sich die verbundenen Messwerte xi und yi signifikant voneinanderunterscheiden. Dazu berechnen wir aus den paarweise zusammengehorigen Messwerten dieDifferenzen

di = xi − yi fur alle i ∈ 1, . . . , N

Diese Reihe der Differenzen di wird als Stichprobe vom Umfang N aus einer normalverteiltenGrundgesamtheit mit dem Erwartungswert µd und der im Allgemeinen unbekannten Varianzσ2d aufgefasst. Die Untersuchung einer signifikanten Abweichung entspricht der Prufung der

alternativen Hypothesen:

H0 : µd = 0, d.h. im Mittel kein Unterschied zwischen den verbundenen Messwerten.

H1 : µd 6= 0, d.h. im Mittel Unterschied zwischen den verbundenen Messwerten.

Damit konnen wir den in Kapitel 9.2 beschriebenen Einstichproben-t-Test mit der Testgrosse

t =d− µd

sd

√N =

d

sd

√N, (9.4.a)

verwenden, wobei

d =1

N

N∑

i=1

di und s2d =1

N − 1

N∑

i=1

(di − d)2

das arithmetische Mittel und die geschatzte Varianz der Differenzenreihe bedeutet. Die Test-grosse t gehorcht einer Student-t-Verteilung mit n = N−1 Freiheitsgraden. Damit konnen wirnun nach Vorgabe eines Signifikanzniveaus α die kritische Grosse tn,1−α

2fur die zweiseitige

Fragestellung bestimmen und den statistischen Schluss ziehen:

• Ist die Testgrosse |t| < tn,1−α2, dann wird die Nullhypothese H0 angenommen, d.h., die

Messwerte unterscheiden sich nur zufallig.

• Ist die Testgrosse |t| ≥ tn,1−α2, dann wird die Nullhypothese H0 auf dem Signifikanzni-

veau α abgelehnt.

Aufgaben

Aufgabe 9.4.1. Es soll untersucht werden, ob zwei Laboranten vergleichbare Ergebnisse beider Bestimmung des Leimungsgrades von Papieren mit einem bestimmten Test liefern. BeideLaboranten haben 8 verschiedene Papiersorten gemessen. Das Signifikanzniveau ist α = 1%.

Sorte 1 2 3 4 5 6 7 8Labor A 18.60 27.60 27.50 25.00 24.50 26.80 29.70 26.50Labor B 18.58 27.37 27.27 24.64 24.10 26.33 29.33 26.63

Aufgabe 9.4.2. Die folgenden Zahlenpaare sind entstanden aus Messungen von spezifischenGewichten einer Anzahl Materialien durch zwei Experimentatoren. Vergleichen Sie sie paar-weise. Sind die beiden Messreihen gleichwertig? Das Signifikanzniveau ist α = 1%.

Page 110: Wahrscheinlichkeitstheorie undStatistik (wst)web.fhnw.ch/.../Stochastik-Skript-Marcel-Steiner.pdf · Anwendungsgebiete der Statistik sind die Industrie und die Technik, die Physik,

102 Kapitel 9. Prufen von Erwartungswerten (Parametertests)

Paar 1 2 3 4 5 6 7 8 9 10x 3.3 3.2 3.8 3.4 3.5 3.4 3.4 3.6 3.9 3.8y 3.2 3.4 3.5 3.2 3.2 3.4 3.4 3.2 3.3 3.1

Paar 11 12 13 14 15 16 17 18 19 20x 3.6 3.3 3.4 3.5 3.8 3.8 3.3 3.2 3.2 3.5y 3.0 3.5 3.4 3.2 3.2 3.5 3.2 3.5 3.1 3.0

Page 111: Wahrscheinlichkeitstheorie undStatistik (wst)web.fhnw.ch/.../Stochastik-Skript-Marcel-Steiner.pdf · Anwendungsgebiete der Statistik sind die Industrie und die Technik, die Physik,

Kapitel 10

Regressionsrechnung

Problemstellung: Gegeben sei eine empirisch vorliegende, d.h. durch eine Anzahl Messpunk-te gegebene Funktion. Gesucht wird eine Funktion f , die diese Funktion nach der GaussschenMethode der kleinsten Quadrate am besten annahert.

Das Prinzip der Regressionsrechnung wird auch als Gausssche Methode der kleinstenQuadrate (MKQ) bezeichnet oder unter dem Begriff Ausgleichsrechnung zusammenge-fasst. Es geht auf Carl Friedrich Gauss, 1777-1855, zuruck.

Abbildung 10.0.i: Carl Friedrich Gauss, 1777-1855

10.1 Regressionsgerade

Gegeben sei eine Punktewolke von n Punkten Pi(xi, yi). Gesucht ist die Gerade mit derGleichung y = ax+b, die diese Punktewolke im Sinne von Gauss moglichst gut annahert. Diesbedeutet, dass die Gerade, d.h. a und b, so gewahlt wird, dass die so genannte Fehlerquadrat-summe

S(a, b) =n∑

i=1

∆y2i ,

die Summe der quadratischen Abweichungen von den gegebenen Punkten minimal ist. Fur

103

Page 112: Wahrscheinlichkeitstheorie undStatistik (wst)web.fhnw.ch/.../Stochastik-Skript-Marcel-Steiner.pdf · Anwendungsgebiete der Statistik sind die Industrie und die Technik, die Physik,

104 Kapitel 10. Regressionsrechnung

b

b

b

b

b

b

Pi(xi, yi)

y = ax + b

∆yi

y

x

Abbildung 10.1.i: Regressionsgerade

die Fehlerquadratsumme erhalten wir

S(a, b) =n∑

i=1

(yi − axi − b)2.

Sie ist zu minimalisieren, also berechnen wir die ersten partiellen Ableitungen und setzen siegleich null

Sa(a, b) = −2

n∑

i=1

(yi − axi − b)xi= 0,

Sb(a, b) = −2n∑

i=1

(yi − axi − b) = 0.

Dies ergibt das lineare Gleichungssystem in den Variablen a und b

a

n∑

i=1

x2i + b

n∑

i=1

xi =n∑

i=1

xiyi, (10.1.a)

a

n∑

i=1

xi + bn =

n∑

i=1

yi, (10.1.b)

welches mit der Cramerschen Regel (vgl. [9] Seiten 86ff) die Losung

a =

det

n∑

i=1

xiyi

n∑

i=1

xi

n∑

i=1

yi n

det

n∑

i=1

x2i

n∑

i=1

xi

n∑

i=1

xi n

=

n

n∑

i=1

xiyi −n∑

i=1

xi

n∑

i=1

yi

n

n∑

i=1

x2i −(

n∑

i=1

xi

)2

Page 113: Wahrscheinlichkeitstheorie undStatistik (wst)web.fhnw.ch/.../Stochastik-Skript-Marcel-Steiner.pdf · Anwendungsgebiete der Statistik sind die Industrie und die Technik, die Physik,

10.1. Regressionsgerade 105

und

b =

det

n∑

i=1

x2i

n∑

i=1

xiyi

n∑

i=1

xi

n∑

i=1

yi

det

n∑

i=1

x2i

n∑

i=1

xi

n∑

i=1

xi n

=

n∑

i=1

x2i

n∑

i=1

yi −n∑

i=1

xi

n∑

i=1

xiyi

n

n∑

i=1

x2i −(

n∑

i=1

xi

)2

ergibt. Der Koeffizient a heisst Regressionskoeffizient und b die Regressionskonstante.Aus der Gleichung (10.1.b) entnehmen wir, dass der Schwerpunkt

P (x, y) = P

(1

n

n∑

i=1

xi,1

n

n∑

i=1

yi

)

der Punktewolke auf der Geraden y = ax+ b liegt.Die Frage stellt sich nun wiederum, ob es sich bei der gefundenen Losung um ein Extremumoder einen Sattelpunkt handelt. Um dies abzuklaren, berechnen wir die zweiten partiellenAbleitungen der Funktion S

Saa(a, b) = 2

n∑

i=1

x2i

Sbb(a, b) = 2n

Sab(a, b) = Sba(a, b) = 2

n∑

i=1

xi

und betrachten

Saa(a, b) · Sbb(a, b)− Sab(a, b)2 = 4n

n∑

i=1

x2i − 4

(n∑

i=i

xi

)2

.

Wir benutzen die Cauchy-Schwarzsche Ungleichung

|〈~u,~v〉| ≤ |~u| · |~v|,

die in Komponenten ausgeschrieben die folgende Form hat

(n∑

i=1

uivi

)2

≤n∑

i=1

u2i

n∑

i=1

v2i .

Wenn die Vektoren ~u und ~v parallel sind, dann gilt die Gleichheit in der Cauchy-SchwarzschenUngleichung.Mit der Setzung u1 = 1, . . . , un = 1 und v1 = x1, . . . , vn = xn folgt

(n∑

i=i

xi

)2

< n

n∑

i=1

x2i ,

Page 114: Wahrscheinlichkeitstheorie undStatistik (wst)web.fhnw.ch/.../Stochastik-Skript-Marcel-Steiner.pdf · Anwendungsgebiete der Statistik sind die Industrie und die Technik, die Physik,

106 Kapitel 10. Regressionsrechnung

da die Vektoren ~u und ~v nicht parallel sind. Damit ergibt sich die hinreichende Bedingung

Saa(a, b) · Sbb(a, b) − Sab(a, b)2 = 4n

n∑

i=1

x2i − 4

(n∑

i=i

xi

)2

> 0

fur einen Extrempunkt. Da Saa(a, b) = 2∑n

i=1 x2i > 0 gilt, handelt es sich in der Tat um ein

Minimum.

Achtung bei Datenmaterial mit Ausreissern!

Die Ausgleichsrechnung ist sehr anfallig auf Ausreisser (vgl. Abbildungen 10.1.ii und 10.1.iii).Deshalb sollten wir immer grosste Vorsicht walten lassen und die Stichprobe zuerst auf Aus-reisser untersuchen. Dies kann entweder grafisch oder mit dem Ausreissertest nach Grubbsgeschehen.

b

b

b

b

b

b

bc

x

y

Abbildung 10.1.ii: Eine falsche Regressi-onsgerade wegen einem Ausreisser

b

b

b

b

b

b

b

bc

x

y

Abbildung 10.1.iii: Eine vorgetauschteAbhangigkeit wegen einem Ausreisser

10.2 Allgemeine Regression

Gegeben seien n Punkte P1(x1, y1), . . . , Pn(xn, yn). Gesucht ist eine Funktion f der Form

f(x) =

m∑

k=1

akfk(x) = a1f1(x) + · · ·+ amfm(x), wobei m < n.

Die m Funktionen f1, . . . , fm sind vorgegebene Funktionen in analytischer Form, wie zumBeispiel x2, sin(x) oder 1

x. Die Koeffizienten a1, . . . , am werden so bestimmt, dass die Fehler-

quadratsumme

S(a1, . . . , am) =

n∑

i=1

(f(xi)− yi)2 =

n∑

i=1

∆y2i

bezuglich der n Punkte Pi minimal wird. Dies stellt eine Verallgemeinerung der bereits be-sprochenen Methode der kleinsten Quadrate dar. Fur a1 = a, a2 = b und f1(x) = x, f2(x) = 1ergibt sich der der Spezialfall einer Ausgleichsgeraden. Wir wollen also die Fehlerquadratsum-me

S(a1, . . . , am) =

n∑

i=1

(f(xi)− yi)2 =

n∑

i=1

(m∑

k=1

akfk(xi)− yi

)2

Page 115: Wahrscheinlichkeitstheorie undStatistik (wst)web.fhnw.ch/.../Stochastik-Skript-Marcel-Steiner.pdf · Anwendungsgebiete der Statistik sind die Industrie und die Technik, die Physik,

10.2. Allgemeine Regression 107

b

b

b

b

b

b

b

b

b

b

b

b

y = f(x)

Pi(xi, yi)

f(xi)

xi

∆yi

y

x

Abbildung 10.2.i: Allgemeine Approximation mit minimalem quadratischen Fehler.

minimieren, dazu berechnen wir alle ersten partiellen Ableitungen

Sa1(a1, . . . , am) = 2

n∑

i=1

(m∑

k=1

akfk(xi)− yi

)f1(xi) = 0,

......

Sam(a1, . . . , am) = 2n∑

i=1

(m∑

k=1

akfk(xi)− yi

)fm(xi) = 0.

Dies ergibt ein lineares Gleichungssystem mit m Gleichungen fur die m unbekannten Koeffi-zienten a1, . . . , am.

n∑

i=1

(m∑

k=1

akfk(xi)

)f1(xi) =

n∑

i=1

(m∑

k=1

akfk(xi)f1(xi)

)=

n∑

i=1

yif1(xi),

......

n∑

i=1

(m∑

k=1

akfk(xi)

)fm(xi) =

n∑

i=1

(m∑

k=1

akfk(xi)fm(xi)

)=

n∑

i=1

yifm(xi).

Page 116: Wahrscheinlichkeitstheorie undStatistik (wst)web.fhnw.ch/.../Stochastik-Skript-Marcel-Steiner.pdf · Anwendungsgebiete der Statistik sind die Industrie und die Technik, die Physik,

108 Kapitel 10. Regressionsrechnung

Jetzt sollen die Summenbildungen vertauscht werden1. Wir erhalten

m∑

k=1

(n∑

i=1

akfk(xi)f1(xi)

)=

m∑

k=1

ak

(n∑

i=1

fk(xi)f1(xi)

)=

n∑

i=1

yif1(xi),

......

m∑

k=1

(n∑

i=1

akfk(xi)fm(xi)

)=

m∑

k=1

ak

(n∑

i=1

fk(xi)fm(xi)

)=

n∑

i=1

yifm(xi)

und ausfuhrlich geschrieben

a1

(n∑

i=1

f1(xi)f1(xi)

)+ · · ·+ am

(n∑

i=1

fm(xi)f1(xi)

)=

n∑

i=1

yif1(xi),

... (10.2.a)

a1

(n∑

i=1

f1(xi)fm(xi)

)+ · · ·+ am

(n∑

i=1

fm(xi)fm(xi)

)=

n∑

i=1

yifm(xi).

Beim linearen Gleichungssystem (10.2.a) handelt es sich um das so genannte Normalglei-chungssystem der Ausgleichsrechnung. Dieses lasst sich auch mit Hilfe vom Matrizen schrei-ben

n∑

i=1

f1(xi)f1(xi) · · ·n∑

i=1

fm(xi)f1(xi)

.... . .

...n∑

i=1

f1(xi)fm(xi) · · ·n∑

i=1

fm(xi)fm(xi)

·

a1...am

=

n∑

i=1

yif1(xi)

...n∑

i=1

yifm(xi)

. (10.2.b)

In abkurzender Schreibweise konnen wir das obige lineare Normalgleichungssystem (10.2.b)gemass

A~a = ~b (10.2.c)

schreiben, wobei die m gesuchten unbekannten Koeffizienten a1, . . . , am zum Vektor ~a zusam-mengefasst wurden. Die Koeffizienten

Akj =

n∑

i=1

fk(xi)fj(xi), k, j ∈ 1, . . . ,m

1Wir betrachten dazu ein vereinfachtes Beispiel von Doppelsummen und der Vertauschung von Summen-zeichen

3∑

i=1

(

2∑

k=1

aik

)

= (a11 + a12) + (a21 + a22) + (a31 + a32)

= (a11 + a21 + a31) + (a12 + a22 + a32)

=2∑

k=1

(

3∑

i=1

aik

)

.

Page 117: Wahrscheinlichkeitstheorie undStatistik (wst)web.fhnw.ch/.../Stochastik-Skript-Marcel-Steiner.pdf · Anwendungsgebiete der Statistik sind die Industrie und die Technik, die Physik,

10.2. Allgemeine Regression 109

der symmetrischen (m×m)-Matrix A und die Koeffizienten

bk =n∑

i=1

yifk(xi), k ∈ 1, . . . ,m.

des Storvektors~b berechnen wir aus den Koordinaten der gegebenen Punkte. Damit lassen sichnun die gesuchten Koeffizienten a1, . . . , am durch losen des linearen Normalgleichungssystems(10.2.c) berechnen. Bei grossem m geschieht dies mittels Computer. Damit ist das gegebeneProblem im Prinzip gelost.

Beispiel 10.2.1. Eine Punktmenge sei durch eine Funktion der Form

f(x) = ax+ b sin(x) [x im Bogenmass]

im Gaussschen Sinne zu approximieren. Die folgenden 8 Punkte P1, . . . , P8 seien tabellarischgegeben

i 1 2 3 4 5 6 7 8

xi 0 1 2 3 4 5 6 7

yi 0.0 0.2 1.1 2.9 4.8 6.0 6.3 6.3

Wir minimieren

S(a, b) =

8∑

i=1

(axi + b sin(xi)− yi)2 ,

dazu berechnen wir die ersten partiellen Ableitungen

Sa(a, b) = 2

8∑

i=1

(axi + b sin(xi)− yi)xi = 0,

Sb(a, b) = 2

8∑

i=1

(axi + b sin(xi)− yi) sin(xi) = 0.

Dies ergibt das lineare Gleichungssystem2 mit zwei Gleichungen fur die zwei unbekanntenKoeffizienten a und b.

8∑

i=1

ax2i +

8∑

i=1

bxi sin(xi) =

8∑

i=1

yixi,

8∑

i=1

axi sin(xi) +

8∑

i=1

b sin2(xi) =

8∑

i=1

yi sin(xi)

oder

a

8∑

i=1

x2i + b

8∑

i=1

xi sin(xi) =8∑

i=1

yixi, (10.2.d)

a

8∑

i=1

xi sin(xi) + b

8∑

i=1

sin2(xi) =8∑

i=1

yi sin(xi).

2Dieses lineare Gleichungssystem hatte sich auch direkt aus dem linearen Normalgleichungssystem (10.2.a)durch Einsetzen von n = 8, m = 2 und a1 = a, a2 = b und f1(x) = x, f2(x) = sin(x) ergeben.

Page 118: Wahrscheinlichkeitstheorie undStatistik (wst)web.fhnw.ch/.../Stochastik-Skript-Marcel-Steiner.pdf · Anwendungsgebiete der Statistik sind die Industrie und die Technik, die Physik,

110 Kapitel 10. Regressionsrechnung

Daraus lassen sich die gesuchten Koeffizienten a und b berechnen. Dieses Normalgleichungs-system lasst sich wiederum mit Hilfe einer Matrizengleichung schreiben

8∑

i=1

x2i

8∑

i=1

xi sin(xi)

8∑

i=1

xi sin(xi)

8∑

i=1

sin2(xi)

·(

a

b

)=

8∑

i=1

yixi

8∑

i=1

yi sin(xi)

.

Aus den Koordinaten der gegebenen 8 Punkte P1(x1, y1), . . . , P8(x8, y8) lassen sich die Koef-fizienten dieses Normalgleichungssystems numerisch berechnen. Wir erhalten3

8∑

i=1

x2i = 140,

8∑

i=1

sin2(xi) = 3.5568,

8∑

i=1

xi sin(xi) = −1.8160,

8∑

i=1

xiyi = 142.2,

8∑

i=1

yi sin(xi) = −5.4297.

Damit ergibt sich das zu losende lineare Normalgleichungssystem

(140 −1.8160

−1.8160 3.5568

)·(

a

b

)=

(142.2

−5.4297

)

mit der Losung4 a = 1.0026 und b = −1.0147. Die gesuchte Ausgleichsfunktion ist also durch

f(x) = 1.0026x − 1.0147 sin(x)

3x im Bogenmass4Wir konnten auch mit der Cramerschen Regel (vgl. [9], Seiten 86ff) die Losung

a =

det

8∑

i=1

yixi

8∑

i=1

xi sin(xi)

8∑

i=1

yi sin(xi)8∑

i=1

sin2(xi)

det

8∑

i=1

x2i

8∑

i=1

xi sin(xi)

8∑

i=1

xi sin(xi)

8∑

i=1

sin2(xi)

=

8∑

i=1

xiyi

8∑

i=1

sin2(xi)−8∑

i=1

yi sin(xi)8∑

i=1

xi sin(xi)

8∑

i=1

x2i

8∑

i=1

sin2(xi)−(

8∑

i=1

xi sin(xi)

)2

und

b =

det

8∑

i=1

x2i

8∑

i=1

yixi

8∑

i=1

xi sin(xi)

8∑

i=1

yi sin(xi)

det

8∑

i=1

x2i

8∑

i=1

xi sin(xi)

8∑

i=1

xi sin(xi)8∑

i=1

sin2(xi)

=

8∑

i=1

x2i

8∑

i=1

yi sin(xi)−8∑

i=1

xi sin(xi)

8∑

i=1

xiyi

8∑

i=1

x2i

8∑

i=1

sin2(xi)−(

8∑

i=1

xi sin(xi)

)2.

direkt aus dem Normalgleichungssystem (10.2.d) explizit berechnen.

Page 119: Wahrscheinlichkeitstheorie undStatistik (wst)web.fhnw.ch/.../Stochastik-Skript-Marcel-Steiner.pdf · Anwendungsgebiete der Statistik sind die Industrie und die Technik, die Physik,

10.2. Allgemeine Regression 111

gegeben.Die berechneten Werte im Vergleich zu den gegebenen ergeben sich zu:

i 1 2 3 4 5 6 7 8

xi 0 1 2 3 4 5 6 7

yi 0.0 0.2 1.1 2.9 4.8 6.0 6.3 6.3

f(xi) 0.0000 0.1488 1.0825 2.8646 4.7783 5.9860 6.2991 6.3526

Aufgaben

Aufgabe 10.2.1. In einem Wald sind die Durchmesser x1, . . . , xn und die dazu gehorigenHohen y1, . . . , yn von n Baumen gemessen worden, so dass n empirische Zahlenpaare (x1, y1),. . . , (xn, yn) gegeben sind. Durch die Punkte kann am ehesten eine passende logarithmischeAusgleichskurve gelegt werden. Bestimmen Sie diese Funktion in der Form

f(x) = a log10(x) + b.

Numerisches Beispiel: P1(1, 1), P2(2, 2), P3(4, 2.5)

Aufgabe 10.2.2. Bestimmen Sie die beste Funktion der Form

f(x) = ax2 + be−x

zu den folgenden Punkten:

i 1 2 3 4 5

xi 0 1 2 3 4

yi -1.0 1.6 7.9 17.9 32.0

Aufgabe 10.2.3. Bestimmen Sie die beste Funktion der Form

f(x) = a+ bx+ c sin(x) [x im Bogenmass]

zu den folgenden Punkten:

i 1 2 3 4

xi 0 1 2 3

yi 1.0000 1.1585 2.0907 3.8589

Aufgabe 10.2.4. Bestimmen Sie die beste Funktion der Form

f(x) = a+ bx2 + c sin(x) + d cos(x) [x im Bogenmass]

zu den folgenden Punkten:

i 1 2 3 4 5 6

xi -2 -1 0 1 2 3

yi -3.7416 1.2391 4.0000 2.9221 -1.9230 -8.8389

Losungen

Losung 10.2.1. a = 2.491 und b = 1.083

Losung 10.2.2. a = 1.999897 und b = −1.007806

Losung 10.2.3. a = 0.9999968, b = 1.00001021 und c = −1.0000318

Losung 10.2.4. a = 2.0000, b = −1.0000, c = 1.0000 und d = 1.9999

Page 120: Wahrscheinlichkeitstheorie undStatistik (wst)web.fhnw.ch/.../Stochastik-Skript-Marcel-Steiner.pdf · Anwendungsgebiete der Statistik sind die Industrie und die Technik, die Physik,

112 Kapitel 10. Regressionsrechnung

Page 121: Wahrscheinlichkeitstheorie undStatistik (wst)web.fhnw.ch/.../Stochastik-Skript-Marcel-Steiner.pdf · Anwendungsgebiete der Statistik sind die Industrie und die Technik, die Physik,

Kapitel 11

Regressionsanalyse

DieRegressionsanalyse behandelt folgendes Problem: Aus den Realisierungen einer Zufalls-grosse X sollen wahrscheinlichkeitstheoretische Aussagen, d.h., Vorhersagen uber die Werteeiner zweiten Zufallsgrosse Y gemacht werden. Dabei sind naturlich nur dann sinnvolle Vor-hersagen moglich, wenn die beiden Zufallsgrossen X und Y abhangig sind, wenn also eineVerbindung zwischen X und Y besteht.

11.1 Allgemeines

Beispiel 11.1.1. Der Bremsweg eines bestimmten Autos hangt wesentlich von der Geschwin-digkeit ab, die das Auto unmittelbar vor dem Bremsbeginn erreicht hat. Diese Geschwindigkeitbestimmt jedoch den Bremsweg nicht eindeutig, weil er durch viele weitere Grossen beeinflusstwird, z.B. durch den Zustand der Bremsen und Reifen, die Strassenbeschaffenheit, das Lade-gewicht und das Verhalten des Fahrers wahrend des Bremsvorgangs. Werden bei konstanterGeschwindigkeit x mehrere Bremsversuche unternommen, so erhalten wir im Allgemeinenverschiedene Bremswege als Realisierungen einer Zufallsvariable Y (x). Zu jedem Geschwin-digkeitswert x gehort also eine Zufallsvariable Y (x). Aus Erfahrung ist bekannt, dass dererwartete Bremsweg und die Streuung der Bremswege mit wachsender Geschwindigkeit x

grosser werden. Es gilt die Faustregel “grossere Geschwindigkeit gleich langerer Bremsweg”.Aus der Geschwindigkeit konnen also keine deterministischen, sondern nur wahrschein-lichkeitstheoretische Aussagen uber den Bremsweg gemacht werden. Diesen Zusammenhangkonnen wir mit einer Regression beschreiben.Die Regressionsanalyse gibt uns Auskunft daruber, wie gut die angepasste Kurve zur Rea-litat passt, d.h., ob das gewahlte Modell (z.B. linear, polynomial oder exponentiel) ein ange-passtes ist.

Beispiel 11.1.2. Im Gegensatz zu Beispiel 11.1.1 ist der Zusammenhang zwischen der Sei-tenlange x eines Quadrates und dessen Flacheninhalt y ein deterministischer. Es gilt diefunktionale Beziehung y = x2. Durch die Vorgabe einer Seitenlange x ist also der Flacheninhalty = x2 eindeutig bestimmt.

Wir unterscheiden zwei Arten von Variablen:

1. Nichtstochastische Variablen, die fest vorgegeben sind, wie Stutzpunkte, Klassen-mitten oder Messpunkte (z.B. Geschwindigkeit vor Bremsbeginn in Beispiel 11.1.1).Diese werden im Allgemeinen mit x bezeichnet.

113

Page 122: Wahrscheinlichkeitstheorie undStatistik (wst)web.fhnw.ch/.../Stochastik-Skript-Marcel-Steiner.pdf · Anwendungsgebiete der Statistik sind die Industrie und die Technik, die Physik,

114 Kapitel 11. Regressionsanalyse

2. Stochastische Variablen, die meistens einer Normalverteilung gehorchen (z.B. Brems-weg in Beispiel 11.1.1). Diese werden im Allgemeinen mit y bezeichnet.

b

b

b

b

b

b

b

b

b

b

b

b

b

b

b

b

b

b b

b

b

b

b

b

b

xi

yi

x

y

Abbildung 11.1.i: Nichtstochastische Variablen in x-Richtung und im Allgemeinen normalver-teilte stochastische Variablen in y-Richtung

Der Zusammenhang des durchschnittlichen Wertes y von y je Messpunkt mit den verschie-denen Messpunkten der nichtstochastischen Variablen x heisst Regression. Im Folgendennehmen wir an, dass die stochastischen Variablen y einer Normalverteilung mit den Parame-tern µ und σ2 gehorchen. Im Wesentlichen gibt es drei typische Falle der Regression:

1. Kein Zusammenhang zwischen x und y, d.h., die y-Werte sind unabhangig von denx-Werten (vgl. Abbildung 11.1.ii).

2. Linearer Zusammenhang zwischen x und y, d.h., die Verbindungslinie der Mittel-werte y der einzelnen Verteilungen liegen alle auf einer Geraden, der so genannten Re-gressionsgeraden (vgl. Abbildung 11.1.iii). Die Regressionsgerade ist der geometrischeOrt der wahrscheinlichsten Werte von y je Messpunkt x.

3. Nichtlinearer Zusammenhang zwischen x und y, d.h., die Verbindungslinie der Mit-telwerte y der einzelnen Verteilungen liegen z.B. auf einer polynomialen (Grad mindes-tens zwei), exponentiellen oder logarithmischen Kurve (vgl. Abbildung 11.1.iv).

b

y1

x1

b

y2

x2

b

y3

x3

b

y4

x4

b

y5

x5

b

y6

x6

b

y7

x7

y = const

x

y

Abbildung 11.1.ii: Kein Zusammenhangzwischen x und y.

b

y1

x1

b

y2

x2

b

y3

x3

b

y4

x4

b

y5

x5

b

y6

x6

b

y7

x7

y = ax + b

x

y

Abbildung 11.1.iii: Linearer Zusammen-hang zwischen x und y

Page 123: Wahrscheinlichkeitstheorie undStatistik (wst)web.fhnw.ch/.../Stochastik-Skript-Marcel-Steiner.pdf · Anwendungsgebiete der Statistik sind die Industrie und die Technik, die Physik,

11.2. Regressionsgerade 115

b

y1

x1

b

y2

x2

b

y3

x3

b

y4

x4

b

y5

x5

b

y6

x6

b

y7

x7

y = f(x)

x

y

Abbildung 11.1.iv: Nichtlinearer Zusam-menhang zwischen x und y

In der Praxis sind meistens nicht so viele Messpunkte vorhanden, dass fur viele Werte vonx annahernde Normalverteilungen entstehen. Es steht meistens eine mehr oder weniger be-schrankte Stichprobe pro Messpunkt zur Verfugung, die von Messpunkt zu Messpunkt anderssein kann.

11.2 Regressionsgerade

Gegeben sei eine Punktewolke von N Punkten P1(x1, y1), . . . , PN (xN , yN ). Gesucht ist dieGerade mit der Gleichung y = ax + b, die diese Punktewolke im Sinne von Gauss moglichstgut annahert. Dies bedeutet, dass die Konstanten a und b der Geraden so gewahlt werden,dass die so genannte Fehlerquadratsumme

S(a, b) =N∑

i=1

∆y2i =N∑

i=1

(yi − axi − b)2,

die Summe der senkrechten quadratischen Abweichungen von den gegebenen Punkten zur Ge-raden minimal ist. Sie ist zu minimieren, also berechnen wir die ersten partiellen Ableitungen

b

b

b

b

b

b

b

b

b b

b

b

b

b

b

b

b

b

Pi(xi, yi)

y = ax + b

∆yi

y

x

Abbildung 11.2.i: Regressionsgerade

Page 124: Wahrscheinlichkeitstheorie undStatistik (wst)web.fhnw.ch/.../Stochastik-Skript-Marcel-Steiner.pdf · Anwendungsgebiete der Statistik sind die Industrie und die Technik, die Physik,

116 Kapitel 11. Regressionsanalyse

und setzen diese gleich null

Sa(a, b) = −2N∑

i=1

(yi − axi − b)xi = 0 und Sb(a, b) = −2N∑

i=1

(yi − axi − b) = 0.

Dies ergibt das lineare Gleichungssystem in den Variablen a und b

a

N∑

i=1

x2i + b

N∑

i=1

xi =N∑

i=1

xiyi und a

N∑

i=1

xi + bN =N∑

i=1

yi, (11.2.a)

welches die Losung

a =

N

N∑

i=1

xiyi −N∑

i=1

xi

N∑

i=1

yi

N

N∑

i=1

x2i −(

N∑

i=1

xi

)2 und b =

N∑

i=1

x2i

N∑

i=1

yi −N∑

i=1

xi

N∑

i=1

xiyi

N

n∑

i=1

x2i −(

N∑

i=1

xi

)2

hat. Der Koeffizient a heisstRegressionskoeffizient und b dieRegressionskonstante. Ausder zweiten Gleichung (11.2.a) entnehmen wir, dass der Schwerpunkt

P (x, y) = P

(1

N

N∑

i=1

xi,1

N

N∑

i=1

yi

)

der Punktewolke auf der Regressionsgeraden y = ax+ b liegt.

11.3 Regressionsanalyse einer Geraden

Wir sollten folgendes Problem bei der Regressionsrechnung nicht aus den Augen verlieren:Da die Punkte um die Regressionsgerade mehr oder weniger streuen, enthalten sowohl derRegressionskoeffizient a als auch die Regressionskonstante b eine gewisse Unsicherheit. DieFrage stellt sich nun, ab welcher Grosse des Regressionskoeffizienten a von einem wirklichenEinfluss der Grosse x auf die Grosse y gesprochen werden kann, d.h., ist die Regressionsgeradesignifikant von einer zur x-Achse parallelen Geraden verschieden.

Eine Berucksichtigung der Variable x ist nur dann sinnvoll, wenn die Veranderung von y nichtrein zufallig auf eine Veranderung von x erfolgt. Mit Hilfe eines statistischen Tests, der sogenannten Regressionsanalyse, wollen wir nun diese Abhangigkeit quantifizieren.

Es seien also N Messpunkte P1(x1, y1), . . . , PN (xN , yN ) gegeben, dabei bezeichnet x die nicht-stochastische und y die stochastische Variable. Nach der Methode der kleinsten Quadrate (vgl.Kapitel 11.2) wurde eine beste Gerade Y = ax+ b an diese Punktewolke angepasst.

Uber den Regressionskoeffizienten a stellen wir nun eine Annahme in Form einer statistischenHypothese auf.

H0 : a = 0, d.h., es besteht keine Abhangigkeit zwischen x und y.

H1 : a 6= 0, d.h., es besteht eine Abhangigkeit zwischen x und y.

Page 125: Wahrscheinlichkeitstheorie undStatistik (wst)web.fhnw.ch/.../Stochastik-Skript-Marcel-Steiner.pdf · Anwendungsgebiete der Statistik sind die Industrie und die Technik, die Physik,

11.3. Regressionsanalyse einer Geraden 117

Zur Beantwortung dieser Fragestellung berechnen wir aus den Koordinaten der N PunkteP1(x1, y1), . . . , PN (xN , yN ) die (theoretischen) Werte

Yi = axi + b

und damit die folgenden Grossen

s2x =1

N − 1

N∑

i=1

(xi − x)2 mit x =1

N

N∑

i=1

xi,

s2y =1

N − 2

N∑

i=1

(yi − Yi)2 =

1

N − 2

N∑

i=1

(yi − (axi + b))2

und aus diesen die Testgrosse

tReg =√N − 1

sx

sya. (11.3.a)

Zur quantitativen Beurteilung der Abhangigkeit der Variablen y von der Grosse x sind fol-gende Gesichtspunkt massgebend:

• Die Varianz s2x der unabhangigen x-Werte um den Mittelwert x. Grosse Streuung inx-Richtung gibt Sicherheit fur die Aussage. Fur die Praxis heisst das, es muss dafurgesorgt werden, dass die Grossen xi uber ein moglichst grossen Bereich vorhanden sind.

• Die Restvarianz s2y der Punkte um die Gerade, d.h. die Fehlerquadratsumme in y-Richtung. Kleine Streuungen in y-Richtung ergeben eine Zuverlassigkeit der Aussage.

Die Testgrosse tReg ist Student-t-verteilt mit n = N − 2 Freiheitsgraden. Das Testverfahrenfur die Nullhypothese kann dann folgendermassen formuliert werden: Nach Wahl eines Signi-fikanzniveaus α ermitteln wir mit Hilfe von Tafel T.3 oder einem Computerprogramm denkritischen Wert tn,1−α

2bei einer zweiseitigen Fragestellung. Dann ziehen wir den statisti-

schen Schluss:

• Ist die Testgrosse |tReg| < tn,1−α2, dann wird die Nullhypothese H0 angenommen, d.h.,

es besteht keine signifikante Abhangigkeit zwischen x und y.

• Ist die Testgrosse |tReg| ≥ tn,1−α2, dann wird die Nullhypothese H0 auf dem Signifikanz-

niveau α abgelehnt, d.h., es besteht eine signifikante Abhangigkeit.

Beispiel 11.3.1. Die Abhangigkeit der stochastischen Variablen y von der Grosse x solluntersucht werden. Die folgenden Messwerte liegen vor.

xi 0 1 3 4 5 6 7 8yi 9 8 7 5 5 3 3 1

Kann von einer signifikanten Abhangigkeit gesprochen werden? Wir wahlen das Signifikanz-niveau α = 0.001. Aus dem vorliegenden Datenmaterial berechnen wir vorerst die Regressi-onsgerade mit der Methode der kleinsten Quadrate

y = −0.9595x + 9.2027.

Nun wollen wir mit der Regressionsanalyse uberprufen, ob a = −0.9595 eine signifikanteAbhangigkeit darstellt. Dazu berechnen wir s2x = 7.9286 und s2y = 0.2973 und daraus die

Page 126: Wahrscheinlichkeitstheorie undStatistik (wst)web.fhnw.ch/.../Stochastik-Skript-Marcel-Steiner.pdf · Anwendungsgebiete der Statistik sind die Industrie und die Technik, die Physik,

118 Kapitel 11. Regressionsanalyse

Testgrosse tReg = −13.1092, wobei N = 8 ist. Zum Signifikanzniveau 0.1% bestimmen wiraus Tafel T.3 oder einem Computerprogramm den kritischen Wert t6,0.9995 = 5.9587.Da |tReg| = 13.1092 ≥ t6,0.9995 = 5.9587 gilt, wird die Nullhypothese abgelehnt. D.h., x

hat einen wesentlichen Einfluss auf y oder anders ausgedruckt, der Regressionskoeffizienta = −0.9595 ist signifikant von Nullverschieden.

Beispiel 11.3.2 (Meeresspiegel in Venedig). Die folgende Messreihe zeigt den jahrlichenmaximalen Meeresspiegel1 [in cm] in Venedig fur die Jahre von 1931 bis 1981.

xi 1931 1932 1933 1934 1935 1936 1937 1938 1939 1940 1941 1942 1943yi 103 78 121 116 115 147 119 114 89 102 99 91 97xi 1944 1945 1946 1947 1948 1949 1950 1951 1952 1953 1954 1955 1956yi 106 105 136 126 132 104 117 151 116 107 112 97 95xi 1957 1958 1959 1960 1961 1962 1963 1964 1965 1966 1967 1968 1969yi 119 124 118 145 122 114 118 107 110 194 138 144 138xi 1970 1971 1972 1973 1974 1975 1976 1977 1978 1979 1980 1981yi 123 122 120 114 96 125 124 120 132 166 134 138

Ist die Zunahme des Meeresspiegels in Venedig uber diese Periode signifikant? Das Signifi-kanzniveau sei 5%.Die Regressionsgerade lautet

y = 0.57x − 989.38.

Mit der Regressionsanalyse uberprufen wir, ob die Steigung a = 0.57 signifikant von Null-verschieden ist. Dazu berechnen wir s2x = 221.00 und s2y = 346.70 und daraus die TestgrossetReg = 3.20, wobei N = 51 ist. Zum Signifikanzniveau 5% bestimmen wir aus Tafel T.3 odereinem Computerprogramm den kritischen Wert t49,0.975 = 2.01.

Da |tReg| = 3.20 ≥ t49,0.975 = 2.01 gilt, wird die Nullhypothese abgelehnt. Das heisst, derAnstieg des Meeresspiegels in Venedig in den Jahren 1931 bis 1981 ist real.

Aufgaben

Aufgabe 11.3.1. Die Abhangigkeit der stochastischen Variablen y von der Grosse x solluntersucht werden. Die folgenden Messwerte liegen vor.

xi 1 2 5 8 10yi 1 1 4 3 6

Kann von einer signifikanten Abhangigkeit gesprochen werden? Das Signifikanzniveau ist 1%.

Aufgabe 11.3.2. Die Abhangigkeit der stochastischen Variablen y von der Grosse x solluntersucht werden. Die folgenden Messwerte liegen vor.

xi 1.1 1.2 1.4 1.6 1.7 1.9 2.0 2.3 2.7 2.8 2.9 3.3 3.8 4.0yi 2.0 1.9 1.8 1.8 1.7 1.7 1.6 1.5 1.5 1.4 1.3 1.2 1.1 0.9xi 4.6 5.1 6.3 7.8 8.3 9.4 10.3 10.5 10.7 11.0 11.6 11.9 12.0 12.6yi 0.8 0.8 0.7 0.6 0.5 0.4 0.3 0.2 0.2 0.1 0.4 0.1 0.0 -0.1

Kann von einer signifikanten Abhangigkeit gesprochen werden? Das Signifikanzniveau ist 1%.

1Der aufmerksame Leser wird realisieren, dass der steigende Meeresspiegel nicht auf ein Ansteigen desWassers in der Adria zuruckzufuhren ist, sondern auf das Sinken der Markierung an der der Meeresspiegel inVenedig abgelesen wird.

Page 127: Wahrscheinlichkeitstheorie undStatistik (wst)web.fhnw.ch/.../Stochastik-Skript-Marcel-Steiner.pdf · Anwendungsgebiete der Statistik sind die Industrie und die Technik, die Physik,

11.4. Regressionsanalyse zweier Geraden 119

11.4 Regressionsanalyse zweier Geraden

In der Praxis kommt es oft vor, dass zwei Stichproben der Umfange N1 und N2 mit denMesspunkten P11(x11, y11), . . . , P1N1(x1N1 , y1N1) und P21(x21, y21), . . . , P2N2(x2N2 , y2N2) gege-ben sind. Daraus lassen sich mit der Methode der kleinsten Quadrate (vgl. Kapitel 11.2) zweiRegressionsgeraden

y = a1x+ b1 und y = a2x+ b2.

berechnen. Es interessiert, ob die beiden Regressionsgeraden nur zufallig voneinander abwei-chen.

b b b

b

b

b

b

b

b

b

bc bc

bc

bc

bc

bc

bc

bc

y = a1x + b1

y = a2x + b2

x

y

Abbildung 11.4.i: Regressionsanalyse zweier Geraden. Unterscheiden sich die beiden Gereadensignifikant, d.h., sind die Steigungen oder die y-Achsenabschnitte verschieden?

Prufung der Regressionskoeffizienten

Dazu stellen wir die alternativen Hypothesen auf.

Ha,0 : a1 = a2, d.h., gleiche Steigungen der beiden Regressionsgeraden.

Ha,1 : a1 6= a2, d.h., verschiedene Steigungen der beiden Regressionsgeraden.

Zur Beantwortung dieser Fragestellung berechnen wir aus den zwei gegebenen StichprobenP11(x11, y11), . . ., P1N1(x1N1 , y1N1) und P21(x21, y21), . . . , P2N2(x2N2 , y2N2) die Grossen

x1 =1

N1

N1∑

i=1

x1i und x2 =1

N2

N2∑

i=1

x2i

und

s2x1=

1

N1 − 1

N1∑

i=1

(x1i − x1)2 und s2x2

=1

N2 − 1

N2∑

i=1

(x2i − x2)2 (11.4.a)

und

s2y1 =1

N1 − 2

N1∑

i=1

(y1i − (a1x1i + b1))2 und s2y2 =

1

N2 − 2

N2∑

i=1

(y2i − (a2x2i + b2))2

(11.4.b)

Page 128: Wahrscheinlichkeitstheorie undStatistik (wst)web.fhnw.ch/.../Stochastik-Skript-Marcel-Steiner.pdf · Anwendungsgebiete der Statistik sind die Industrie und die Technik, die Physik,

120 Kapitel 11. Regressionsanalyse

und damit

s2a =(N1 − 2)s2y1 + (N2 − 2)s2y2

N1 +N2 − 4

(1

(N1 − 1)s2x1

+1

(N2 − 1)s2x2

).

Aus diesen Grossen entsteht dann schlussendlich die Testgrosse

ta =a1 − a2

sa. (11.4.c)

Die Testgrosse ta ist Student-t-verteilt mit n = N1 + N2 − 4 Freiheitsgraden. Nach Wahleines Signifikanzniveaus α ermitteln wir mit Hilfe von Tafel T.3 oder einem Computerpro-gramm den kritischen Wert tn,1−α

2bei einer zweiseitigen Fragestellung. Dann ziehen wir den

statistischen Schluss:

• Ist die Testgrosse |ta| < tn,1−α2, dann wird die Nullhypothese Ha,0 angenommen, d.h.,

es besteht kein signifikanter Unterschied zwischen den Steigungen der beiden Regressi-onsgeraden.

• Ist die Testgrosse |ta| ≥ tn,1−α2, dann wird die Nullhypothese Ha,0 auf dem Signifikanz-

niveau α abgelehnt, d.h., die beiden Regressionsgeraden haben signifikant verschiedeneSteigungen.

Wird die Nullhypothese nicht abgelehhnt, so mussen wir zur Untersuchung der Gleichheitder beiden Regressionsgeraden auch noch die Regressionskonstanten auf einen signifikantenUnterschied hin untersuchen. Dies geschieht abermals mit einem statistischen Test und einergeeigneten Testgrosse.

Prufung der Regressionskonstanten

Dazu stellen wir die alternativen Hypothesen auf.

Hb,0 : b1 = b2, d.h., gleiche y-Achsenabschnitte der beiden Regressionsgeraden.

Hb,1 : b1 6= b2, d.h., verschiedene y-Achsenabschnitte der beiden Regressionsgeraden.

Zur Beantwortung dieser Fragestellung berechnen wir aus den Messpunkten die Grossen (vgl.Gleichungen (11.4.a) und (11.4.b)) s2x1

, s2x2und s2y1 , s

2y2

und

s2b =(N1 − 2)s2y1 + (N2 − 2)s2y2

N1 +N2 − 4

(x21

(N1 − 1)s2x1

+x22

(N2 − 1)s2x2

+1

N1+

1

N2

).

Aus diesen Grossen entsteht dann schlussendlich die Testgrosse

tb =b1 − b2

sb. (11.4.d)

Die Testgrosse tb ist Student-t-verteilt mit n = N1 + N2 − 4 Freiheitsgraden. Nach Wahleines Signifikanzniveaus α ermitteln wir mit Hilfe von Tafel T.3 oder einem Computerpro-gramm den kritischen Wert tn,1−α

2bei einer zweiseitigen Fragestellung. Dann ziehen wir den

statistischen Schluss:

Page 129: Wahrscheinlichkeitstheorie undStatistik (wst)web.fhnw.ch/.../Stochastik-Skript-Marcel-Steiner.pdf · Anwendungsgebiete der Statistik sind die Industrie und die Technik, die Physik,

11.4. Regressionsanalyse zweier Geraden 121

• Ist die Testgrosse |tb| < tn,1−α2, dann wird die Nullhypothese Hb,0 angenommen, d.h.,

es besteht kein signifikanter Unterschied zwischen den y-Achsenabschnitten der beidenRegressionsgeraden.

• Ist die Testgrosse |tb| ≥ tn,1−α2, dann wird die Nullhypothese Hb,0 auf dem Signifikanz-

niveau α abgelehnt, d.h., die beiden Regressionsgeraden haben signifikant verschiedeney-Achsenabschnitte.

Haben wir beide Nullhypothesen Ha,0 und Hb,0 angenommen, so konnen wir davon ausgehen,dass sich die beiden Regressionsgeraden nur zufallig voneinander unterscheiden.

Aufgaben

Aufgabe 11.4.1. Es soll untersucht werden, ob die Steigungen und die y-Achsenabschnitteder beiden Ausgleichsgeraden, die durch das folgende Datenmaterial gegeben sind, signifikantvoneinander verschieden sind. Das Signifikanzniveau ist 10%.

x1i 1 2 5 8 10 x2i 3 4 5 6 7 8y1i 1 1 4 3 6 y2i 1 1 1 7 6 7

Aufgabe 11.4.2. Es soll untersucht werden, ob die beiden Ausgleichsgeraden, die durch dasfolgende Datenmaterial gegeben sind, signifikant voneinander verschieden sind. Das Signifi-kanzniveau ist 5%.

x1i 1 2 5 8 10 12 14 18 x2i 3 4 5 6 7 8 9 10 11 12y1i 1 1 4 3 6 9 8 13 y2i 1 1 1 7 6 7 8 9 14 15

Aufgabe 11.4.3. Es soll untersucht werden, ob die beiden Ausgleichsgeraden, die durch dasfolgende Datenmaterial gegeben sind, signifikant voneinander verschieden sind. Das Signifi-kanzniveau ist 1%.

x1i 1.0 1.3 1.4 1.6 1.7 1.9 2.0 2.1 2.4 2.6 2.7 2.9 3.0 3.3 3.6 3.7 3.9y1i 7.0 6.9 6.7 6.5 6.3 5.4 5.4 5.3 5.2 5.1 5.0 4.9 4.8 4.6 4.4 4.3 4.2x1i 4.0 4.1 4.2 4.4 4.8 5.4 5.8 6.1 6.3 7.3 7.4 8.5 8.6y1i 4.0 3.9 3.7 3.4 3.2 2.9 2.8 2.8 2.7 2.5 2.3 2.1 2.0

x2i 1.4 1.4 1.5 1.6 1.7 1.8 1.9 2.0 2.1 2.3 2.5 2.6 2.6 2.7 2.7 2.9 3.0y2i 10.0 9.4 9.3 9.4 8.6 8.5 8.4 8.2 8.1 7.9 7.5 7.0 6.9 6.3 6.2 5.9 5.3x2i 3.3 3.5 3.6 3.7 3.9 4.2 4.6 4.8 4.9 5.0 5.1 5.7 5.8 6.0 6.7 7.0y2i 5.1 4.7 4.2 3.7 3.4 2.8 2.5 2.1 1.8 1.6 1.3 1.0 0.5 0.0 -0.5 -1.0

Page 130: Wahrscheinlichkeitstheorie undStatistik (wst)web.fhnw.ch/.../Stochastik-Skript-Marcel-Steiner.pdf · Anwendungsgebiete der Statistik sind die Industrie und die Technik, die Physik,

122 Kapitel 11. Regressionsanalyse

Page 131: Wahrscheinlichkeitstheorie undStatistik (wst)web.fhnw.ch/.../Stochastik-Skript-Marcel-Steiner.pdf · Anwendungsgebiete der Statistik sind die Industrie und die Technik, die Physik,

Anhang A

Mathematische Hilfsmittel

A.1 Rechnen mit dem Summenzeichen

In der Statistik und in der Wahrscheinlichkeitsrechnung treffen wir oft auf Zahlenmengen,zum Beispiel in Form von Datensatzen, die zum Teil sehr viele Elemente enthalten konnen.Diese wollen wir effizient verarbeiten konnen. Die Einzeldaten konnen wir uns in jedem Fallnummeriert vorstellen, sie bilden also eine Zahlenfolge. Die Nummerierung erfolgt mit dennaturlichen Zahlen N. Jedes Element erhalt zu seiner Nummerierung eine Ordnungszahl alsIndex

ai mit i ∈ 1, . . . , n.

Im Falle einer endlichen Stichprobe bleibt n dabei endlich. Es kann aber auch vorkommen,dass die Zahlenfolge auch unendlich lang sein kann, in diesem Fall haben wir n → +∞.

Zur Verarbeitung des Datenmaterials benotigen wir haufig nicht nur die Elemente der Zah-lenfolge einzeln, sondern auch die Gesamtsumme aller Elemente oder die Summe all ihrerQuadrate. Um die Schreibarbeit zu vereinfachen, verwenden wir fur solche Summen endlicherReihen das Summenzeichen

n∑

i=1

ai = a1 + a2 + · · ·+ ai−1 + ai + ai+1 + · · ·+ an−1 + an

= a1 + · · · + an.

Dabei sprechen wir von der Summe aller ai von i = 1 bis n. Der Index i ist dabei eineLaufvariable, die nur innerhalb der Summe Bedeutung hat, genau gleich wie die Integrations-variable x in einem bestimmten Integral.

Beispiel A.1.1.

a.n∑

i=1

1 = 1 + 1 + · · ·+ 1 = n

b. Es gilt die Formel

S =n∑

i=1

i = 1 + 2 + · · · + n =1

2n(n+ 1).

123

Page 132: Wahrscheinlichkeitstheorie undStatistik (wst)web.fhnw.ch/.../Stochastik-Skript-Marcel-Steiner.pdf · Anwendungsgebiete der Statistik sind die Industrie und die Technik, die Physik,

124 Anhang A. Mathematische Hilfsmittel

Beweis. Wir schreiben

1 + 2 + 3 + · · · + (n − 1) + n = S

+ n + (n− 1) + (n− 2) + · · · + 2 + 1 = S

(n+ 1) + (n+ 1) + (n+ 1) + · · · + (n + 1) + (n+ 1) = 2S

Also folgt 2S = n(n+ 1) und damit die Behauptung.

c. Es giltn∑

i=1

i2 = 12 + 22 · · ·+ n2 =1

6n(n+ 1)(2n + 1).

Der Beweis dieser Formel gelingt mittels einer vollstandigen Induktion, siehe Aufga-be A.1.3. Fur eine explizite Herleitung siehe [14], Summenformeln spezieller endlicherReihen.

Rechenregeln fur endliche Summen

Es sei k eine reelle Konstante, dann gilt

a.n∑

i=1

k = nk,

b.n∑

i=1

kai = k

n∑

i=1

ai,

c.n∑

i=1

(ai + bi) =n∑

i=1

ai +n∑

i=1

bi und

d.n∑

i=1

ai+1 −n∑

i=1

ai = an+1 − a1. Es handelt sich dabei um eine Teleskopsumme.

Aufgaben

Aufgabe A.1.1. Schreiben Sie folgende Reihen mit dem Summensymbol.

a. 14 + 24 + 34 + 44 + 54 + 64 + 74

b. 20 + 21 + 22 + 23 + 24 + 25

c. 1 + 12 +

13 + 1

4 + 15 +

16 + 1

7 + 18

d. 12 − 2

3 +34 − 4

5 + 56 − 6

7

Aufgabe A.1.2. Schreiben Sie gliedweise und berechnen Sie.

a.6∑

i=2

2i

Page 133: Wahrscheinlichkeitstheorie undStatistik (wst)web.fhnw.ch/.../Stochastik-Skript-Marcel-Steiner.pdf · Anwendungsgebiete der Statistik sind die Industrie und die Technik, die Physik,

A.2. Die Regeln von de Morgan 125

b.10∑

i=6

i2

c.9∑

i=1

(−1)i

d.

6∑

k=1

cos

(kπ

6

)

Aufgabe A.1.3. Beweisen Sie die Formel (c) in Beispiel A.1.1 mit vollstandiger Induktion.

Aufgabe A.1.4. Beweisen Sie die Rechenregeln fur endliche Summen.

Aufgabe A.1.5. Beweisen Sie die Formel

n∑

i=1

(λai + µbi)2 = λ2

n∑

i=1

a2i + 2λµ

n∑

i=1

aibi + µ2n∑

i=1

b2i .

Losungen

Losung A.1.1.

a.∑7

i=1 i4

b.∑6

i=1 2i−1

c.∑8

i=11i

d.∑6

i=1(−1)i+1 ii+1

Losung A.1.2. a. 40

b. 330

c. −1

d. −1

Losung A.1.3. Vergleichen Sie das entsprechende Kapitel in [14].

Losung A.1.4. Summen ausschreiben, neu ordnen und zusammenfassen – fertig.

Losung A.1.5. Summen ausschreiben, neu ordnen und zusammenfassen – fertig.

A.2 Die Regeln von de Morgan

Es seien A und B zwei Ereignisse (Mengen), dann gelten die Regeln von de Morgan:

A ∪B = A ∩B

A ∩B = A ∪B

Page 134: Wahrscheinlichkeitstheorie undStatistik (wst)web.fhnw.ch/.../Stochastik-Skript-Marcel-Steiner.pdf · Anwendungsgebiete der Statistik sind die Industrie und die Technik, die Physik,

126 Anhang A. Mathematische Hilfsmittel

Aufgabe

Aufgabe A.2.1. Beweisen Sie die Regeln von de Morgan.

Losung

Losung A.2.1. Der Beweis lauft grafisch, indem Sie die Mengen A und B in allgemeinerLage zeichnen.

A.3 Die Gausssche Fehlerfunktion

Es gibt Funktionen, die keine elementaren Stammfunktionen besitzen. Das beruhmteste Bei-spiel ist sicher die Wahrscheinlichkeitsdichte der (standardisierten) Normalverteilung

f(z) = ϕ(z, 0, 1) =1√2π

e−z2

2 fur −∞ < z < ∞.

Gleichwohl ist es moglich das bestimmte Integral

I =

∫ ∞

0e−u2

du =

√π

2. (1.3.a)

zu berechnen.In der Tat: Wir betrachten

I2 =

∫ ∞

0e−u2

du ·∫ ∞

0e−u2

du

=

∫ ∞

0e−u2

du ·∫ ∞

0e−v2 dv

=

∫ ∞

0

∫ ∞

0e−u2

e−v2 dudv

=

∫ ∞

0

∫ ∞

0e−(u2+v2) dudv.

Dieses Doppelintegral lasst sich nun mit der Substitution u = r cos(ϕ) und u = r sin(ϕ) losendas heisst, wir verwenden Polarkoordinaten. Dabei mussen wir beachten, dass dudv = rdrdϕ(vgl. Analysis IV, [14]. Also folgt

I2 =

∫ ∞

0

∫ ∞

0e−(u2+v2) dudv

=

∫ π2

0

∫ ∞

0e−r2 rdrdϕ

=

∫ π2

0−1

2e−r2

∣∣∣∣∞

0

=

∫ π2

0

1

2dϕ

4.

Page 135: Wahrscheinlichkeitstheorie undStatistik (wst)web.fhnw.ch/.../Stochastik-Skript-Marcel-Steiner.pdf · Anwendungsgebiete der Statistik sind die Industrie und die Technik, die Physik,

A.3. Die Gausssche Fehlerfunktion 127

Damit folgt sofort, dass I =√

π4 =

√π2 .

Ein weiteres Integral, die so genannte Gausssche Fehlerfunktion1

erf(x) =1√π

∫ x

−x

e−u2du =

2√π

∫ x

0e−u2

du (1.3.b)

treffen wir auch haufig an. Wir bemerken, dass im Exponenten des Integranden der Fak-tor 1

2 fehlt. Mit der Substitution u = 1√2z wird der Faktor wieder eingefuhrt, aber dabei

transformiert sich auch das Argument der Funktion.Der Zusammenhang mit der Verteilungsfunktion der Standardnormalverteilung

F (z) = Φ(z, 0, 1) =1√2π

∫ z

−∞e−

u2

2 du

ist durch

Φ(z, 0, 1) =1

2+

1

2erf

(1√2z

)

respektiveerf(x) = 2Φ(

√2x, 0, 1) − 1

gegeben.

1erf steht fur das engl. error f unction: Fehlerfunktion

Page 136: Wahrscheinlichkeitstheorie undStatistik (wst)web.fhnw.ch/.../Stochastik-Skript-Marcel-Steiner.pdf · Anwendungsgebiete der Statistik sind die Industrie und die Technik, die Physik,

128 Anhang A. Mathematische Hilfsmittel

Page 137: Wahrscheinlichkeitstheorie undStatistik (wst)web.fhnw.ch/.../Stochastik-Skript-Marcel-Steiner.pdf · Anwendungsgebiete der Statistik sind die Industrie und die Technik, die Physik,

Tafeln

T.1 Verteilungsfunktion Φ(z, 0, 1) der standardisierten Normalverteilung . . . . 130T.2 Quantile zq der standardisierten Normalverteilung . . . . . . . . . . . . . . 131T.3 Quantile tn,q der Student-t-Verteilung mit n Freiheitsgraden . . . . . . . . . 132

Die nachfolgenden Tafeln wurden alle mit Hilfe von Excel berechnet.

Aufgabe

Aufgabe T.1. Fertigen Sie selber mit Hilfe eines Computerprogrammes (z.B. Excel) dienachfolgenden Tafeln an.

129

Page 138: Wahrscheinlichkeitstheorie undStatistik (wst)web.fhnw.ch/.../Stochastik-Skript-Marcel-Steiner.pdf · Anwendungsgebiete der Statistik sind die Industrie und die Technik, die Physik,

130 Tafeln

0 z

Φ(z, 0, 1)

z

ϕ(z, 0, 1)

Tabelle T.1: Verteilung Φ(z, 0, 1) der standardisierten Normalverteilung N (0, 1)

z 0.00 0.01 0.02 0.03 0.04 0.05 0.06 0.07 0.08 0.090.0 0.5000 0.5040 0.5080 0.5120 0.5160 0.5199 0.5239 0.5279 0.5319 0.53590.1 0.5398 0.5438 0.5478 0.5517 0.5557 0.5596 0.5636 0.5675 0.5714 0.57530.2 0.5793 0.5832 0.5871 0.5910 0.5948 0.5987 0.6026 0.6064 0.6103 0.61410.3 0.6179 0.6217 0.6255 0.6293 0.6331 0.6368 0.6406 0.6443 0.6480 0.65170.4 0.6554 0.6591 0.6628 0.6664 0.6700 0.6736 0.6772 0.6808 0.6844 0.6879

0.5 0.6915 0.6950 0.6985 0.7019 0.7054 0.7088 0.7123 0.7157 0.7190 0.72240.6 0.7257 0.7291 0.7324 0.7357 0.7389 0.7422 0.7454 0.7486 0.7517 0.75490.7 0.7580 0.7611 0.7642 0.7673 0.7704 0.7734 0.7764 0.7794 0.7823 0.78520.8 0.7881 0.7910 0.7939 0.7967 0.7995 0.8023 0.8051 0.8078 0.8106 0.81330.9 0.8159 0.8186 0.8212 0.8238 0.8264 0.8289 0.8315 0.8340 0.8365 0.8389

1.0 0.8413 0.8438 0.8461 0.8485 0.8508 0.8531 0.8554 0.8577 0.8599 0.86211.1 0.8643 0.8665 0.8686 0.8708 0.8729 0.8749 0.8770 0.8790 0.8810 0.88301.2 0.8849 0.8869 0.8888 0.8907 0.8925 0.8944 0.8962 0.8980 0.8997 0.90151.3 0.9032 0.9049 0.9066 0.9082 0.9099 0.9115 0.9131 0.9147 0.9162 0.91771.4 0.9192 0.9207 0.9222 0.9236 0.9251 0.9265 0.9279 0.9292 0.9306 0.9319

1.5 0.9332 0.9345 0.9357 0.9370 0.9382 0.9394 0.9406 0.9418 0.9429 0.94411.6 0.9452 0.9463 0.9474 0.9484 0.9495 0.9505 0.9515 0.9525 0.9535 0.95451.7 0.9554 0.9564 0.9573 0.9582 0.9591 0.9599 0.9608 0.9616 0.9625 0.96331.8 0.9641 0.9649 0.9656 0.9664 0.9671 0.9678 0.9686 0.9693 0.9699 0.97061.9 0.9713 0.9719 0.9726 0.9732 0.9738 0.9744 0.9750 0.9756 0.9761 0.9767

2.0 0.9772 0.9778 0.9783 0.9788 0.9793 0.9798 0.9803 0.9808 0.9812 0.98172.1 0.9821 0.9826 0.9830 0.9834 0.9838 0.9842 0.9846 0.9850 0.9854 0.98572.2 0.9861 0.9864 0.9868 0.9871 0.9875 0.9878 0.9881 0.9884 0.9887 0.98902.3 0.9893 0.9896 0.9898 0.9901 0.9904 0.9906 0.9909 0.9911 0.9913 0.99162.4 0.9918 0.9920 0.9922 0.9925 0.9927 0.9929 0.9931 0.9932 0.9934 0.9936

2.5 0.9938 0.9940 0.9941 0.9943 0.9945 0.9946 0.9948 0.9949 0.9951 0.99522.6 0.9953 0.9955 0.9956 0.9957 0.9959 0.9960 0.9961 0.9962 0.9963 0.99642.7 0.9965 0.9966 0.9967 0.9968 0.9969 0.9970 0.9971 0.9972 0.9973 0.99742.8 0.9974 0.9975 0.9976 0.9977 0.9977 0.9978 0.9979 0.9979 0.9980 0.99812.9 0.9981 0.9982 0.9982 0.9983 0.9984 0.9984 0.9985 0.9985 0.9986 0.9986

3.0 0.9987 0.9987 0.9987 0.9988 0.9988 0.9989 0.9989 0.9989 0.9990 0.99903.1 0.9990 0.9991 0.9991 0.9991 0.9992 0.9992 0.9992 0.9992 0.9993 0.99933.2 0.9993 0.9993 0.9994 0.9994 0.9994 0.9994 0.9994 0.9995 0.9995 0.99953.3 0.9995 0.9995 0.9995 0.9996 0.9996 0.9996 0.9996 0.9996 0.9996 0.99973.4 0.9997 0.9997 0.9997 0.9997 0.9997 0.9997 0.9997 0.9997 0.9997 0.9998

3.5 0.9998 0.9998 0.9998 0.9998 0.9998 0.9998 0.9998 0.9998 0.9998 0.99983.6 0.9998 0.9998 0.9999 0.9999 0.9999 0.9999 0.9999 0.9999 0.9999 0.99993.7 0.9999 0.9999 0.9999 0.9999 0.9999 0.9999 0.9999 0.9999 0.9999 0.99993.8 0.9999 0.9999 0.9999 0.9999 0.9999 0.9999 0.9999 0.9999 0.9999 0.99993.9 1.0000 1.0000 1.0000 1.0000 1.0000 1.0000 1.0000 1.0000 1.0000 1.0000

Page 139: Wahrscheinlichkeitstheorie undStatistik (wst)web.fhnw.ch/.../Stochastik-Skript-Marcel-Steiner.pdf · Anwendungsgebiete der Statistik sind die Industrie und die Technik, die Physik,

Tafeln 131

0 zq

q

1 − q

z

ϕ(z, 0, 1)

Tabelle T.2: Die q-Quantile zq der standardisierten Normalverteilung N (0, 1). Es giltz1−q = −zq.

q 0.000 0.001 0.002 0.003 0.004 0.005 0.006 0.007 0.008 0.0090.50 0.000 0.003 0.005 0.008 0.010 0.013 0.015 0.018 0.020 0.0230.51 0.025 0.028 0.030 0.033 0.035 0.038 0.040 0.043 0.045 0.0480.52 0.050 0.053 0.055 0.058 0.060 0.063 0.065 0.068 0.070 0.0730.53 0.075 0.078 0.080 0.083 0.085 0.088 0.090 0.093 0.095 0.0980.54 0.100 0.103 0.105 0.108 0.111 0.113 0.116 0.118 0.121 0.1230.55 0.126 0.128 0.131 0.133 0.136 0.138 0.141 0.143 0.146 0.1480.56 0.151 0.154 0.156 0.159 0.161 0.164 0.166 0.169 0.171 0.1740.57 0.176 0.179 0.181 0.184 0.187 0.189 0.192 0.194 0.197 0.1990.58 0.202 0.204 0.207 0.210 0.212 0.215 0.217 0.220 0.222 0.2250.59 0.228 0.230 0.233 0.235 0.238 0.240 0.243 0.246 0.248 0.251

0.60 0.253 0.256 0.259 0.261 0.264 0.266 0.269 0.272 0.274 0.2770.61 0.279 0.282 0.285 0.287 0.290 0.292 0.295 0.298 0.300 0.3030.62 0.305 0.308 0.311 0.313 0.316 0.319 0.321 0.324 0.327 0.3290.63 0.332 0.335 0.337 0.340 0.342 0.345 0.348 0.350 0.353 0.3560.64 0.358 0.361 0.364 0.366 0.369 0.372 0.375 0.377 0.380 0.3830.65 0.385 0.388 0.391 0.393 0.396 0.399 0.402 0.404 0.407 0.4100.66 0.412 0.415 0.418 0.421 0.423 0.426 0.429 0.432 0.434 0.4370.67 0.440 0.443 0.445 0.448 0.451 0.454 0.457 0.459 0.462 0.4650.68 0.468 0.470 0.473 0.476 0.479 0.482 0.485 0.487 0.490 0.4930.69 0.496 0.499 0.502 0.504 0.507 0.510 0.513 0.516 0.519 0.522

0.70 0.524 0.527 0.530 0.533 0.536 0.539 0.542 0.545 0.548 0.5500.71 0.553 0.556 0.559 0.562 0.565 0.568 0.571 0.574 0.577 0.5800.72 0.583 0.586 0.589 0.592 0.595 0.598 0.601 0.604 0.607 0.6100.73 0.613 0.616 0.619 0.622 0.625 0.628 0.631 0.634 0.637 0.6400.74 0.643 0.646 0.650 0.653 0.656 0.659 0.662 0.665 0.668 0.6710.75 0.674 0.678 0.681 0.684 0.687 0.690 0.693 0.697 0.700 0.7030.76 0.706 0.710 0.713 0.716 0.719 0.722 0.726 0.729 0.732 0.7360.77 0.739 0.742 0.745 0.749 0.752 0.755 0.759 0.762 0.765 0.7690.78 0.772 0.776 0.779 0.782 0.786 0.789 0.793 0.796 0.800 0.8030.79 0.806 0.810 0.813 0.817 0.820 0.824 0.827 0.831 0.834 0.838

0.80 0.842 0.845 0.849 0.852 0.856 0.860 0.863 0.867 0.871 0.8740.81 0.878 0.882 0.885 0.889 0.893 0.896 0.900 0.904 0.908 0.9120.82 0.915 0.919 0.923 0.927 0.931 0.935 0.938 0.942 0.946 0.9500.83 0.954 0.958 0.962 0.966 0.970 0.974 0.978 0.982 0.986 0.9900.84 0.994 0.999 1.003 1.007 1.011 1.015 1.019 1.024 1.028 1.0320.85 1.036 1.041 1.045 1.049 1.054 1.058 1.063 1.067 1.071 1.0760.86 1.080 1.085 1.089 1.094 1.098 1.103 1.108 1.112 1.117 1.1220.87 1.126 1.131 1.136 1.141 1.146 1.150 1.155 1.160 1.165 1.1700.88 1.175 1.180 1.185 1.190 1.195 1.200 1.206 1.211 1.216 1.2210.89 1.227 1.232 1.237 1.243 1.248 1.254 1.259 1.265 1.270 1.276

0.90 1.282 1.287 1.293 1.299 1.305 1.311 1.317 1.323 1.329 1.3350.91 1.341 1.347 1.353 1.359 1.366 1.372 1.379 1.385 1.392 1.3980.92 1.405 1.412 1.419 1.426 1.433 1.440 1.447 1.454 1.461 1.4680.93 1.476 1.483 1.491 1.499 1.506 1.514 1.522 1.530 1.538 1.5460.94 1.555 1.563 1.572 1.580 1.589 1.598 1.607 1.616 1.626 1.6350.95 1.645 1.655 1.665 1.675 1.685 1.695 1.706 1.717 1.728 1.7390.96 1.751 1.762 1.774 1.787 1.799 1.812 1.825 1.838 1.852 1.8660.97 1.881 1.896 1.911 1.927 1.943 1.960 1.977 1.995 2.014 2.0340.98 2.054 2.075 2.097 2.120 2.144 2.170 2.197 2.226 2.257 2.2900.99 2.326 2.366 2.409 2.457 2.512 2.576 2.652 2.748 2.878 3.090

Page 140: Wahrscheinlichkeitstheorie undStatistik (wst)web.fhnw.ch/.../Stochastik-Skript-Marcel-Steiner.pdf · Anwendungsgebiete der Statistik sind die Industrie und die Technik, die Physik,

132 Tafeln

0 tn,q

q

1 − q

t

fn(t)

Tabelle T.3: q-Quantile tn,q der Student-t-Verteilung mit n Freiheitsgraden. Da die Dichtesymmetrisch ist, gilt tn,1−q = −tn,q.

q

n 0.9000 0.9500 0.9750 0.9900 0.9950 0.9990 0.9995 n

1 3.078 6.314 12.706 31.821 63.656 318.289 636.578 12 1.886 2.920 4.303 6.965 9.925 22.328 31.600 23 1.638 2.353 3.182 4.541 5.841 10.214 12.924 34 1.533 2.132 2.776 3.747 4.604 7.173 8.610 45 1.476 2.015 2.571 3.365 4.032 5.894 6.869 56 1.440 1.943 2.447 3.143 3.707 5.208 5.959 67 1.415 1.895 2.365 2.998 3.499 4.785 5.408 78 1.397 1.860 2.306 2.896 3.355 4.501 5.041 89 1.383 1.833 2.262 2.821 3.250 4.297 4.781 9

10 1.372 1.812 2.228 2.764 3.169 4.144 4.587 1011 1.363 1.796 2.201 2.718 3.106 4.025 4.437 1112 1.356 1.782 2.179 2.681 3.055 3.930 4.318 1213 1.350 1.771 2.160 2.650 3.012 3.852 4.221 1314 1.345 1.761 2.145 2.624 2.977 3.787 4.140 1415 1.341 1.753 2.131 2.602 2.947 3.733 4.073 1516 1.337 1.746 2.120 2.583 2.921 3.686 4.015 1617 1.333 1.740 2.110 2.567 2.898 3.646 3.965 1718 1.330 1.734 2.101 2.552 2.878 3.610 3.922 1819 1.328 1.729 2.093 2.539 2.861 3.579 3.883 19

20 1.325 1.725 2.086 2.528 2.845 3.552 3.850 2021 1.323 1.721 2.080 2.518 2.831 3.527 3.819 2122 1.321 1.717 2.074 2.508 2.819 3.505 3.792 2223 1.319 1.714 2.069 2.500 2.807 3.485 3.768 2324 1.318 1.711 2.064 2.492 2.797 3.467 3.745 2425 1.316 1.708 2.060 2.485 2.787 3.450 3.725 2526 1.315 1.706 2.056 2.479 2.779 3.435 3.707 2627 1.314 1.703 2.052 2.473 2.771 3.421 3.689 2728 1.313 1.701 2.048 2.467 2.763 3.408 3.674 2829 1.311 1.699 2.045 2.462 2.756 3.396 3.660 29

30 1.310 1.697 2.042 2.457 2.750 3.385 3.646 3040 1.303 1.684 2.021 2.423 2.704 3.307 3.551 4050 1.299 1.676 2.009 2.403 2.678 3.261 3.496 5060 1.296 1.671 2.000 2.390 2.660 3.232 3.460 6070 1.294 1.667 1.994 2.381 2.648 3.211 3.435 7080 1.292 1.664 1.990 2.374 2.639 3.195 3.416 8090 1.291 1.662 1.987 2.368 2.632 3.183 3.402 90

100 1.290 1.660 1.984 2.364 2.626 3.174 3.390 100150 1.287 1.655 1.976 2.351 2.609 3.145 3.357 150200 1.286 1.653 1.972 2.345 2.601 3.131 3.340 200300 1.284 1.650 1.968 2.339 2.592 3.118 3.323 300400 1.284 1.649 1.966 2.336 2.588 3.111 3.315 400500 1.283 1.648 1.965 2.334 2.586 3.107 3.310 500600 1.283 1.647 1.964 2.333 2.584 3.104 3.307 600800 1.283 1.647 1.963 2.331 2.582 3.100 3.303 8001000 1.282 1.646 1.962 2.330 2.581 3.098 3.300 1000

∞ 1.282 1.645 1.960 2.326 2.576 3.090 3.291 ∞

Page 141: Wahrscheinlichkeitstheorie undStatistik (wst)web.fhnw.ch/.../Stochastik-Skript-Marcel-Steiner.pdf · Anwendungsgebiete der Statistik sind die Industrie und die Technik, die Physik,

Literaturverzeichnis

[1] K. Bosch, Großes Lehrbuch der Statistik, Oldenburg Verlag, Munchen, 1996.

[2] K. Bosch, Elementare Einfuhrung in die Wahrscheinlichkeitsrechnung, 7. Auflage,Vieweg Studium-Basiswissen, 2000.

[3] P. Gschwind, Stochastik, Skriptum FHBB, Version 00, 2000.

[4] T. Heim, Stochastik, Skriptum FHBB, 2003.

[5] A. Hoffmann, B. Marx und W. Vogt, Mathematik fur Ingenieure 2. Vektoranalysis,Integraltransformationen, Differenzialgleichungen, Stochastik - Theorie und Numerik,Pearson Studium, 2006.

[6] R. Ineichen, Stochastik - Einfuhrung in die elementare Statistik und Wahrscheinlich-keitsrechnung, 9. Auflage, Raeber-Verlag, Luzern, 1996.

[7] U. Krengel, Einfuhrung in die Wahrscheinlichkeitstheorie und Statistik, 6. Auflage,Vieweg Studium-Aufbaukurs Mathematik, 2000.

[8] E. Kreyszig, Statistische Methoden und ihre Anwendungen, Vandenhoeck & Ruprecht,6. Auflage, 1977.

[9] L. Papula, Mathematik fur Ingenieure und Naturwissenschaftler, Band 2, 10. Auflage,Viewegs Fachbucher der Technik, 2001.

[10] L. Papula, Mathematik fur Ingenieure und Naturwissenschaftler, Band 3, 4. Auflage,Viewegs Fachbucher der Technik, 2001.

[11] J. A. Rice, Mathematical Statistics and Data Analysis, 2nd Edition, Duxbury Press,1995.

[12] S. M. Ross, Initiation aux probabilites, Presses polytechniques et universitaires roman-des, 1999.

[13] W. A. Stahel, Statistische Datenanalyse, 4. Auflage, Viewegs Fachbucher der Technik,2002.

[14] M. Steiner-Curtis, Analysis I - IV, Skriptum FHNW, 2020.

[15] M. Steiner-Curtis, Datenanalyse, Skriptum FHNW, 2020.

[16] R. Storm, Wahrscheinlichkeitsrechnung, mathematische Statistik und statistische Qua-litatskontrolle, 11. Auflage, Fachbuchverlag Leipzig, 2001.

133

Page 142: Wahrscheinlichkeitstheorie undStatistik (wst)web.fhnw.ch/.../Stochastik-Skript-Marcel-Steiner.pdf · Anwendungsgebiete der Statistik sind die Industrie und die Technik, die Physik,

134 Literaturverzeichnis

[17] B. L. van der Warden, Mathematische Statistik, 3. Auflage, Springer-Verlag, Berlin,1973.

Page 143: Wahrscheinlichkeitstheorie undStatistik (wst)web.fhnw.ch/.../Stochastik-Skript-Marcel-Steiner.pdf · Anwendungsgebiete der Statistik sind die Industrie und die Technik, die Physik,

Index

Additionssatz, 36Alternativhypothese, 56, 81Approximation, 106arithmetische Mittel, 9Ausgleichs

-kurve, 106Ausreisser, 10, 15Ausreissergrenzen, 15Axiome

der Wahrscheinlichkeit, 34

von Kolmogoroff, 34

Bezier-Kurven, 53Bar Chart, 7Behrens-Fisher-Problem, 99Bernstein Polynome, 53BINOMVERT, 51Boxplot, 2, 15

Carl Friedrich Gauss, 1777-1855, 74, 103Cramersche Regel, 104

Datendiskrete, 5metrische, 5qualitative, 5stetige, 5

Datensatz, 6Durchschnittsmenge, 18

Einstichproben-t-Test, 90Ereignis, 18

sicheres, 18unmogliches, 18

Ereignisbaum, 22Error Function, 127

Erwartungswert, 49Binomialverteilung, 55diskret, 49Normalverteilung, 71

Poissonverteilung, 60

stetig, 67stetige Zufallsgrosse, 67

Student-t-Verteilung, 93

erzeugende Funktion, 54

Excel

BINOMVERT, 51

NORMINV, 73

NORMVERT, 68

POISSON, 60

STANDNORMVERT, 68

Fehler1. Art, 85

2. Art, 85

grobe, 74

systematische, 74

zufallige, 74

Fehlerfunktion, 127

Fehlerkurve, 73

Fehlerquadratsumme, 103, 106, 115

Formparameter, 13

Freiheitsgrade

Student-t-Test, 91

Student-t-Verteilung, 92

Galton Francis, 1822-1911, 78

Galtonsches Brett, 78

Gammafunktion, 93

Gausssche

Fehlerfunktion, 127

Glockenkurve, 70

Normalverteilung, 68

Gegenereignis, 19

geometrisches Mittel, 10

Gleichungssystem

lineares, 107

Gleichverteilung, 50, 64

Gosset William Sealey, 1876-1937, 94

135

Page 144: Wahrscheinlichkeitstheorie undStatistik (wst)web.fhnw.ch/.../Stochastik-Skript-Marcel-Steiner.pdf · Anwendungsgebiete der Statistik sind die Industrie und die Technik, die Physik,

136 Index

Grenzwertsatzvon de Moivre und Laplace, 77

zentraler, 75Grundgesamtheit, 2, 3

Haufigkeitstabellen, 6

Haufigkeitsverteilung, 6harmonisches Mittel, 10

heteroskedastisch, 99Histogramm, 7homoskedastisch, 97

Hypothese, 56, 81Hypothesentest, 56

Intervallschatzungen, 94

Irrtumswahrscheinlichkeit, 57, 82

Klassenbreite, 7Klasseneinteilung, 7

Kolmogoroff N. Andrey, 1903-1987, 34Kombinationen, 24

Komplementarmenge, 19Konfidenzintervall

Einstichproben-t-Test, 94

Erwartungswert, 94Student-t-Test, 94

Kreisdiagramm, 8

Lageparameter, 2, 9

Macht eines Tests, 86

Median, 10Methode der kleinsten Quadrate, 11, 103,

115Mittel

arithmetische, 9geometrisches, 10harmonisches, 10

Mittelwert, 9geschatzt, 90, 97, 99

MKQ, 103, 115Monte-Carlo-Methoden, 34Multiplikationssatz, 37, 38

Normalgleichungssystem, 108Normalverteilung, 68

standardisierte, 68, 71

Transformation, 71

NORMINV, 73NORMVERT, 68Nullhypothese, 56, 81

oder, 18

Operationscharakteristik, 87Optimumseigenschaften, 11

Paarweiser Vergleichzweier Mittelwerte, 100

Permutationen, 23

permutieren, 23Perzentil, 72q-Perzentil, 72Pie Chart, 8POISSON, 60

Poissonverteilung, 59Prufen

von Erwartungswerten, 89Produktregel, 22

Quantil, 72einseitig, 73

Student-t-Verteilung, 93zweiseitig, 73

q-Quantil, 72Quartile, 10, 15Quartilsweite, 15

Regelvon Cramer, 104

Regeln von de Morgan, 125Regression, 114Regressions

-funktion, 106-gerade, 103, 114, 115-koeffizient, 105, 116-konstante, 105, 116

Regressionsanalyse, 113

einer Geraden, 116zweier Geraden, 119

Regressionsrechnung, 103Remis, 21Restvarianz, 117

robust, 10, 91

Saulendiagramm, 7Schach

Page 145: Wahrscheinlichkeitstheorie undStatistik (wst)web.fhnw.ch/.../Stochastik-Skript-Marcel-Steiner.pdf · Anwendungsgebiete der Statistik sind die Industrie und die Technik, die Physik,

Index 137

Remis, 21σ-Bereich, 69

Signifikanzniveau, 57, 82Signifikanztest, 57, 82

Simulation, 32Spannweite, 13Standardabweichung, 13, 49

Binomialverteilung, 55Poissonverteilung, 61

standardisierte Normalverteilung, 68STANDNORMVERT, 68Statistik

beschreibende, 5deskriptive, 5

induktive, 3schliessende, 3

statistischen Tests, 57, 82

Statistischer Test, 81statistischer Test, 56

Stetigkeitskorrektur, 77Stichprobe, 2, 3

zufallige, 4

Stichprobenraum, 17Stichprobenumfang, 2

erforderlicher, 96Stochastik, 1Streuung

Binomialverteilung, 55diskret, 49

diskrete Zufallsgrosse, 49Normalverteilung, 71Poissonverteilung, 60

stetig, 67stetige Zufallsgrosse, 67

Student-t-Verteilung, 93Student-t-Test, 90Student-t-Verteilung, 92

Summenfunktion, 44Summenzeichen, 123

vertauschen, 108

t-Verteilung, 92

Tafeln, 128Quantile der standardisierten Normal-

verteilung, 131standardisierte Normalverteilung, 130

Student-t-Verteilung, 132

Teleskopsumme, 124

Test, 56Einstichproben-t-, 90

t-, 90

z-, 85Zweistichproben-t-, 96

Testen einer Hypothese, 56

Testgrosse, 82Regressionsanalyse, 117

Regressionskoeffizienten, 120

Regressionskonstanten, 120Student-t-Test, 90

Trennscharfe eines Tests, 86

und, 18Ungleichung

Cauchy-Schwarz, 105

Variablenichtstochastisch, 113

stochastisch, 114

VarianzBinomialverteilung, 55

diskret, 49

diskrete Zufallsgrosse, 49geschatzt, 90, 97, 99

gewogene Mittel der, 97

Normalverteilung, 71

Poissonverteilung, 60stetig, 67

stetige Zufallsgrosse, 67

Student-t-Verteilung, 93Variation, 24

Vereinigungsmenge, 18

VerteilungBinomial-, 51

Gleich-, 64

Normal-, 68Poisson-, 59

Student-t, 92

t-, 92Verteilungsfunktion, 44, 64

Vertrauensintervall

Einstichproben-t-Test, 94Erwartungswert, 94

Student-t-Test, 94

Vertrauenswahrscheinlichkeit, 95

Page 146: Wahrscheinlichkeitstheorie undStatistik (wst)web.fhnw.ch/.../Stochastik-Skript-Marcel-Steiner.pdf · Anwendungsgebiete der Statistik sind die Industrie und die Technik, die Physik,

138 Index

Viertelwerte, 11

Wahrscheinlichkeit, 29, 43experimentelle, 32theoretische, 29

Wahrscheinlichkeitsdichte, 63, 64Wahrscheinlichkeitsverteilung, 33, 43Whiskersplot, 2, 15

z-Test, 85Zahlenlotto, 29Zentralwert, 11Zufall, 17Zufallsexperiment, 2, 17Zufallsgrosse, 43

diskret, 44stetig, 44, 63

Zufallsvariable, 43Zufallszahlen, 35zusammengesetzter Versuch, 21Zweistichproben-t-Test, 96

unbekannte Varianzen, 99unbekannten gleiche Varianzen, 97